LSAT PrepTest 81 Unlocked · Similarly, PT 81 was the first test since June '07 (PT June '07) to...

212

Transcript of LSAT PrepTest 81 Unlocked · Similarly, PT 81 was the first test since June '07 (PT June '07) to...

Page 1: LSAT PrepTest 81 Unlocked · Similarly, PT 81 was the first test since June '07 (PT June '07) to require at least 94 questions correct to get a 172 (99th percentile score). Number
Page 2: LSAT PrepTest 81 Unlocked · Similarly, PT 81 was the first test since June '07 (PT June '07) to require at least 94 questions correct to get a 172 (99th percentile score). Number

LSAT®

PREPTESTS81UNLOCKED

EXCLUSIVEDATA,ANALYSIS,&EXPLANATIONSFORTHEJUNE2017LSAT

LSAT®isaregisteredmarkoftheLawSchoolAdmissionCouncil,Inc.

Page 3: LSAT PrepTest 81 Unlocked · Similarly, PT 81 was the first test since June '07 (PT June '07) to require at least 94 questions correct to get a 172 (99th percentile score). Number

TableofContentsPrepTests81Unlocked:FrontMatter

Preptest81:CoverPreptest81:TitlePagePreptest81:Copyright

Preptest81:June2017Preptest81:Intro—BundleOnlyPreptest81:Unlocked:TheInsideStoryPreptest81:SectionI:ReadingComprehensionPreptest81:SectionII:LogicalReasoningPreptest81:SectionIII:LogicalReasoningPreptest81:SectionIV:LogicGames

GlossaryGlossaryIntroGlossary

Page 4: LSAT PrepTest 81 Unlocked · Similarly, PT 81 was the first test since June '07 (PT June '07) to require at least 94 questions correct to get a 172 (99th percentile score). Number

LSAT®isaregisteredmarkoftheLawSchoolAdmissionCouncil,Inc.,whichneithersponsorsnorendorsesthisproduct.

Thispublicationisdesignedtoprovideaccurateinformationinregardtothesubjectmattercoveredasofitspublicationdate,withtheunderstandingthatknowledgeandbestpracticeconstantlyevolve.Thepublisherisnotengagedinrenderingmedical,legal,accounting,orotherprofessionalservice.Ifmedicalorlegaladviceorotherexpertassistanceisrequired,theservicesofacompetentprofessionalshouldbesought.Thispublicationisnotintendedforuseinclinicalpracticeorthedeliveryofmedicalcare.Tothefullestextentofthelaw,neitherthePublishernortheEditorsassumeanyliabilityforanyinjuryand/ordamagetopersonsorpropertyarisingoutoforrelatedtoanyuseofthematerialcontainedinthisbook.

©2017byKaplan,Inc.

PublishedbyKaplanPublishing,adivisionofKaplan,Inc.750ThirdAvenueNewYork,NY10017

ISBN:978-1-5062-2342-110987654321

AllrightsreservedunderInternationalandPan-AmericanCopyrightConventions.Bypaymentoftherequiredfees,youhavebeengrantedthenon-exclusive,non-transferablerighttoaccessandreadthetextofthiseBookonscreen.Nopartofthistextmaybereproduced,transmitted,downloaded,decompiled,reverseengineered,orstoredinorintroducedintoanyinformationstorageandretrievalsystem,inanyformorbyanymeans,whetherelectronicormechanical,nowknownorhereinafterinvented,withouttheexpresswrittenpermissionofthepublisher.

Page 5: LSAT PrepTest 81 Unlocked · Similarly, PT 81 was the first test since June '07 (PT June '07) to require at least 94 questions correct to get a 172 (99th percentile score). Number

PREPTEST81

Page 6: LSAT PrepTest 81 Unlocked · Similarly, PT 81 was the first test since June '07 (PT June '07) to require at least 94 questions correct to get a 172 (99th percentile score). Number

PrepTest81:TheInsideStory

PREPTEST81:THEINSIDESTORY

PrepTest81wasadministeredinJune2017.Itchallenged27,606testtakers.Whatmadethistestsohard?Here'sabreakdownofwhatKaplanstudentswhoweresurveyedaftertakingtheofficialexamconsideredPrepTest81'smostdifficultsection.

Page 7: LSAT PrepTest 81 Unlocked · Similarly, PT 81 was the first test since June '07 (PT June '07) to require at least 94 questions correct to get a 172 (99th percentile score). Number

HARDESTPREPTEST81SECTIONASREPORTEDBYTESTTAKERS

Page 8: LSAT PrepTest 81 Unlocked · Similarly, PT 81 was the first test since June '07 (PT June '07) to require at least 94 questions correct to get a 172 (99th percentile score). Number

Basedontheseresults,youmightthinkthatstudyingLogicGamesisthekeytoLSATsuccess.Well,LogicGamesisimportant,buttesttakers'perceptionsdon'ttellthewholestory.Forthat,youneedtoconsiderstudents'actualperformance.Alas,PrepTest81issorecentthatasofthetimeofpublicationwedon'tyethavesufficientdataaboutstudentperformanceonthistest.***Youronlinematerialswillbeupdatedtoreflectperformancedatalaterinthefallof2017.***

Actualstudentperformancetypicallytellsquiteadifferentstory.Usuallystudentsarefairlyequallylikelytomissquestionsinallthreeofthedifferentsectiontypes.

Page 9: LSAT PrepTest 81 Unlocked · Similarly, PT 81 was the first test since June '07 (PT June '07) to require at least 94 questions correct to get a 172 (99th percentile score). Number

MaybestudentsoverestimatethedifficultyoftheLogicGamessectionbecauseit'ssounusual,ormaybeit'sbecauseareallyhardlogicgameissoeasytorememberafterthetest.Butthetruthisthatthetestmakerplaceshardquestionsthroughoutthetest.Whenweupdatetheinformationinfallof2017,we'llincludethelocationsofthe10hardest(mostmissed)questionsintheexam.

TomaximizeyourpotentialontheLSAT,youneedtotakeacomprehensiveapproach.Testyourselfrigorously,andreviewyourperformanceoneverysectionofthetest.Kaplan'sLSATexplanationsprovidetheexpertiseandinsightyouneedtofullyunderstandyourresults.TheexplanationsarewrittenandeditedbyateamofLSATexperts,whohavehelpedthousandsofstudentsimprovetheirscores.Kaplanalwaysprovidesdata-drivenanalysisofthetest,rankingthedifficultyofeveryquestionbasedonactualstudentperformance.Thetenhardestquestionsoneverytestarehighlightedwitha4-stardifficultyrating,thehighestwegive.Theanalysisbreaksdowntheremainingquestionsinto1-,2-,and3-starratingssothatyoucancompareyourperformancetothousandsofothertesttakersonallLSACmaterial.***Assoonaswegetsufficientdata,we'llupdatethestarratingsforPrepTest81infallof2017.***

7CAN'TMISSFEATURESOFPREPTEST81With10InferencequestionsinLR,PT81hadthemostInferencequestionssinceDecember'94(PT13).TheLogicGamessectionhasonlystartedwithaHybridgameninetimesever.Thatsaid,PT81wasthethirdtestinthreeyearstodoso.PT81'sReadingComprehensionsectioncontainedonlytwoGlobalquestions.ThattiesJune'16(PT78)asthefewestever.

Page 10: LSAT PrepTest 81 Unlocked · Similarly, PT 81 was the first test since June '07 (PT June '07) to require at least 94 questions correct to get a 172 (99th percentile score). Number

PREPTEST81INCONTEXTAsmuchfunasitistofindoutwhatmakesaPrepTestuniqueornoteworthy,it'sevenmoreimportanttoknowjusthowrepresentativeitisofotherLSATadministrations(and,thus,howlikelyitistoberepresentativeoftheexamyouwillfaceonTestDay).ThefollowingchartscomparethenumbersofeachkindofquestionandgameonPrepTest81totheaveragenumbersseenonallofficiallyreleasedLSATsadministeredoverthepastfiveyears(from2012through2016).

NumberofLRQuestionsbyType:PrepTest81vs.2012–2016Average

WhatreplacedthoseGlobalquestions?TherewerefourLR-ParallelReasoningquestionsintheRCsection.That'sanewrecord,andequalsthesamenumbertherewereonPT77–80combined!(D)-lightful!Therewereatleasttwomore(D)'sthan(C)'sor(E)'sinallthreesections!A/BTesting?ThesecondLRsectionhadsomeprettyunusualletteranswerstreaks.#7to#15werealleither(A)or(B)—includingfourstraight(B)'sinonestretch.DangerousCurveAhead!PT81wasthefirsttestsinceOctober'08(PT58)torequireatleast83questionscorrecttogeta164(90thpercentilescore).Similarly,PT81wasthefirsttestsinceJune'07(PTJune'07)torequireatleast94questionscorrecttogeta172(99thpercentilescore).

Page 11: LSAT PrepTest 81 Unlocked · Similarly, PT 81 was the first test since June '07 (PT June '07) to require at least 94 questions correct to get a 172 (99th percentile score). Number

NumberofLGGamesbyType:PrepTest81vs.2012–2016Average

Page 12: LSAT PrepTest 81 Unlocked · Similarly, PT 81 was the first test since June '07 (PT June '07) to require at least 94 questions correct to get a 172 (99th percentile score). Number

NumberofRCQuestionsbyType:PrepTest81vs.2012–2016Average

Page 13: LSAT PrepTest 81 Unlocked · Similarly, PT 81 was the first test since June '07 (PT June '07) to require at least 94 questions correct to get a 172 (99th percentile score). Number

Thereisn'tusuallyahugedifferenceinthedistributionofquestionsfromLSATtoLSAT,butifthistestseemsharder(oreasier)toyouthananotheryou'vetaken,comparethenumberofquestionsofthetypesonwhichyou,personally,arestrongestandweakest.Andthen,explorewithineachsectiontoseeifyourbestorworstquestiontypescameearlierorlater.

StudentsinKaplan'scomprehensiveLSATcourseshaveaccesstoeveryreleasedLSAT,andtoanonlineQbankwiththousandsofofficiallyreleasedquestions,games,andpassages.Ifyouarestudyingonyourown,youhavetodoabitmoreworktoidentifyyourstrengthsandyourareasofopportunity.Quantitativeanalysis(likethatinthechartsabove)isanimportanttoolforunderstandinghowthetestisconstructed,andhowyouareperformingonit.

Page 14: LSAT PrepTest 81 Unlocked · Similarly, PT 81 was the first test since June '07 (PT June '07) to require at least 94 questions correct to get a 172 (99th percentile score). Number

LSATPrepTest81

Page 15: LSAT PrepTest 81 Unlocked · Similarly, PT 81 was the first test since June '07 (PT June '07) to require at least 94 questions correct to get a 172 (99th percentile score). Number

SECTIONI:READINGCOMPREHENSION

Passages

Passage1:WyntonMarsalisandtheStateofJazz

Passage2:Inferentialvs.NoninferentialThoughts

Passage3:Dowsing

Passage4:TheUseofIndependentResearchbyJudges

PASSAGE1:WYNTONMARSALISANDTHESTATEOFJAZZ

Q# QuestionType Correct Difficulty

1 Global D Checkyouronlineresources.

2 Inference C Checkyouronlineresources.

3 Inference D Checkyouronlineresources.

4 LogicReasoning(ParallelReasoning) B Checkyouronlineresources.

5 Detail E Checkyouronlineresources.

6 Inference A Checkyouronlineresources.

7 Detail E Checkyouronlineresources.

Page 16: LSAT PrepTest 81 Unlocked · Similarly, PT 81 was the first test since June '07 (PT June '07) to require at least 94 questions correct to get a 172 (99th percentile score). Number

PASSAGE2:INFERENTIALVS.NONINFERENTIALTHOUGHTS

Q# QuestionType Correct Difficulty

8 Global B Checkyouronlineresources.

9 LogicReasoning(Weaken) C Checkyouronlineresources.

10 Inference E Checkyouronlineresources.

11 LogicReasoning(ParallelReasoning) A Checkyouronlineresources.

12 Detail D Checkyouronlineresources.

13 Detail C Checkyouronlineresources.

14 Inference D Checkyouronlineresources.

PASSAGE3:DOWSING

Q# QuestionType Correct Difficulty

15 LogicFunction C Checkyouronlineresources.

16 Detail A Checkyouronlineresources.

17 LogicReasoning(ParallelReasoning) D Checkyouronlineresources.

18 Inference B Checkyouronlineresources.

19 Detail D Checkyouronlineresources.

20 Inference E Checkyouronlineresources.

Page 17: LSAT PrepTest 81 Unlocked · Similarly, PT 81 was the first test since June '07 (PT June '07) to require at least 94 questions correct to get a 172 (99th percentile score). Number

PASSAGE4:THEUSEOFINDEPENDENTRESEARCHBYJUDGES

Q# QuestionType Correct Difficulty

21 LogicReasoning(Principle) C Checkyouronlineresources.

22 Inference A Checkyouronlineresources.

23 Inference D Checkyouronlineresources.

24 Inference B Checkyouronlineresources.

25 Inference C Checkyouronlineresources.

26 LogicReasoning(ParallelReasoning) B Checkyouronlineresources.

27 Inference D Checkyouronlineresources.

PASSAGE1:WYNTONMARSALISANDTHESTATEOFJAZZStep1:ReadthePassageStrategically

SampleRoadmap

Discussion

Inthefirstparagraph,theauthorintroducesastartlingcontrast:JazzmusicianWyntonMarsalis(Topic)waswidelypraisedfordecades,gaining"unqualifiedadmiration"andhaving"unsurpassedinfluence."Butthatchanged,ascriticismtowardMarsalisledtoanuncertainfateforbothhimandjazzmusic,ingeneral.

Page 18: LSAT PrepTest 81 Unlocked · Similarly, PT 81 was the first test since June '07 (PT June '07) to require at least 94 questions correct to get a 172 (99th percentile score). Number

ThesecondparagraphgoesintodetailaboutthedeteriorationofbothMarsalis'scareerandjazz.Afteramassiveoutput(15CDsinoneyear),Marsaliswentyearswithoutreleasingnewmusic.Henolongerhadarecordcontract,andrecordcompaniesstoppeddevelopingnewjazzartists.Itmakesthereaderwonder,whathappenedtoMarsalis'ssterlingreputation,andhowdidthisaffectjazzasawhole?ThesequestionsserveastheScopeofthepassage.AndthePurposeistoanswerthosequestionsandexplainwhathappened.

Thethirdparagraphstartstooffersomeexplanation,presentingthepointofviewofMarsalis'scritics.TheypartiallyblameMarsalisforhisunbendingandstiflingrelianceonclassicism,whichimpededinnovation.

TheauthordefendsMarsalissomeinthefourthparagraph,admittingtoMarsalis'semphasisontradition,butarguingthatMarsaliswasusingthattraditionasinspirationforreinventionandexpression.However,recordcompaniestookadifferentview:Iftraditionalmusicissogreat,thenwhoneedsnewmusic?

Andthatleadstotheultimateeffectdescribedinthelastparagraph:Recordcompaniesstoppedpushingnewartistsandinsteadfocusedonmakinglotsofmoneysellingarchivedrecordingsofolderartists.ThiswrapsuptheMainIdeaofthepassage:Despitehisinitialsuccess,Marsalis'semphasisontraditionaljazzstylesultimatelyhurthiscareerandledthemusicindustrytostopsupportingnewjazzmusicians.

1. (D)GlobalStep2:IdentifytheQuestionType

Thequestionasksforthe"mainpoint"oftheentirepassage,makingthisaGlobalquestion.

Page 19: LSAT PrepTest 81 Unlocked · Similarly, PT 81 was the first test since June '07 (PT June '07) to require at least 94 questions correct to get a 172 (99th percentile score). Number

Step3:ResearchtheRelevantText

Don'tgobackintothepassage.JustconsidertheMainIdeaaspredictedafterreadingthepassage.

Step4:MakeaPrediction

TheMainIdeawasthat,despitehisinitialsuccess,Marsalis'semphasisontraditionaljazzhurthiscareerandultimatelyledrecordcompaniestoabandontheirsupportfornewjazzmusicians.

Step5:EvaluatetheAnswerChoices

(D)offersanaccuratesummary,bringingupMarsalis'semphasisontraditionandhowthatledtodecreasedsupportfornewjazzartists.

(A)focusestoomuchonMarsalis,completelyignoringtheeffectonjazzmusicingeneral.

(B)addressestheauthor'sdefenseofMarsalisinthefourthparagraph.However,thisnotonlymissesthepointofthepassageasawhole,butitalsosuggeststhatjazznowhasawideraudience—aclaimnotsupportedanywhereinthepassage.

(C)isaDistortion.ItisneversuggestedthatMarsalisevermovedawayfromtraditionalism.And,ifanything,thepassagesuggeststhatMarsalis'sstylecausedtherecordcompaniestoshifttheirfocus,nottheotherwayaround.

(E)isfartoonarrow,focusingmerelyonwhatismentionedinthefirstparagraph.However,thiscompletelyleavesouttheeffectsdescribedthroughouttherestofthepassage.

Page 20: LSAT PrepTest 81 Unlocked · Similarly, PT 81 was the first test since June '07 (PT June '07) to require at least 94 questions correct to get a 172 (99th percentile score). Number

2. (C)InferenceStep2:IdentifytheQuestionType

Thequestionaskswhatsomeone"mostlikelymeans"whenmakingaclaim,makingthisanInferencequestion.

Step3:ResearchtheRelevantText

Thequoteinquestionispresentedattheendofthethirdparagraph.Don'tjustfocusonthequoteitself.Considerhowitrelatestothepointoftheparagraphasawhole.

Step4:MakeaPrediction

InsayingMarsalishasa"retroideology,"theexecutivealsoclaimsthatMarsalis'sideasweremore"museumlikeinnature,alookback."ThisfitsthecriticismthroughouttheparagraphthatMarsaliswasunbendinginhisfocusonclassicismandwasstiflinginitsorthodoxy.Inshort,theexecutiveissuggestingthatMarsaliswastoocaughtupinthepast.

Step5:EvaluatetheAnswerChoices

(C)correctlyidentifiestheideathatMarsaliswastoofocusedontradition.

(A)isaDistortion.Marsalisisrevivingtraditionalideas,butthereisnosuggestionthattheseideaswereeverdiscredited.

(B)isaFaultyUseofDetail.TheauthorreferencesMarsalisrecombiningideasinthefourthparagraph(line43),butthisisnotrelatedtotheexecutive'sclaim,whichismoreaboutMarsalis'sideasbeingoutdated(museumlike).

(D)isalsoaFaultyUseofDetail,anda180.It'stheauthorwhopraisesMarsalisforreinventionandreinterpretation(lines41–44),notthe

Page 21: LSAT PrepTest 81 Unlocked · Similarly, PT 81 was the first test since June '07 (PT June '07) to require at least 94 questions correct to get a 172 (99th percentile score). Number

executive.TheexecutiveismorecriticalandseesMarsalisasstuckinthepast,ratherthanbeinginventive.

(E)isaDistortion.ThereisnosuggestionthattheideasMarsalisusedwereinanywayinauthentic.

3. (D)InferenceStep2:IdentifytheQuestionType

Thequestionasksaboutwhattheauthorwould"mostlikely"believe,whichmeansthecorrectanswerwon'tbedirectlystated,butitwillbedirectlysupported.ThatmakesthisanInferencequestion.

Step3:ResearchtheRelevantText

Thequestionstemreferstothe"stateofaffairsinjazz,"aphrasethat(notcoincidentally)appearswordforwordinline24.Thephrase"thisgrimstateofaffairs"indicatestheauthorisreferringtothescenariodescribedimmediatelybeforehand,inthesecondparagraph.

Step4:MakeaPrediction

Thegrimsituationisthatrecordcompanieshave"reduced[their]rosterofactivejazzmusicians,"emphasized"reissuesofoldrecordings,"and"essentiallygaveupondevelopingnewartists."Thatsuggeststheauthorwouldbemuchhappierifrecordcompaniesstartingfocusingonnewjazzmusiciansagain.

Step5:EvaluatetheAnswerChoices

(D)issupported,asitwouldperfectlyaddressthede-emphasisonnewartists.

(A)isaDistortion.Theauthorwouldcertainlyappreciatethis,especiallygiventhedefenseofMarsalispresentedinthefourthparagraph.

Page 22: LSAT PrepTest 81 Unlocked · Similarly, PT 81 was the first test since June '07 (PT June '07) to require at least 94 questions correct to get a 172 (99th percentile score). Number

However,thequestionasksaboutwhatwouldmaketheauthorlessnegativeaboutthestateofaffairsinjazzmusicingeneral,notinthetreatmentofMarsalispersonally.

(B)istoofocusedonrevivingMarsalis'scareerratherthanimprovingthe"stateofaffairsinjazz,"asthequestionasks.

(C)isalsotoofocusedondefendingMarsalispersonally,ratherthanaddressingthestateofaffairsinjazzoverall.

(E)isaDistortion.Theauthordoesnothaveaproblemwithyoungjazzmusicians.Theproblemiswiththerecordcompanies,whoarebasicallyabandoningnewjazzmusicians.

4. (B)LogicReasoning(ParallelReasoning)Step2:IdentifytheQuestionType

Thequestionasksforasituationthatis"mostanalogous"toonedescribedinthepassage.ThatmakesthisaLogicReasoningquestion,specificallyonethatmimicsaParallelReasoningquestion.

Step3:ResearchtheRelevantText

ThesituationfacingMarsalisisdescribedthroughouttheentirepassage.Usethemarginnotestostayfocusedonthemajorthemes,ratherthangettingcaughtupinthedetails.

Step4:MakeaPrediction

Asdescribedthroughoutthepassage,Marsalisiscriticizedforfocusingtoomuchontraditionalideas,andthishasledmusiccompaniestoabandonnewmusiciansinfavorofsellingolderrecordings.Thecorrect

Page 23: LSAT PrepTest 81 Unlocked · Similarly, PT 81 was the first test since June '07 (PT June '07) to require at least 94 questions correct to get a 172 (99th percentile score). Number

answerwillmimicthisideaofhowone'sfocusontraditioncanleadotherstoabandonnewideasinfavorofsellingoldones.

Step5:EvaluatetheAnswerChoices

(B)isaperfectmatch.LikeMarsalis,theresearchfirmistryingtocreatenewproductsthataresimilartotraditionalproducts.However,thathasjustledtopeopleabandoningthenewproductsinfavorofseekingoutthetraditionalproducts.

(A)doesnotmatch.Here,theunintendedconsequenceiscomparativelyhigherpriceincreases.There'snofocusontraditionalstylesorabandonmentofnewideas.

(C)doesnotmatch.Here,thefocusisonpeoplefindingsyntheticproductslessattractive.There'snothingaboutMarsalis'ssituationthatsuggestspeoplearelessattractedtonewmusicthantooldmusic—therecordcompaniesarejustfocusingonmarketingoldmusic.

(D)doesnotmatch,andisa180atworst.Ifanything,Marsalisisfacingcompetitionfromestablishedcompaniesfocusedonprofitingfromarchivedrecordings,notsomeupstartswiththeirnewfangledideas.

(E)doesnotmatch.Inthisscenario,somebodytriestosaveonething(endangeredfish),andanewthingcomesalongtodestroyit.Marsalis,ontheotherhand,istryingtouseoldideastocreatesomethingnew,butit'stheoldideasthatthriveanddiminishthepresenceofnewstuff.

5. (E)DetailStep2:IdentifytheQuestionType

Page 24: LSAT PrepTest 81 Unlocked · Similarly, PT 81 was the first test since June '07 (PT June '07) to require at least 94 questions correct to get a 172 (99th percentile score). Number

"Accordingtothepassage"indicatesthatthecorrectanswerwillbedirectlystated,makingthisaDetailquestion.

Step3:ResearchtheRelevantText

ThequestionaskswhatMarsalisencouragedyoungjazzmusicianstodo.Thisisnotamajorpartoftheglobaltheme,sodoaquickscanforContentClues,e.g.,encouragedor"youngmusicians."Thosewordsshoulddirectyourattentiontolines37–39.

Step4:MakeaPrediction

Inlines37–39,itisdirectlystatedthatMarsalisencouragedyoungmusiciansto"payattentionto[jazz]music'straditions."Thecorrectanswershouldsayexactlythat,ifabitparaphrased.

Step5:EvaluatetheAnswerChoices

(E)isaperfectmatch.

(A)isa180,atworst.EventhoughMarsalisencouragedpayingattentiontotradition,theauthorclaimsMarsalishimselfreinventedtraditionalelementsforinnovativepurposes(lines41–44),soitwouldseemunlikelythatMarsaliswouldencouragemusicianstorestrainthatkindofimpulse.

(B)isOutofScope.WhileMarsalishimselfcomposedmusic,thereisnomentionofhimencouragingotherstodoso.

(C)isOutofScope.Thereisnomentionanywhereofplayingwitholdermusicians.

(D)isaDistortion.InthesamesentencethatindicatesMarsalis'sencouragementtoyoungmusicians,itissaidhealsoseekstoelevatethe

Page 25: LSAT PrepTest 81 Unlocked · Similarly, PT 81 was the first test since June '07 (PT June '07) to require at least 94 questions correct to get a 172 (99th percentile score). Number

publicperception,notignoreit.

6. (A)InferenceStep2:IdentifytheQuestionType

Thequestionasksforsomethingwithwhichtheauthoris"mostlikelytoagree,"makingthisanInferencequestion.

Step3:ResearchtheRelevantText

TherearenoContentCluesorlinereferences,sotheentiretextisrelevant.

Step4:MakeaPrediction

Withnoreferencepointtostart,therearetoomanypossibleinferencestomakeasolidprediction.Sticktothemajorthemesandgothroughtheanswersoneatatime.EliminateanythingthatgoesagainstthemainthemeanduseContentCluesinthechoicestotestwhat'snecessary.

Step5:EvaluatetheAnswerChoices

(A)issupported,evenifweakly.Thesupportforthisclaimcomesfromlines51–53.There,theauthorraisedcontrastingopinionsregardingMarsalis'straditionalviews.Tocritics,Marsalis'sclassicismwasidolatry,i.e.,seeingpastmusiciansasidolstobeworshipedandemulated.However,atleastMarsaliswascreatingnewmusic.Forrecordcompanies,classicismwasjust"playtheoldstuffagain."Theyrejectednewmusicentirely,makingthemalittlemorerigid.

(B)isaDistortion.Marsalisencouragednewmusicianstorespecttraditionalviews,butit'sneversuggestedthatMarsalisdirectlypromotedthosenewmusicianspersonally.

Page 26: LSAT PrepTest 81 Unlocked · Similarly, PT 81 was the first test since June '07 (PT June '07) to require at least 94 questions correct to get a 172 (99th percentile score). Number

(C)isnotsupported.Theauthordoesmentionthatbothcriticsandfellowmusiciansweredispleased(lines7–9),butneversuggestsonegroupwasmorevocal.Infact,theauthoronlypresentstheviewsofcritics(lines27–36).

(D)isOutofScope.Theviewsofyoungermusiciansareneveraddressedinthepassage.

(E)isaDistortion.ThereleaseoffifteenCDsismentionedinline12,butthere'snoindicationthatthishadanyimpactoncriticalperception.

7. (E)DetailStep2:IdentifytheQuestionType

Thecorrectanswerwillbeaquestionthatisdirectlyansweredbyadetailinthepassage.

Step3:ResearchtheRelevantText

WithnoContentCluesorlinereferences,theentiretextisrelevant.

Step4:MakeaPrediction

Withnoresearchclues,apredictionisnotpossible.Instead,gothroughthechoicesoneatatime,onlydoingresearchwhennecessarytoensurethequestioninthecorrectchoiceisanswered.

Step5:EvaluatetheAnswerChoices

(E)isansweredinlines53–58.It'smoney,ofcourse.Whatelsewouldencouragerecordcompaniestodothis?

(A)isnotanswered.Marsalisdidn'treleaseanymusicinthattime,butit'snotknownwhetherornothewascomposinganymusic.

Page 27: LSAT PrepTest 81 Unlocked · Similarly, PT 81 was the first test since June '07 (PT June '07) to require at least 94 questions correct to get a 172 (99th percentile score). Number

PASSAGE2:INFERENTIALVS.NONINFERENTIALTHOUGHTSStep1:ReadthePassageStrategically

SampleRoadmap

line# Keyword/phrase ¶Marginnotes

2 but Commonbelief:knowownthoughts;inferothers’

4 while

5 But

6 challenged Studieswithchildrendispute

9 whilenonetheless

12 but

13 argue Psych:peopleinferownthoughts

15 everybit

16 Accordingto

(B)isOutofScope.NodescriptionofMarsalis'sfanbaseisgiven.

(C)isnotansweredinthepassage.ThismaybetemptingforanyonefamiliarwithMarsalis,whodidindeedreleasesuchCDs.However,thecontentofMarsalis'sCDsarenotdescribedinthispassage,andthisisthusnotcorrect.

(D)isnotanswered.ItisonlymentionedthatMarsalisdidnothaveacontract.Itisneveractuallysaidwhy.

Page 28: LSAT PrepTest 81 Unlocked · Similarly, PT 81 was the first test since June '07 (PT June '07) to require at least 94 questions correct to get a 172 (99th percentile score). Number

19 Itfollows

20 wrong

23 sotenaciously;illusorybelief Knowingthoughtsisanillusion

25 suggest;analogousto

28 notonly;but similartoexpertise

31 whereas

32 Forinstance

35 so

36 expert Inferquickly;don'tnotice

37 fail

38 failure;leadsnaturallyto

39 supposition

42 claiming

43 perilouslyclose Auth:potentialdangerousclaim

46 But;infact But,psych.avoidproblem

47 donot

48 suggest

51 e.g.

53 explainswhy Inferencesmadeinternally

57 Thus;crucial

line# Keyword/phrase ¶Marginnotes

text text text

Page 29: LSAT PrepTest 81 Unlocked · Similarly, PT 81 was the first test since June '07 (PT June '07) to require at least 94 questions correct to get a 172 (99th percentile score). Number

Discussion

Thepassageopensupwithwhatcommonsensesuggests:Presumably,wejustknowourownthoughtswithcompleteaccuracy,andwemerelyguessotherpeople'sthoughts.However,ascouldbeexpectedinanLSATpassage,thisassumptionisrejected.Theauthorpresentsastudyinwhichchildrencanaccuratelydescribecertainevents,buthavetroubledescribingtheirownthoughtsaboutthoseevents.Fromthis,"psychologistsargue"(line13)thatourthoughtsarenotdirectlyobservable;we'remerelyinferringourownthoughts,too,andwecanbewrongaboutthem.

Thesecondparagraphpresentsanexplanationwhyweinsistthatweknowourownthoughtsinfallibly.Psychologistsexplainitthroughananalogyinvolvingexpertise.Whenwegainexpertiseinafield,itappearstochangeourknowledgeandperception.Wethoughtweweremakinginferencesbeforeandnowjustseethetruth.However,psychologistssuggestwe'restillmakinginferences;we'rejustgettingsofastatmakingthemthatwedon'tevenrealizeit.It'simportanttonotethepersistentuseofphrasessuchas"itappears"and"thesupposition."It'sconstantlysuggestedthatpeoplejustbelievethey'reobservingthingsdirectlyandinfallibly,butthey'renot—they'rejustmakinginferences.That'samajorthemeofthispassage.

Inthethirdparagraph,theauthorbringsupapotentiallydangerousimplication:Psychologistsarealmostsayingourinferencesaresolelybasedonourexternalbehaviors.But,theauthorqualifiesthatthey'renotsayingthat.Instead,psychologistssayourinferencesarebasedon

Page 30: LSAT PrepTest 81 Unlocked · Similarly, PT 81 was the first test since June '07 (PT June '07) to require at least 94 questions correct to get a 172 (99th percentile score). Number

internalactivityinthebrain.Thisactivityiswhatmakesourinferencessoreliableandseeminglyinfallible.

There'salotofpsychologicaljargonhere,butstayfocusedontheoverridingtheme.TheTopicisourthoughts,andtheScopeiswhetherwedirectlyobserveourthoughtsinfalliblyornot.ThePurposeistopresenttheviewsofpsychologists(notehowalmostallopinionsinthepassageareattributedtothem).TheMainIdeaisthat,contrarytowhatpeopleassume,psychologistsarguethatwedonotknowourownthoughtsdirectly;wesimplyinferthem,andthoseinferencesarenotbasedsolelyonobservationsofourexternalbehavior.

8. (B)GlobalStep2:IdentifytheQuestionType

Thequestionasksforthe"mainpointofthepassage,"makingthisaGlobalquestion.

Step3:ResearchtheRelevantText

Noneedtogobackintothepassage.JustconsidertheMainIdeaaspredictedafterreadingthepassage.

Step4:MakeaPrediction

Themainideaisthat,accordingtopsychologists,wedonotdirectlyobserveourthoughtsinfallibly,asiscommonlyassumed.Instead,we'reactuallyjustmakinginferencesaboutourownthoughts.

Step5:EvaluatetheAnswerChoices

(B)isaperfectmatch.

(A)isaDistortionanda180.Thismisinterpretstheinformationaboutexpertiseinthesecondparagraph.Expertisemakesitappearthatweare

Page 31: LSAT PrepTest 81 Unlocked · Similarly, PT 81 was the first test since June '07 (PT June '07) to require at least 94 questions correct to get a 172 (99th percentile score). Number

observingourthoughtsdirectlyandinfallibly.However,thepsychologistsarguethatthisisstilljustanillusion.Nobodyissaidtodirectlyobservetheirownthoughts—notevenexperts.

(C)isalsoaDistortionanda180.First,thepsychologists'claimsarenotsaidtobe"inresponse"tothecommonbelief.Moreover,thiscontradictslines52–54,whichstatethatwecanmakequickandreliableinferences.

(D)isaDistortion.Theexperimentwithchildrenisjustastartingpointfortheargumentmadeinthepassage,notaprimaryfocus.Andthepsychologistsneverblameanythingonthelackofexpertise.

(E)isa180.Thisistheclaimthattheauthorsayspsychologistsare"perilouslyclose"tomaking(lines43–46).However,theauthorimmediatelyrejectsthatandsuggestspsychologistsarenotmakingthatclaim.

9. (C)LogicReasoning(Weaken)Step2:IdentifytheQuestionType

Thequestionasksforsomethingthatwould"callintoquestion"anargument,makingthisaWeakenquestionlikethosefoundinLogicalReasoning.

Step3:ResearchtheRelevantText

Thequestionprovidesthelinereferencesfortheprimaryargument,butithelpstoconsiderthefulldetailsoftheexperiment,asdescribedinlines6–10.

Step4:MakeaPrediction

Page 32: LSAT PrepTest 81 Unlocked · Similarly, PT 81 was the first test since June '07 (PT June '07) to require at least 94 questions correct to get a 172 (99th percentile score). Number

Thepsychologistsconcludethatpeopleinfertheirownthoughtsbasedonevidenceofastudyinvolvingchildren.Inthestudy,thechildrenhavetroubledescribingtheirthoughtsaboutcertainevents.Thepsychologistsassumethistroubleisduetothechildreninferringtheirthoughts,andnothingelse.Toweakentheargument,thecorrectanswershouldprovideanalternateexplanationforwhychildrenhavetroubledescribingtheirthoughts.

Step5:EvaluatetheAnswerChoices

(C)iscorrect.Ifthechildrenarestumblingduetolimitedlanguageskills,thentheirinabilitytodescribetheirthoughtsmayhavenothingtodowithmakinginferences.Theymaybeseeingtheirthoughtsdirectly,butjusthavingahardtimeexpressingthemselves.

(A)isa180.Thissuggestskidsarejustascapableasadultsatidentifyingtheirthoughts.Thatwouldmeankidsarejustasvalidasourceoftestingasadultswouldbe,makingtheexperimentandthepsychologists'deductionsseemappropriate.

(B)isa180.Thissuggestschildrenandadultscanbeequallyaccurate(orequallyinaccurate),whichmeanschildrencouldbejustasvalidasamplegroupasanyone.

(D)isOutofScope.Thechildrendon'thavetoknowthedifference.Whatmattersiswhatthepsychologistsobserveduringtheexperiment.

(E)isalsoOutofScope.Evenifthestudywasintendedforotherreasons,it'sstillacceptableforpsychologiststodrawconclusionsaboutotherconceptsfromthatstudy.

Page 33: LSAT PrepTest 81 Unlocked · Similarly, PT 81 was the first test since June '07 (PT June '07) to require at least 94 questions correct to get a 172 (99th percentile score). Number

10. (E)InferenceStep2:IdentifytheQuestionType

Thequestionaskswhattheauthoris"mostlikelytobelieve"regardingaclaimfromthepassage.ThatmakesthisanInferencequestion.

Step3:ResearchtheRelevantText

Theclaiminquestionisatthebeginningofthethirdparagraph,butbesuretoconsiderthecontextoftheparagraphasawhole.

Step4:MakeaPrediction

Theclaiminquestionisonetheauthorsayspsychologistsare"perilouslyclose"tomaking.But(line46),theauthorimmediatelystatesthatpsychologistsarenotactuallymakingthatclaim.Thecorrectanswerwillindicatetheauthor'sbeliefthattheclaiminquestionis,ultimately,notsupported.

Step5:EvaluatetheAnswerChoices

(E)iscorrect.Theauthordoesnotbelievethereissupportforthatclaim—evenifpsychologistsareperilouslyclosetomakingit.

(A)isOutofScope.Theauthorneversuggeststhatit'simpossibletostudythinkingprocesses.

(B)isaDistortion.Theclaiminquestionisonethatpsychologistscomeclosetomaking,butdon'tactuallymake.Iftheydon'tactuallymakethatclaim,thentheycan'tpossiblemisunderstandit.

(C)isaDistortion.Theprevailingviewthatexperimentsundermineisthecommonbeliefpresentedinlines1–5.Theclaiminquestionisnotaprevailingview.Infact,theauthorsayspsychologistsdon'tevenreallybelieveit.

Page 34: LSAT PrepTest 81 Unlocked · Similarly, PT 81 was the first test since June '07 (PT June '07) to require at least 94 questions correct to get a 172 (99th percentile score). Number

(D)isa180.Theauthorclaimsthatpsychologistsdon'tactuallybelievethisview.Andbysayingthatpsychologistscome"perilouslyclose"toclaimingit,theauthorsuggestsit'sadangerousideaandnotlikelytobe"basicallysound."

11. (A)LogicReasoning(ParallelReasoning)Step2:IdentifytheQuestionType

Thequestionasksforasituation"mostcloselyanalogous"toonepresentedinthepassage.ThatmakesthisaLogicReasoningquestion,specificallyonethatmimicsParallelReasoning.

Step3:ResearchtheRelevantText

Theexplanationforpeople'sfailuretonoticethey'remakinginferencesisdescribedthroughoutthesecondparagraph,primarilyinlines27–41.

Step4:MakeaPrediction

Thefailureisdirectlydescribedinlines35–38:Wemakeinferencessofastthatwefailtonoticewe'remakingthem.Basedonthesurroundinglines,thisisbecauseweappeartograsprelationsthroughexpertiseandjustassumewe'reseeingthingsdirectlyinstead.So,thecorrectanswerwilldescribesomeonewhohasdevelopedexpertiseandassumes(incorrectly)thateverythingisnowbeingobserveddirectly.

Step5:EvaluatetheAnswerChoices

(A)providesagoodexample.Inthiscase,theanthropologisthasbecomesofamiliarwithhisculturethathetakesitforgrantedandjustassumesheseesthetruth—andhe'swrong!

(B)doesnotmatch.Thisplacesalimitonstudyingsomethingduetoarequirement,whichhasnothingtodowiththeillusionofdirect

Page 35: LSAT PrepTest 81 Unlocked · Similarly, PT 81 was the first test since June '07 (PT June '07) to require at least 94 questions correct to get a 172 (99th percentile score). Number

observation.

(C)doesnotmatch.Thefailurepeoplehaveinthepassagehasnothingtodowithaninabilitytogofromabstractideastoconcreteexperiences.

(D)doesnotmatch.Conflictofinterestdoesnotmatchtheideaofmakingbadassumptionsbasedonexperience.

(E)doesnotmatch.Wefailtonoticeourinferencesbecauseweassumewe'reseeingthingsdirectly,notbecausewe're"toobusy"doingsomethingelseandhavetopassalongthework.

12. (D)DetailStep2:IdentifytheQuestionType

"Accordingtothepassage"indicatesthatthecorrectanswerwillbedirectlystated,makingthisaDetailquestion.

Step3:ResearchtheRelevantText

Thequestionasksabouttheresultofgaininggreaterexpertise,whichisdescribedinlines27–32.

Step4:MakeaPrediction

Accordingtothepassage,greaterexpertiseappearstochange"ourknowledgeof[an]area"andour"perceptionofentitiesinthatarea,"anditappearsweareableto"grasptheseentitiesandtheirrelationsdirectly."

Step5:EvaluatetheAnswerChoices

(D)matchesthedescribedchangeinourperceptionandthewayweunderstand(i.e.,grasp)therelationsinaparticulararea.

Page 36: LSAT PrepTest 81 Unlocked · Similarly, PT 81 was the first test since June '07 (PT June '07) to require at least 94 questions correct to get a 172 (99th percentile score). Number

(A)isOutofScope.Nothingismentionedaboutexpressingjudgmentaboutissues.

(B)isOutofScope.Nothingismentionedabouttakingadetail-orientedapproach.

(C)isaDistortion.Wemayfailtonoticewe'remakinginferences,butthatdoesn'tmeanweignoreerrors.Youcan'tignoresomethingifyoudon'tevenrealizeit'sthere.

(E)isaFaultyUseofDetail.Thisreferstothesensationsandemotionsbroughtupinline51–52.However,itisnotsuggestedthatwereduceourrelianceonthesesensationsandemotions.Onthecontrary,theymakeitpossibletomakeinferencesinthefirstplace.

13. (C)DetailStep2:IdentifytheQuestionType

Thequestionasksforsomethingdirectlymentionedaccordingtoviews"citedinthepassage,"makingthisaDetailquestion.

Step3:ResearchtheRelevantText

The"illusionofdirectknowledge"refersbacktolines38–41("thesuppositionthat...weareperceiving[things]directly").

Step4:MakeaPrediction

Thelastsentenceofthesecondparagraphstatesthat"[t]hisfailureleadsnaturally"totheillusioninquestion.Thatphraserefersbacktotheprevioussentence(lines35–38),wherepsychologistsclaimthatwemakeinferencessofastthatwefailtonoticewe'remakingthen.So,theillusion

Page 37: LSAT PrepTest 81 Unlocked · Similarly, PT 81 was the first test since June '07 (PT June '07) to require at least 94 questions correct to get a 172 (99th percentile score). Number

ofdirectknowledgecomesfromthatfailuretonoticewe'remakinginferences.

Step5:EvaluatetheAnswerChoices

(C)matchesthestatedsourceaccordingtothepsychologists.

(A)isOutofScope.Thereisnodiscussionofgettingfeedbackontheaccuracyofourinferences.

(B)isa180.Itisfrequentlysuggestedthatwedonothaveunmediated(i.e.,direct)knowledgeofourthoughts.

(D)isa180.Itisoftensuggestedthatwebelieveourinferencesareinfallible(i.e.,absolutelyaccurate),butthat'snotactuallythecase.

(E)isaDistortion.Wemakeincrediblyfastinferenceswhichmaynotbeinfallible,butthere'snosuggestionthatthoseinferencesareinanywaycloudedoruncertain.We'recertainwe'reright,evenifthat'snotactuallythecase.

14. (D)InferenceStep2:IdentifytheQuestionType

Thequestionasksforsomethingthatcan"mostreasonablybeinferred,"makingthisanInferencequestion.

Step3:ResearchtheRelevantText

Theuseofchildrenfortheexperimentsisdiscussedinthefirstparagraph.

Step4:MakeaPrediction

Accordingtolines10–13,childrenhavethesamethoughtsasadults,whichmakesthemequallyvalidsubjects.Thedifference,though,isthat

Page 38: LSAT PrepTest 81 Unlocked · Similarly, PT 81 was the first test since June '07 (PT June '07) to require at least 94 questions correct to get a 172 (99th percentile score). Number

PASSAGE3:DOWSINGStep1:ReadthePassageStrategically

SampleRoadmap

line# Keyword/phrase ¶Marginnotes

2 Dowsingdefined

childrenare"muchlesscapableofidentifyingthesethoughts."Thatmusthavebeenthereasonthestudyusedchildreninsteadofadults,suggestingthere'sabenefittousingsubjectsthathavegreatertroublerecognizingtheirthoughts.

Step5:EvaluatetheAnswerChoices

(D)matchesthepredictionthattheadvantagecomesfromthelikelihoodofmakingmistakes(i.e.,beinglesscapableofidentifyingtheirownthoughts).

(A)isOutofScope.Thestudyisnotaboutcreativity,it'saboutwhetherthoughtsarerecognizeddirectlyorbyinference.

(B)isa180.It'sthechildrenthataremorelikelytobeinaccurate,nottheadults.

(C)isa180.Itisfrequentlysuggestedinthepassagethatnobodyisactuallyinfallible.Everyonemakesinferencesabouttheirownthoughts.

(E)isaDistortion.Thestudyisnotabouttheabilitytoinferthethoughtsofothers.It'saboutinferringone'sownthoughts.

Page 39: LSAT PrepTest 81 Unlocked · Similarly, PT 81 was the first test since June '07 (PT June '07) to require at least 94 questions correct to get a 172 (99th percentile score). Number

4 Forexample Ex.findingwater

7 claiming

16 skeptical Skeptics:

17 crudeness

18 assert 1)Crudetools=actuallyuse

subsconsious

23 Further

24 skepticssay;whileafew

25 considerable

26 success 2)Inconsistentresults

27 generallyisnotablyinconsistent;Finally;

skeptics

28 note

30 unlikely 3)Hand-pickedlocations

32 Proponents

33 contend Proponents:

34 shouldbe 1)Manytechniques

35 alsonote 2)Studiesskewed

39 Proponents

40 claim 3)Senseelectromag.

43 alsoclaim 4)Moresuccessfulthanothers

47 corroborated Studysupportsproponents

57 significantlymoreaccurate

58 even

Page 40: LSAT PrepTest 81 Unlocked · Similarly, PT 81 was the first test since June '07 (PT June '07) to require at least 94 questions correct to get a 172 (99th percentile score). Number

line# Keyword/phrase ¶Marginnotes

text text text

Discussion

Thefirstparagraphisveryintroductory,startingoffwithadefinitionoftheTopic:dowsing.Thisinvolvesfindingthingsundergroundusingbasictools.Thedefinitionisfollowedbyadetailedexamplethatinvolvesfindingwaterwithatreebranch.

Thesecondparagraphofferssomeopinions,whichhelpidentifytheScopeofthepassage:Howeffectiveisdowsing?Accordingtotheskepticsinthesecondparagraph,notverymuchatall.Theircriticismsboildownto:1)Themethodsarecrude,andthetoolsdonothing—it'sallinthedowser'ssubconscious;2)studiesshowinconsistentresults;and3)dowsersjusthappentogowheresuccessisstatisticallymorelikelyinthefirstplace.

Thethirdparagraphoffersadefensefromproponentsofdowsing.Inshort:1)Therearevariousdistincttechniques,soyoucan'tjustlumpthemalltogetherandjudge;2)studiestendtouseinappropriatesubjectswhomerelyclaimtobeexpertsbuthavenocertification;3)successfuldowsersaresensitivetoundergroundconditions;4)dowsersaremoresuccessfulthanscientistswhousefancyschmancytools.

Page 41: LSAT PrepTest 81 Unlocked · Similarly, PT 81 was the first test since June '07 (PT June '07) to require at least 94 questions correct to get a 172 (99th percentile score). Number

Thelastparagraphpresentsastudythatsupportsthelasttwoclaimsoftheproponents.Inthestudy,dowserscompetedagainstgeologistsandhydrologiststofindwaterinaparticulararea,andthedowsersweremoresuccessful—evenfindinganareawithnowaterwhenaskedtodoso.

ThePurposeofthisDebatepassageismostlytopresenttheviewsofbothpartiesabouttheefficacyofdowsing.Theauthordoesoffersupportfortheproponentsinthelastparagraph,whichmaysuggestsometacitendorsementofdowsing.ButtheoverallMainIdeaisprettyneutral:Somepeopleareskeptical,butthereissupportthatdowserscanfindthingsundergroundeffectively.

15. (C)LogicFunctionStep2:IdentifytheQuestionType

Thequestionasksforthe"primarypurposeofthesecondparagraph."Inotherwords,it'saskingforthefunctionoftheparagraphwithinthecontextaroundit.

Step3:ResearchtheRelevantText

There'snoneedtorereadtheactualtext.JustusetheMarginNotestoseehowthesecondparagraphfitswithintheoverallstructure.

Step4:MakeaPrediction

Thesecondparagraphconsistsoftheskeptics'criticismsofdowsing,theconceptdescribedinthefirstparagraph.Thosecriticismsarecounteredbyproponentsinthethirdparagraph.

Step5:EvaluatetheAnswerChoices

(C)iscorrect.Thesecondparagraphcontainstheargumentsagainstdowsing,andthethirdparagraphcountersthosecomplaints.

Page 42: LSAT PrepTest 81 Unlocked · Similarly, PT 81 was the first test since June '07 (PT June '07) to require at least 94 questions correct to get a 172 (99th percentile score). Number

(A)isnotaccurate.Thesecondparagraphisentirelyaboutopinions,notjustsupplementarydetails.

(B)isnotaccurate.Thereisoneconsistentpointofviewinthesecondparagraph,andthere'snosynthesisofpointsofviewinthelastparagraph.

(D)isaDistortion.Theparagraphoffersopinionsaboutthedetailsinthefirstparagraph.However,theopinionspresentedinthesecondparagraphareverybroadandhardly"explore[d]indetail."Andtherearenoramificationstospeakof.It'sjustadiscussionofhowonegroupofpeoplefindssomethingtobeineffective.

(E)isaDistortionandOutofScope.Thesecondparagraphonlydiscussesonesideofthedispute(theskeptics),andthethirdparagraphdiscussestheotherside.Noresolutionistobefound.

16. (A)DetailStep2:IdentifytheQuestionType

"Accordingtothepassage"indicatesthatthecorrectanswerwillbeadetailthatisdirectlystatedinthepassage.

Step3:ResearchtheRelevantText

Theskeptics'pointofviewisoutlinedthroughoutthesecondparagraph.

Step4:MakeaPrediction

Theskepticshavealotofcomplaints.However,thecontrastKeywordwhileinline24indicatesabriefconcession:"afewdowsershavedemonstratedconsiderableandconsistentsuccess."

Step5:EvaluatetheAnswerChoices

Page 43: LSAT PrepTest 81 Unlocked · Similarly, PT 81 was the first test since June '07 (PT June '07) to require at least 94 questions correct to get a 172 (99th percentile score). Number

(A)matchestheskeptics'acknowledgmentwordforword.

(B)isaDistortion.Somescientistsarementionedinthepassage(e.g.,geologistsandhydrologists),butanycriticismtowarddowsinginthepassageismadesolelybytheskeptics,notanyscientists.

(C)isaFaultyUseofDetail.Thisisthecomplaintproponentsmakeinthethirdparagraph(lines34–39),butthereisnoacknowledgmentofthisbytheskeptics.

(D)isalsoaFaultyUseofDetail.Skepticsdoclaimthatdowsersmaybeworkingsubconsciously(lines18–22),butthespecificconceptofbeingsensitivetoEarth'selectromagneticfieldisraisedbytheproponents(lines39–42),nottheskeptics.

(E)isalsoaFaultyUseofDetail.Separateevaluationisencouragedbytheproponents(lines32–34),nottheskeptics.

17. (D)LogicReasoning(ParallelReasoning)Step2:IdentifytheQuestionType

Thecorrectanswerwillhavereasoning"mostanalogousto"thatofanargumentinthepassage.ThatmakesthisaLogicReasoningquestionalongthelinesofParallelReasoning.

Step3:ResearchtheRelevantText

Theskeptics'argumentsarepresentedinthesecondparagraph.

Step4:MakeaPrediction

Unfortunately,thequestionasksforsomethingparalleltoanargumentmadebytheskeptics,andtheymakeafew.It'simpossibletoknowwhich

Page 44: LSAT PrepTest 81 Unlocked · Similarly, PT 81 was the first test since June '07 (PT June '07) to require at least 94 questions correct to get a 172 (99th percentile score). Number

onetochoose.Haveaquickideaofthethreearguments:1)Dowsers'toolsarecrude;they'rejustusingtheirsubconscious;2)studiesshowinconsistency;3)dowsersjustgowherethey'remorelikelytosucceedinthefirstplace.Thecorrectanswerwillshowsomeoneconsistentwithoneofthesearguments.

Step5:EvaluatetheAnswerChoices

(D)isperfectlyparalleltothelastargument.Asdowsersjusthappentogowherewateriseverywhereandsay"look,Ifoundwater!,"thepeopleinthisanswerjusthappentogowherefishareeverywhereandsay"look,Ifoundfish!"

(A)doesnotmatch.Theskepticsneverclaimtherearetoolsthataremoreaccuratethanwhatdowserssuggest.

(B)doesnotmatch.Theskepticsneveraccusedowsersofhavinglittleevidencetosupporttheirclaims.

(C)doesnotmatch.Theskepticsneversuggestthatdowsersclaimtheirabilitiesareinnate,nordoskepticssuggestthatanysuccessofdowsingisduetointensepractice.

(E)doesnotmatch.Thismayseemsomewhatsimilartothefirstargument,inwhichdowsersclaimtheirtoolsworkbutit'sallinthesubconscious.However,theskepticsarguethatdowsers'subconsciousdeterminationisbasedon"cluesderivedfromsurfaceconditions,"notjustthoughtsofthingsthatdidn'tactuallyhappen.

18. (B)InferenceStep2:IdentifytheQuestionType

Page 45: LSAT PrepTest 81 Unlocked · Similarly, PT 81 was the first test since June '07 (PT June '07) to require at least 94 questions correct to get a 172 (99th percentile score). Number

Thequestionasksforsomethingwithwhichtheauthoris"mostlikelytoagree,"makingthisanInferencequestion.

Step3:ResearchtheRelevantText

Thequestionasksaboutthestudyinthefinalparagraph.

Step4:MakeaPrediction

Atthebeginningoftheparagraph,theauthorclaimsthatthestudycorroboratesthe"lasttwoclaims"oftheproponents.Goingbacktothepreviousparagraph,thoseclaimswerethat1)dowserscandetectchangesintheelectromagneticfield;and2)dowserscanbemoresuccessfulthanotherscientists.Thefactsofthestudyareconsistentwithbothpoints.

Step5:EvaluatetheAnswerChoices

(B)iscorrect.Thestudydoesn'tprovethatdowserscandetectsuchchanges,butthepossibilityiscertainlythere.

(A)isExtreme.Whilethedowsersmayhavehadmoresuccess,thatdoesn'tmeantheotherscientistswouldbe"oflittleservicetoanygroundwater-locatingeffort."

(C)isExtreme.Thestudymaycorroboratesomeideas,butthat'shardlythesameasprovingdowsingisthe"mostdependable."

(D)isExtremeandaDistortion.Thestudydoesnothingtoshowwhatmakesdowsersmostsuccessful.Further,itmakesnosensetosuggestthatdowsersusedanytoolsotherthantheirown.

Page 46: LSAT PrepTest 81 Unlocked · Similarly, PT 81 was the first test since June '07 (PT June '07) to require at least 94 questions correct to get a 172 (99th percentile score). Number

(E)isa180.Whilefocusingononetypeofterrainmightindicatethestudyisn'taconclusiverebuttal,itdefinitelydoeshelptorefutesomeoftheskeptics'arguments.

19. (D)DetailStep2:IdentifytheQuestionType

ThecorrectanswerwillbeaquestionthatisanswereddirectlybyaDetailinthepassage.

Step3:ResearchtheRelevantText

WithnoResearchClues,theentirepassageisrelevant.

Step4:MakeaPrediction

Apredictioncannotbemadehere.Instead,gothroughtheanswersoneatatimeandresearchwhennecessarytomakesurethereisadirectlystatedanswertothequestionprovided.

Step5:EvaluatetheAnswerChoices

(D)isansweredinthefirstsentence.Dowsingisusedtodetectresources(e.g.,water)orobjects—whichsuggestsphysicalitemsotherthanwater.

(A)isnotanswered.Notimelineisgiveninthepassage.

(B)isnotanswered.Theeffectofrainisnotbroughtup.

(C)isnotanswered.Forkedsticksarebroughtupasonemethodforfindingwater,butthere'snomentionofwhetherthisisthemostcommonorhowitcomparesstatisticallytoothermethods.

(E)isnotanswered.Skepticsonlybroadlyrefertousingsurfaceclues(lines21–22),butnevermentionanyspecificclues.

Page 47: LSAT PrepTest 81 Unlocked · Similarly, PT 81 was the first test since June '07 (PT June '07) to require at least 94 questions correct to get a 172 (99th percentile score). Number

20. (E)InferenceStep2:IdentifytheQuestionType

Thecorrectanswerissomethingforwhichthereis"supportforinferring,"makingthisanInferencequestion.

Step3:ResearchtheRelevantText

TherearenoResearchClues,sotheentiretextisrelevant.

Step4:MakeaPrediction

Withnoclues,there'snochoicebuttogothroughtheanswers,eliminatethosethatareclearlywrong,andtesttheremaininganswersasnecessary.

Step5:EvaluatetheAnswerChoices

(E)issupported.Accordingtoproponentsofdowsing,successfuldowsers"arenotwellrepresentedinthetypicalstudy"(lines38–39).However,thestudyinthelastparagraphwasextensiveandusedteamsof"themostsuccessfuldowsers."Combined,thatsuggeststhelaststudyisnotyourtypicalstudyinvolvingapoorrepresentativesample.

(A)isaDistortion.Thestudyinthelastparagraphwasconductedaroundnarrow,tiltedfracturezones.Andwhiledowsersdidfindadryzoneonrequest,thatisnottosaytheentireregionwasarid.Nordoesthatsuggestthatsuchfracturezonesaremorecommoninaridregionsthaninotherregions.Thecomparisonisunsubstantiated.

(B)isExtremeandOutofScope.Thepassageonlydiscussesstudiesrelatedtofindinggroundwater,nototherresources.Further,there'snothingtosuggestthatnoreliablestudieshavebeenperformed.

Page 48: LSAT PrepTest 81 Unlocked · Similarly, PT 81 was the first test since June '07 (PT June '07) to require at least 94 questions correct to get a 172 (99th percentile score). Number

PASSAGE4:THEUSEOFINDEPENDENTRESEARCHBYJUDGESStep1:ReadthePassageStrategically

SampleRoadmap

line# Keyword/phrase ¶Marginnotes

PassageA

2 ? Whyopposeind.research?

3 One;objections;distorts 1)Distortssystem

5 undermining

6 Anotherfear 2)Judgesmayresearchpoorly

10 While;somemerit;do Auth:ind.researchcanbegood

11 notjustify

12 First

14 ill-suited 1)Goodforspecializedknowledge

19 Because

20 considerableinfluence

(C)isnotsupported.There'snomentioninthefinalstudywhattoolswereusedorwhethertheywouldbeanydifferentfromtoolsusedindifferentzones.

(D)isaDistortion.Itismerelysaidthatdowserswereabletolocateadryzone.Thatdoesn'tmeanthatotherscientistscouldn't.Perhapstheyalsodid,orperhapstheyjustweren'tasked.

Page 49: LSAT PrepTest 81 Unlocked · Similarly, PT 81 was the first test since June '07 (PT June '07) to require at least 94 questions correct to get a 172 (99th percentile score). Number

21 erroneous;detract

22 couldhelp helpsavoiderrors

24 Second 2)Trialstructurepreventsbadresults

25 reducing

26 outlandish

27 ratherthan Supplements;doesn'treplace

28 so

PassageB

30 Regardless

31 shouldresist Auth:Appellateshouldnotuseind.research

33 Asageneralrule

34 Thus;lack Appellatecourtslacklivetestimonyand

cross-exam

35 critical

36 :

39 And Benefitofcross-exam

45 However

50 Thus Nolivecomment

53 inparticular Usurpstrialcourt'sfunction

54 comeundercriticism;potential

unreliability

56 ignores Ignoresfunctionofappellatecourt

57 questionable

Page 50: LSAT PrepTest 81 Unlocked · Similarly, PT 81 was the first test since June '07 (PT June '07) to require at least 94 questions correct to get a 172 (99th percentile score). Number

59 criticism

60 fullforce

61 regardless

line# Keyword/phrase ¶Marginnotes

text text text

Discussion

TheauthorofpassageAstartswithaquestion:Whyaresometrialjudgesagainstconductingindependentresearch?AswithmostquestionsinanLSATpassage,thisoneisansweredandservesasafocalpointforthewholepassage.Therestoftheparagraphdescribessomeobjections:1)Independentresearchcanskewresultsandundermineotherimportantevidence;and2)judgesmaynothavethebestresearchtechniques.

Theauthorrecognizestheconcerns,butofferstworeasonsoverthenexttwoparagraphswhyindependentresearchcanbegood.First,incasesrequiringspecializedknowledge,theevidenceraisedbybothpartiescanleadtoconflictsandfutureproblemsthatindependentresearchcanhelpavoid.Second,trialshaveastructurethatreducesthechancesofjudges'researchproducingcrazyresultsandensuressuchresearchisasupplementtootherevidence,notareplacement.

TheTopicofpassageAisindependentresearch,withtheauthorfocusedontheScopeofitsbenefits.Theauthor'sPurposeistosupporttheuseof

Page 51: LSAT PrepTest 81 Unlocked · Similarly, PT 81 was the first test since June '07 (PT June '07) to require at least 94 questions correct to get a 172 (99th percentile score). Number

independentresearch.TheMainIdeaisthattherearecircumstancesinwhichjudgesdoingindependentresearchisacceptable.

TheauthorofpassageBstickstotheTopicofindependentresearch,butshiftstheScopetoitsusespecificallyinappellatecourts.Theauthorimmediatelysuggeststhatappellatecourtsshouldnotconductindependentresearch.ThatsuggeststhePurposeofthispassagewillbetoexplainwhyitshouldn'tbeused.

Thesecondandthirdparagraphofferevidenceagainstusingindependentresearchinappellatecourts.Appellatecourtslackthecriticalcomponentsoflivepresentationandcross-examinationfoundintrialcourts.Thesecondparagraphfocusesonthevalueofcross-examination,whilethethirdparagraphexplainswhylivepresentationisvaluableandhowraisingnewinformationinappellatecourtswouldstealthatfunctionfromatrialcourts.

ThelastparagraphwrapsuptheMainIdeathatindependentresearchisinappropriateinappellatecourtsandgoesagainstthefunctionofanappellatecourtasacourtofreview.

Thepassagesaredefinitelyofdifferentmindsaboutindependentresearch.However,itshouldbenotedthattheauthorofpassageAsticksmainlytoitsbenefitsintrialcourts,whiletheauthorofpassageBismoreconcernedwithitsusespecificallyinappellatecourts.

21. (C)LogicReasoning(Principle)Step2:IdentifytheQuestionType

Thequestiondirectlyasksforaprincipleunderlyingbothpassages,makingitaLogicReasoningquestionofthePrinciplevariety.

Page 52: LSAT PrepTest 81 Unlocked · Similarly, PT 81 was the first test since June '07 (PT June '07) to require at least 94 questions correct to get a 172 (99th percentile score). Number

Step3:ResearchtheRelevantText

Thequestionasksabouttheoverallargumentsinbothpassages,sotheentiretextisrelevant.

Step4:MakeaPrediction

TheauthorofpassageAarguesthatindependentresearchisbeneficialbecauseithelpsavoidconflictandsupplementswhatisprovidedbythestructureofthetrialcourt.TheauthorofpassageBarguesagainstindependentresearchbecauseittakesawaythefunctionoftrialcourtsandgoesagainstthefunctionoftheappellatecourt.Bothauthorsareintentonmakingsurethatindependentresearchhelpssupplementacourt'sstructureandfunction,notgoagainstthat.

Step5:EvaluatetheAnswerChoices

(C)iscorrect.Asthefirstauthorclaims,independentresearch"supplements,ratherthanreplaces"(lines27–28)evidencefromopposingparties.Andthesecondauthorclaimsusingindependentresearchisbadbecauseappellatecourtswould"substituteitsownquestionableresearchresultsforevidencethatshouldhavebeentestedinthetrialcourt"(lines57–59).So,bothauthorswanttoensureindependentresearchdoesnotsupersedetheelementsofatrial.

(A)mightfitwellwiththeauthorofpassageB,buttheauthorofpassageAnevermakesacomparisonbetweentrialcourtsandappellatecourts,sosuchaprinciplewouldbeirrelevant.

(B)isirrelevanttopassageB,whichinvolvesdoingresearchinappellatecourt,whichwouldtakeplaceafteratrial.

Page 53: LSAT PrepTest 81 Unlocked · Similarly, PT 81 was the first test since June '07 (PT June '07) to require at least 94 questions correct to get a 172 (99th percentile score). Number

(D)isOutofScope.WhiletheconceptofquestioningwitnessesisraisedasasidenoteinpassageB(line43–45),it'snotamainpartoftheargumentandhasnobearingontheargumentinpassageA.

(E)isa180forpassageB.TheauthorofpassageBweighsinonthereliabilityofsomeoutsideresourcesinlines53–54,andcitesthelackoftheirreliabilityasareasonagainstappellatejudgesusingoutsideresearchatall.Furthermore,passageAmakesnomentionofwhat"[b]othtrialandappellatejudges"shoulddoandalsodoesnotdiscusswhereoutsideresearchshouldhavetocomefrom.

22. (A)InferenceStep2:IdentifytheQuestionType

Thequestionasksforsomethingthat"canbeinferred,"makingthisanInferencequestion.

Step3:ResearchtheRelevantText

Thequestionasksaboutbothpassages,andwhatshouldbedoneifjudgesdoconductindependentresearch.That'sraisedinpassageAinthethirdparagraph(lines24–29),andinpassageBinthethirdparagraph(lines47–54).

Step4:MakeaPrediction

AccordingtotheauthorofpassageA,independentresearchisguidedbythestructureofatrialandshouldbesupplementary,notasubstitution.AndtheauthorofpassageBsaysthattheappellatecourtsbringingupnewinformationwould"usurpthetrialcourt'sfact-findingfunction,"whichsuggeststhatpassageBagreesthatindependentresearchshouldberestrictedtowhereitbelongs:thetrialcourts,ifanywhere.

Step5:EvaluatetheAnswerChoices

Page 54: LSAT PrepTest 81 Unlocked · Similarly, PT 81 was the first test since June '07 (PT June '07) to require at least 94 questions correct to get a 172 (99th percentile score). Number

(A)fitsbothauthors'beliefthatindependentresearchshouldconformtothefunctionofatrialcourt.

(B)isnotsupported.Bothpassagesraisethepossibilityofunreliablesources,butpassageAdoesnotlayoutlimitsonthesourcesofoutsideresearch.PassageBwarnsoftheunreliabilityofinternetsources,butdoescondonetheuseof"reliablesources"asanalternativeeither.

(C)isa180.Bothauthorsarguethatitshouldnotreplacesuchevidence.

(D)mixesthetwoopinions.However,passageAdoesnotaddresswhetheritshouldbeusedinappellatecourts,andpassageBneverdirectlyarguesthatitshouldbeusedintrialcourts.

(E)isaFaultyUseofDetail.OnlypassageBmentionsthis,andonlyincontextofusingindependentresearchinanappellatecourt.

23. (D)InferenceStep2:IdentifytheQuestionType

Thequestionasksforaphrasethatconveysaparticularmeaning.ThesekindofdefinitionquestionsareavariationonInferencequestions.Thedefinitionwon'tbedirectlystated,butitwillbeinferredfromthecontext.Inthiscase,thecorrectanswerwillbe"mostcloselyrelated"toanotherphrase,whichmeansthisquestionalsosharessomequalitieswithParallelReasoningquestionsfromtheLogicalReasoningsection.

Step3:ResearchtheRelevantText

StartbylookingatthecontextofthequotefrompassageA.Thatreferstojudgeswhoareconcernedwiththeirabilityto"conductfirst-rate

Page 55: LSAT PrepTest 81 Unlocked · Similarly, PT 81 was the first test since June '07 (PT June '07) to require at least 94 questions correct to get a 172 (99th percentile score). Number

research."ThispotentialforbadresearchisaddressedinthelastparagraphofpassageB.

Step4:MakeaPrediction

InthelastparagraphofpassageB,theauthorreferstothepotentialforanappellatecourtsubstituting"itsownquestionableresearchresults"(lines56–59),asentimentechoingtheconcernraisedinpassageA.

Step5:EvaluatetheAnswerChoices

(D)matchestheprediction.

(A)referstoexpertsandtheirknowledgebeyondwhatisprinted.Thishasnothingtodowithjudgesandtheirconcernabouttheirpoorresearchtechniques.

(B)isaboutjudgesparticipatinginquestioningwitnesses,whichdoesnotreflecttheauthorofpassageB'sconcernaboutjudgesdoingresearch.

(C)makesreferencetoliveresponsestoinformation,whichisnotthesameasbeingworriedaboutresearchingthatinformationinthefirstplace.

(E)referstousingoutsidematerial,butdoesnotmimictheconcernaboutresearchingsuchmaterialinaless-than-stellarway.

24. (B)InferenceStep2:IdentifytheQuestionType

ThequestionaskswhattheauthorofpassageBwouldbe"mostlylikelytotakeissuewith,"makingthisanInferencequestion.

Step3:ResearchtheRelevantText

Page 56: LSAT PrepTest 81 Unlocked · Similarly, PT 81 was the first test since June '07 (PT June '07) to require at least 94 questions correct to get a 172 (99th percentile score). Number

Thequestionstartswithareferencetolines39–43,inwhichtheauthorofpassageBclaimsthatpartiesinatrialcanperformcross-examinationonnewinformationtoensureitiscredibleandreliable.ThisgoescontrarytoideaspresentedinpassageAabouthowscientificevidence"ensuresconflictingandpartisantestimony."

Step4:MakeaPrediction

TheauthorofpassageBdoesnotsharetheconcernsthatscientificinformationwillbedefinitivelyproblematic.Thecorrectanswerwilladdressthisdisputedconcept.

Step5:EvaluatetheAnswerChoices

(B)iscorrect.AccordingtotheauthorofpassageB,cross-examinationhelpsmakesurespecializedknowledgecanbehandledandintroducedwithoutaproblem.

(A)isnotdisputedbytheauthorofpassageB.ThefirstlineofpassageBis"Regardlessofwhattrialcourtsmaydo...."TheauthorofpassageBmerelywantsindependentresearchtoberemovedfromappellatecourts,nottrialcourts.

(C)isOutofScopeforpassageB.Thediscussionofcross-examinationonlyappliestothetrialsathand,nottofuturetrials.

(D)isa180.TheauthorofpassageBwouldnotdisputethis.Ifanything,cross-examinationwouldconfirmthaterroneousdecisionscanbeexposed.

(E)isa180.TheauthorofpassageBdoesnotdisputethestructureofatrialcourtanditsabilitytoinvolveindependentresearch.PassageB

Page 57: LSAT PrepTest 81 Unlocked · Similarly, PT 81 was the first test since June '07 (PT June '07) to require at least 94 questions correct to get a 172 (99th percentile score). Number

merelyarguesthatitgoesagainstthefunctionofanappellatecourt.

25. (C)InferenceStep2:IdentifytheQuestionType

ThisisavariationofInferencequestionthatasksfordefiningatermwithinthecontextofthepassage.Inthiscase,thecorrectanswerwon'tprovidetheactualdefinition,butwillbeanotherwordthathasthesamedefinitionincontext,whichalsomakesthissimilartoParallelReasoningquestionsfromLogicalReasoning.

Step3:ResearchtheRelevantText

Startbylookingatthewordcrucibleincontext.Theentiresentencesaysthatnewliteratureintroducedattheappellatelevel"cannotbetestedinthecrucibleoftheadversarialsystem."Thisrefersbacktothepreviousparagraph,inwhichtheauthordiscussesthetestingofnewinformationthroughthecriticalprocessofcross-examination.

Step4:MakeaPrediction

ThecorrectanswerwilllikelybeawordtakenfromthesecondparagraphofpassageBthatdirectlyrelatestotheprocessofcross-examination.

Step5:EvaluatetheAnswerChoices

(C)isamatch.Theengineinline42directlyreferstotheprocessofcross-examination.

(A)doesnotmatch.Thisreferstoadesiretoconductresearch,nottheprocessofcross-examination.

(B)doesnotmatch.Cross-examinationisusedtotestcredibility,butit'sthepracticeofcross-examinationthatisimportant(i.e.,thecrucible),notthecredibilityoftheevidence.

Page 58: LSAT PrepTest 81 Unlocked · Similarly, PT 81 was the first test since June '07 (PT June '07) to require at least 94 questions correct to get a 172 (99th percentile score). Number

(D)doesnotmatch.Thefunctioninline53referstothepurposeofconductingthetrial(fact-finding),notaspecificprocess(cross-examination)thatisusedinthatfunction.

(E)doesnotmatch.Thisreferstothesourceofinformation(e.g.,magazine,journal),whichhasnothingtodowiththeprocessofcross-examination.

26. (B)LogicReasoning(ParallelReasoning)Step2:IdentifytheQuestionType

Thecorrectanswerwillbeapairoftitlesthatindicatearelationship"mostanalogous"tothatbetweenthetwopassages.ThatmakesthisaLogicReasoningquestion,similartoParallelReasoning.

Step3:ResearchtheRelevantText

Becauseitreferstotherelationshipbetweenbothpassagesasawhole,theentiretextisrelevant.

Step4:MakeaPrediction

Themajorrelationshipbetweenthesetwopassagesisthattheytakedifferentperspectivesontheuseofindependentresearch.Onesupportsit,andonerejectsit.However,theybothmaketheirjudgmentsindifferentcontexts.Thefirstjustsaysitcanbebeneficial,whilethesecondmerelyrejectsitsusageinappellatecourts.Thecorrectanswershouldhaveasimilarrelationship:Thefirstsupportingsomethinginsomecontexts,andthesecondrejectingthatideainaparticularcontext.

Step5:EvaluatetheAnswerChoices

(B)isamatch,withthefirstshowingsupportforsomething(salt)insomecases,andthesecondsayingdon'tdoitinaparticularcontext(people

Page 59: LSAT PrepTest 81 Unlocked · Similarly, PT 81 was the first test since June '07 (PT June '07) to require at least 94 questions correct to get a 172 (99th percentile score). Number

withhighbloodpressure).

(A)putsanegativespinonsaltinbothcases,whichgoescontrarytothepositivetakebytheauthorofpassageA.

(C)isHalf-Right/Half-Wrong.ThefirsttitlenicelymimicshowtheauthorofpassageAdefendsindependentresearchagainsttheconcernofjudges.However,thesecondtitletalksaboutinconclusiveresearch,whichisnotcomparabletothediscussioninpassageB.

(D)isOutofScope.Neitherpassageadvocatesindependentresearchasasubstituteforanything,andtheauthorofpassageBdoesnottalkaboutanythingcomingunderfire(i.e.,beingcriticizedbyothers).

(E)isHalf-Right/Half-Wrong.ThefirsttitledefinitelymatchesthesupportivestanceofpassageA.However,thelackofsomethinginasamplepopulationdoesnotmatchconceptsinpassageB.

27. (D)InferenceStep2:IdentifytheQuestionType

Thequestionasksaboutthe"stancesof"bothauthors,whichreferstotheirattitudesonatopic.Andthequestionaskshowthosestancescanbe"mostaccuratelydescribed."So,thestancesaren'tdirectlystated,buttheyaredirectlydeducedfromthelanguageofthepassages,makingthisanInferencequestion.

Step3:ResearchtheRelevantText

Bothpassagesareentirelyaboutindependentresearch,soallofthetextisrelevant.However,thequestiondoesaskdirectlyaboutitsusebytrialjudges,sosticktothatcontext.

Page 60: LSAT PrepTest 81 Unlocked · Similarly, PT 81 was the first test since June '07 (PT June '07) to require at least 94 questions correct to get a 172 (99th percentile score). Number

Step4:MakeaPrediction

Overall,theauthorofpassageAissupportiveoftrialjudgesusingindependentresearchasasupplement,eventhoughjudges'"concernshavesomemerit."Whilethere'ssomesuggestionthattheauthorofpassageBisokaywithindependentresearchintrialcourts,theprimaryfocusofpassageBisstillontheuseofsuchresearchintheappellatecourt,notthetrialcourt.Infact,intheveryfirstsentence,theauthorclaimsthattheargumentaboutappellatecourtsstands"[r]egardlessofwhattrialcourtsmaydo,"indicatingthattheauthorofpassageBultimatelyisn'tconcernedaboutitsuseintrialcourts.Thecorrectanswershouldindicateattitudesofsupportforthefirstauthorandrelativeambivalenceforthesecond.

Step5:EvaluatetheAnswerChoices

(D)matchestheattitudesperfectly.

(A)isaDistortioninthattheauthorofpassageAnevercomesacrossasresigned,anda180,ifanything,aspassageBneverseemstodisapproveofindependentresearchbytrialjudges.

(B)isinaccurateinsuggestingthattheauthorofpassageAisambivalent.

(C)isa180fortheauthorofpassageA,whoisnotskeptical.AndtheauthorofpassageBdoesnotseemtobeharboringhostilitytowardindependentresearchbytrialjudges.

(E)isExtremefortheauthorofpassageA,whoiscertainlysupportivebutnotquiteforceful.AndtheauthorofpassageBisopposedtoindependentresearchattheappellatelevel,butnotnecessarilyatthetriallevel.

Page 61: LSAT PrepTest 81 Unlocked · Similarly, PT 81 was the first test since June '07 (PT June '07) to require at least 94 questions correct to get a 172 (99th percentile score). Number

LSATPrepTest81

Page 62: LSAT PrepTest 81 Unlocked · Similarly, PT 81 was the first test since June '07 (PT June '07) to require at least 94 questions correct to get a 172 (99th percentile score). Number

SECTIONII:LOGICALREASONING

Q# QuestionType Correct Difficulty

1 Paradox D Checkyouronlineresources.

2 Inference B Checkyouronlineresources.

3 Assumption(Sufficient) D Checkyouronlineresources.

4 Weaken B Checkyouronlineresources.

5 Assumption(Necessary) E Checkyouronlineresources.

6 Principle(Identify/Strengthen) B Checkyouronlineresources.

7 Inference C Checkyouronlineresources.

8 Flaw D Checkyouronlineresources.

9 Assumption(Sufficient) C Checkyouronlineresources.

10 Principle(Apply/Inference) E Checkyouronlineresources.

11 Assumption(Necessary) B Checkyouronlineresources.

12 Paradox A Checkyouronlineresources.

13 Strengthen A Checkyouronlineresources.

14 MainPoint C Checkyouronlineresources.

15 Inference D Checkyouronlineresources.

16 Strengthen/Weaken(EvaluatetheArgument) A Checkyouronlineresources.

17 Inference D Checkyouronlineresources.

18 RoleofaStatement A Checkyouronlineresources.

Page 63: LSAT PrepTest 81 Unlocked · Similarly, PT 81 was the first test since June '07 (PT June '07) to require at least 94 questions correct to get a 172 (99th percentile score). Number

19 Inference C Checkyouronlineresources.

20 Flaw A Checkyouronlineresources.

21 Principle(Parallel) B Checkyouronlineresources.

22 Weaken D Checkyouronlineresources.

23 Assumption(Necessary) A Checkyouronlineresources.

24 ParallelFlaw C Checkyouronlineresources.

25 Flaw D Checkyouronlineresources.

26 PointatIssue E Checkyouronlineresources.

1. (D)ParadoxStep1:IdentifytheQuestionType

Thequestionasksforsomethingthat"mosthelpsexplain"asituation.ThatmakesthisaParadoxquestion.

Step2:UntangletheStimulus

WithParadoxquestions,lookforasurprisingcontrast.Inthiscase,asmallanimalcalledthedunnartisbornwiththinskin,whichisunusualbecausemostanimalsofitskindneedthickskinforbodywarmthandwaterretention.

Step3:MakeaPrediction

Themysteryisthis:Howdoesthedunnartsurvivewiththinskinifanimalsofitskindnormallyneedthickskin?Theauthorgivesahintbystatingthattheskindoesgetthickerasthedunnartmaturesinitsmother'spouch.So,theremaybesomethingaboutthatpouchthathelpsprovide

Page 64: LSAT PrepTest 81 Unlocked · Similarly, PT 81 was the first test since June '07 (PT June '07) to require at least 94 questions correct to get a 172 (99th percentile score). Number

thenecessarybenefitsofthickskin(maintainingbodytemperatureandreducingwaterloss)untilthedunnartmatures.

Step4:EvaluatetheAnswerChoices

(D)iscorrect.

(A)isirrelevant.Evenwitharespiratorysystem,thedunnartstillhasthinskinthatwon'thelpmaintainbodytemperatureorreducewaterloss.

(B)isanIrrelevantComparison.Evenifthisistrue,thethinskinwon'thelpmaintainthatbodytemperature,nordoesthisaddresstheretentionofwater.

(C)isanIrrelevantComparison.Theparadoxisaboutnewborns,notadults.

(E)doesnothelp.Thisdoesnotaddresswhathappensatnight,whentemperaturesmaycooldrastically.Nordoesitaddressotherdunnartsthatmaynotliveinsuchafortunateenvironment.

2. (B)InferenceStep1:IdentifytheQuestionType

Thecorrectanswerwillfillintheblankattheendofthestimulus.ThatblankisprecededbytheKeyword[t]hus,whichindicatesthattheblankwillbeaconclusionsupportedbytheprevioustext.Somethingsupportedbytheinformationgivenisaninference.

Step2:UntangletheStimulus

Theauthordiscussesstand-upcomedianswhocanholdanaudience'sattentionforhoursandmakeinterestingpoints.Thisisaccomplishedby

Page 65: LSAT PrepTest 81 Unlocked · Similarly, PT 81 was the first test since June '07 (PT June '07) to require at least 94 questions correct to get a 172 (99th percentile score). Number

usinghumor.Universityprofessorswanttoachievethesameresults.

Step3:MakeaPrediction

Ifprofessorswanttoachievethesameresults,thenitmakessensetoconcludethattheyshouldusethesametechnique:humor.

Step4:EvaluatetheAnswerChoices

(B)matchestheprediction.

(A)isnotsupported.Itisstatedthatprofessorshopetoachievethesameresultsasstand-upcomedians,whichsuggeststheymaynotnecessarilyhavethesameskills.

(C)isExtreme.Theauthorissuggestingthathumormightbeavalidtechnique,butthere'snosuggestionit'stheonlysolution.

(D)isExtreme.Humormaymakesomelonglecturesinteresting,butthatdoesn'tmeanthere'snowayitwillavoidlosingtheaudience'sattention.Evenwithhumor,therecouldbesomethingelsethatmakesthelectureunengaging.

(E)isExtreme.Somecomediansmightbeabletoaddresscertainserioustopicsusinghumor,buttheauthordoesn'tnecessarilysuggesthumorwouldbeacceptableforeverytopic,includingthemostserious.

3. (D)Assumption(Sufficient)Step1:IdentifytheQuestionType

Theargumentpresentedwillbelogicallysoundifthecorrectanswerisassumed.So,thecorrectanswerwillbeasufficientassumption,i.e.,an

Page 66: LSAT PrepTest 81 Unlocked · Similarly, PT 81 was the first test since June '07 (PT June '07) to require at least 94 questions correct to get a 172 (99th percentile score). Number

assumptionthatisgoodenough,whenaddedtotheevidenceprovided,toguaranteetheconclusion.

Step2:UntangletheStimulus

Thereviewerconcludes([s]o)thattheadviceinmanagementbookswon'tbeveryusefulformostmanagers.TheevidenceisthatmostmanagersarenotCEOs,andmanagementbooksarewrittenfromaCEOperspective.

Step3:MakeaPrediction

Thisargument,likealmostallSufficientAssumptionarguments,ishinderedbyMismatchedConcepts.Theconclusionisabouttheusefulnessofthebooks,whiletheevidencemerelytalksaboutperspective.Thereviewerassumesthoseconceptsaresomehowconnected.Morespecifically,thereviewerassumesthatreaderswon'tfindbooksusefulifthosebooksarewrittenfromadifferentperspectivethantheirown.

Step4:EvaluatetheAnswerChoices

(D)iscorrect.Thisissayingabookneedstobewrittenfromthereader'sperspectivetobeuseful,i.e.,ifit'snotwrittenfromthatperspective,itwon'tbeuseful.

(A)isnotgoodenough.Evenifthisweretrue,there'sstillnoevidencetosupportwhetherornotsuchbooksareuseful.

(B)isOutofScope.Itdoesn'tmatterwhatreaderswanttobe.Theargumentisbasedontheircurrentperspectives,andthisoffersnosupportforwhetherornotthebookswillbeuseful.

Page 67: LSAT PrepTest 81 Unlocked · Similarly, PT 81 was the first test since June '07 (PT June '07) to require at least 94 questions correct to get a 172 (99th percentile score). Number

(C)isirrelevant.EvenifCEOswereoncelowermanagers,theirperspectivescouldhavechangedwhentheybecameCEO.Regardless,thisisstillnotenoughtoreachtheconclusionaboutwhetherornotthebooksareuseful.

(E)isirrelevant.Itdoesn'tmatterwhatmanagersprefertoread.Thisstilloffersnoevidencetoguaranteetheconclusionaboutmanagementbooksandwhetherornottheyare,indeed,useful.

4. (B)WeakenStep1:IdentifytheQuestionType

Thequestionasksforsomethingthat"underminesthemayor'sdefense,"whichmeansitwillweakenthatargument.

Step2:UntangletheStimulus

Themayorisbeingaccusedoftakingabribeintheformofhomeimprovementstohisvacationhome.Themayorarguesitwasn'tabribe.Hisevidenceisthathepaideverybillforthatprojectthatwaspresentedtohim.

Step3:MakeaPrediction

Themayoristryingtosuggesthepaidfortheproject.However,hedoesn'tsayhepaidallthebillsforthatproject.Hejustclaimshepaidallthebillspresentedtohim.Whataboutallofthebillsthatwerenotpresentedtothemayor?Ifthemayorletsomebodyelsepayforthose(say,forexample,acityconsultantwhowantstofinanceaniceprojectforthemayorinreturnforcontinuedsupport),thenhisdefensefallsapart.

Step4:EvaluatetheAnswerChoices

(B)iscorrect,attackingthemayor'spresumptivesuggestionthathewasactuallypayingforthewholeproject.

Page 68: LSAT PrepTest 81 Unlocked · Similarly, PT 81 was the first test since June '07 (PT June '07) to require at least 94 questions correct to get a 172 (99th percentile score). Number

(A)isOutofScope.Themayorismerelydefendinghimself.Whethertheconsultanttookbribesornotisirrelevant.Evenso,thischoiceonlystatesthatauthoritiesareinvestigatingthesituation,whichmeansit'sstillpossibletherewasnobriberyonanyone'saccount.

(C)isirrelevant.Itdoesn'tmatterwhodidthework.Forthequestionofbribery,allthatmattersiswhopaidforit.

(D)isirrelevant.Theactualcostdoesn'tmatter.Ifthemayorpaidforit,it'snotabribe.

(E)isanIrrelevantComparison.Theconsultant'ssalaryfromthecitycouldjustbealegitimatelysizableamount,anditdoesn'tmatterhowthatsalarycomparestothecostofthemayor'svacationhomeimprovements.Allthatmattersiswhopaidforthemayor'simprovements,andthisoffersnoreasontoquestionthemayor'sclaims.

5. (E)Assumption(Necessary)Step1:IdentifytheQuestionType

Thequestiondirectlyasksforanassumption,andonethatis"requiredbytheargument,"makingthisaNecessaryAssumptionquestion.

Step2:UntangletheStimulus

Thearchaeologistisrejectingacommonbelief,essentiallyarguingthathumansdidnotneedfiretomigratetothecoldclimateofEurope.Theevidenceisthattheearliestcontrolledfiresdatebackjust400,000years.

Step3:MakeaPrediction

Page 69: LSAT PrepTest 81 Unlocked · Similarly, PT 81 was the first test since June '07 (PT June '07) to require at least 94 questions correct to get a 172 (99th percentile score). Number

Toarguethatfirewasn'tnecessary,thearchaeologistmustbelievethathumanswereabletosurviveincoldEuropebeforetheycouldcontrolfire.Ifcontrolledfiregoesback400,000years,thenthearchaeologistmustassumethatpeoplewereinEuropebeforethattime,andhencewereabletosurvivewithoutfire.

Step4:EvaluatetheAnswerChoices

(E)mustbeassumed.UsingtheDenialTest,ifnobodywasinEuropeearlierthan400,000yearsago,thenthemigrationtoEuropehappenedafterfirewascontrolled.Inthatcase,thearchaeologisthasnoreasontosuggestfirewasn'tneeded.So,thearchaeologistmustbelievetherewerepeopleinEuropeearlier,beforefirewascontrolled.

(A)isnotnecessary.Thearchaeologist'sargumentcouldbevalidwhetherearlyhumansusedfireforcookingornot.

(B)isanIrrelevantComparison.Itdoesn'tmatterwhetherit'scoldernoworitwascolderbackthen.Wasfireneededornot?That'sthefocusoftheargument.

(C)isOutofScope.Whetherhumansutilizednaturalfiresornot,theargumentisabouttheneedformasteryoffire.Thissuggestspeopledidusefirebeforeitwascontrolled,butthearchaeologistdoesnotneedthattobetruetoclaimthatmasteryoffirewasnecessaryformigration.

(D)isExtreme.Thissuggeststhattheneedforheatwastheonlyreasonhumansmasteredfire.Evenifthatweren'ttrue(i.e.,evenifhumanswouldhavemasteredfireforotherreasons),thearchaeologist'sargumentisnotaffected.

Page 70: LSAT PrepTest 81 Unlocked · Similarly, PT 81 was the first test since June '07 (PT June '07) to require at least 94 questions correct to get a 172 (99th percentile score). Number

6. (B)Principle(Identify/Strengthen)Step1:IdentifytheQuestionType

Thequestiondirectlyasksforaprinciple,whichwillbefoundinthecorrectanswer,makingthisanIdentifythePrinciplequestion.Further,thecorrectanswerwill"helptojustify"theargument,whichmeansthisquestionwillalsoutilizetheskillsofaStrengthenquestion.

Step2:UntangletheStimulus

Theastronomerusesacommonargumentativetechnique:negatingtheviewsofanopponent.Somepeoplearguethataspacetelescopeprojectshouldbecancelledforbeingoverbudget.Theastronomersaysotherwise,i.e.,don'tcanceltheproject.Theevidenceisthatcancellingtheprojectwouldbeawasteofthemoneyalreadyspent,whichisgreaterthantheamountneededtofinishtheproject.

Step3:MakeaPrediction

Theastronomer'sargumentisbasedonaprincipleofmoney.Theargumentwouldbejustifiediftheastronomerheldthesamefinancialprincipleforanyproject:don'tcancelitifthemoneyalreadyspentisgreaterthantheremainingcost.

Step4:EvaluatetheAnswerChoices

(B)iscorrect.Ifthespaceagencyhasalreadyspentmorethantheremainingcostsandisalreadyoverbudget,thenithasalreadyspentmost(i.e.,morethanhalf)ofthetotalcost.Bythisprinciple,theprojectshouldbecompleted,i.e.,notcancelled,astheastronomerargues.

(A)isOutofScope.Theastronomerdoesnotrefertotheagency'soverallbudget,norhowsmallorlargethetelescopeprojectiswithrespecttotheoverallbudget.

Page 71: LSAT PrepTest 81 Unlocked · Similarly, PT 81 was the first test since June '07 (PT June '07) to require at least 94 questions correct to get a 172 (99th percentile score). Number

(C)isOutofScope,andapotential180.Theprojectissaidtobe"wayoverbudget,"butthere'snoindicationwhetherthismeansmorethantwicetheoriginalbudget.Inanyevent,thisprincipleoffersareasontocanceltheproject,whichtheastronomeristryingtoavoid.

(D)isa180.Thisissuggestingthattheagencyshouldn'tspendanymoremoneyontheproject,whichsoundsalotlikesayingtheprojectshouldbecancelled—contrarytotheastronomer'splea.

(E)isOutofScope.Theargumentisnotbasedonthelikelihoodofimportantnewdiscoveries.Also,it'saboutwhetheraprojectshouldbecanceledornot,notwhatshouldgetfundinginthefirstplace.

7. (C)InferenceStep1:IdentifytheQuestionType

Thecorrectanswerwillbe"stronglysupportedby"theinformationprovided,makingthisanInferencequestion.

Step2:UntangletheStimulus

Thenaturalistclaimsthatdifferentprimatescanbehaveindifferentways.Thisisillustratedbytwoexamplesdescribinghowtwodifferentprimates(achimpanzeeandanorangutan)wouldbehaveifazookeeperdroppedascrewdrivernearby.Thechimpwouldplayaroundwithitalittle,thenmoveontosomethingelse.Theorangutanwouldpretendtoignoreit,thenuseittotearapartthecagewhenthezookeeperleaves.

Step3:MakeaPrediction

Twoverydifferentbehaviors,indeed.Thechimpwouldactcurious,butquicklygetbored.Theorangutan,ontheotherhand,wouldbequitecunning—playingitcoolandwaitingforthezookeepertoleavebefore

Page 72: LSAT PrepTest 81 Unlocked · Similarly, PT 81 was the first test since June '07 (PT June '07) to require at least 94 questions correct to get a 172 (99th percentile score). Number

carryingoutthedeviousplan.Thecorrectanswerwillbebaseddirectlyontheseobservations.Don'tmakeanyassumptionsaboutwhatthisbehaviormightindicate.Thecorrectanswermustbedirectlysupported.It'salsoimportanttonotethatthesearejusttwoexamples.Bewaryofanswersthatmakeoverlybroadclaimsfromtheseexamples.

Step4:EvaluatetheAnswerChoices

(C)issupported,astheorangutanissaidto"pretendtoignore"thescrewdriver,justtodeceivethezookeeperintothinkingnothingisgoingtohappen—untilthezookeeperleaves.Then,we'veallseenPlanetoftheApes,soweknowwhathappensnext.

(A)isExtremeandOutofScope.Somemightconsidertheorangutan'splanasignofhighintelligence,butthat'snotdirectlysupported.Inanyevent,thestimulusonlymentionstwoprimates.Withoutknowinghowotherprimateswouldbehave,there'snosupportthatorangutanswouldbethemostintelligent.

(B)isnotsupported.Walkingawayfromthescrewdriverdoesn'tmeanthechimphasaninferiormemory.

(D)isnotsupported.Whiletheorangutan'splanmightcertainlyindicateadislikeforbeingcaged,there'snoevidencethatthechimpisanylessdispleased.Perhapsthechimpjustdidn'ttakethetimetoconcoctasdestructiveanddeviousaschemeastheorangutan.

(E)isnotsupported.Walkingawayfromthescrewdriverdoesnotnecessarilyindicatethatthechimpdidn'tunderstanditsuse.Perhapsthechimpunderstoodbutjustwasn'tinterested.

Page 73: LSAT PrepTest 81 Unlocked · Similarly, PT 81 was the first test since June '07 (PT June '07) to require at least 94 questions correct to get a 172 (99th percentile score). Number

8. (D)FlawStep1:IdentifytheQuestionType

Thequestiondirectlyaskswhy"themanager'sargumentisflawed,"makingthisaFlawquestion.

Step2:UntangletheStimulus

Themanagerconcludes([t]hus)thatLiangshouldnotreceiveabonus.Theevidenceisthatbonusesonlygotoexceptionallyproductiveemployees,andLiangworksinadivisionthatisnotexceptionallyproductive.

Step3:MakeaPrediction

PoorLiang;themanagerisdenyingherabonusbecauseoftheperformanceofherdivision.However,theruleforbonusesisbasedontheproductivityoftheemployeeindividually,notthedivisionthatemployeebelongsto.Themanager'sreasoningisthusunsound,judgingLiangonhergroup'sperformanceratherthanherownindividualperformance.

Step4:EvaluatetheAnswerChoices

(D)pointsoutthemanager'serror,judginganindividualmember(Liang)basedtheperformanceofhergroup(thewholedivision).

(A)isanIrrelevantComparison.Itdoesn'tmatterhowthestandardscomparefromonedivisiontothenext.Ifthegroupdidn'treachitsownuniqueproductivitygoals,thenthemanagerhasarighttosayit'snotexceptionallyproductive.

(B)isOutofScope.Theprofitabilityofthecompanyhasnothingtodowiththeargumentathand,whichfocusesonbonusesandproductivity.

Page 74: LSAT PrepTest 81 Unlocked · Similarly, PT 81 was the first test since June '07 (PT June '07) to require at least 94 questions correct to get a 172 (99th percentile score). Number

(C)isaDistortion.Themanagerusesagroup'sperformanceasabasisforjudgingoneindividualwithinthatgroup,notforjudgingadifferentgroup.

(E)isOutofScope.ThemanagerisnotassumingLiangwon'tbeexceptionallyproductiveinthefuture.Shejustwasn'tproductivethisyear(allegedly),andthat'sallthatmattersfortheassignmentofbonuses.

9. (C)Assumption(Sufficient)Step1:IdentifytheQuestionType

Thequestiondirectlyasksforsomethingassumed,andtheargumentwillbelogicalifthatassumptionisinplace,makingthisaSufficientAssumptionquestion.

Step2:UntangletheStimulus

Theauthorconcludes([t]hus)thatthejournalistinquestionisdefinitelygoingtorevealherinformant'sidentity.TheevidenceincludessomeFormalLogic:Ifthejournalist'seditororajudgeordershertorevealtheidentity,shewill.

Step3:MakeaPrediction

BytheFormalLogic,therearetwothingsthatwouldensurethesourcebeingrevealed:thejudgeorderingitortheeditororderingit.Thecorrectanswershouldconfirmoneofthosetwothingswillhappen.Therearetwootherideastoconsider:1)Theauthorstatesthattheinformationconcernssafetyviolations.It'sunclearhowthisfactorsintotheargument,soitappearstobeaglaringMismatchedConcept.There'sagoodchancethecorrectanswerwillshowhowconcernsoversafetyviolationswillleadtoadefiniterevealofthesource;2)Atthebeginning,

Page 75: LSAT PrepTest 81 Unlocked · Similarly, PT 81 was the first test since June '07 (PT June '07) to require at least 94 questions correct to get a 172 (99th percentile score). Number

thejournalistpromisedhersourcethatshewouldn'trevealthesource'sidentify—aslongastheinformationisnotfalse.

If~false→~reveal

Ifreveal→false

However,thatdoesn'tmeanthejournalistwilldefinitelyrevealthesourceiftheinformationisfalse,sothatcouldbeatrapanswer.Andeveniftheinformationisaccurate,theFormalLogicsuggeststhatacourtorderoraneditor'sorderwouldsupersedethatandrequirehertobreakthatpromise.Inshort,thepromiseisultimatelyanon-issueandshouldnotbefactoredintotheassumption.

Step4:EvaluatetheAnswerChoices

(C)iscorrect.Theinformationdoesconcernsafety.So,accordingtothislogic,ajudgewillordertheidentitytoberevealed.Thus,bytheFormalLogicintheevidence,theconclusionisconfirmed:thesource'sidentitywillberevealed.

(A)isnotgoodenough.Thejournalistpromisedtokeeptheidentitysecretiftheinformationwasnotfalse.However,thatdoesn'tmeanshewoulddefinitelyrevealtheidentifyiftheinformationwasfalse.That'sanimproperuseofFormalLogic.

(B)isaDistortion.Bythislogic,itwouldbenecessarythattheinformationbesafety-relatedfortheeditortodemandaname.However,it'snotsufficient.Eventhoughthetheinformationis,infact,safety-related,thislogicdoesnotguaranteethattheeditorwilldemandtheidentityberevealed.

Page 76: LSAT PrepTest 81 Unlocked · Similarly, PT 81 was the first test since June '07 (PT June '07) to require at least 94 questions correct to get a 172 (99th percentile score). Number

(D)isaDistortion.Evenifrevealingthesourceistheonlywaytoverifytheinformation,thatdoesn'tguaranteethejournalistwillbreakherpromiseandrevealtheidentity.

(E)isOutofScope.Whattheinformantunderstandsisirrelevant.Thisshowsthattheinformantwasn'tignorant.Theinformantknewthatajudge'sorderwouldoverrideanypromisemadebythejournalist.However,it'sstillnotsaidwhethersuchanorderwasmade,sothere'snoreasontobelievethejournalistwouldbreakherpromisejustyet.

10. (E)Principle(Apply/Inference)Step1:IdentifytheQuestionType

Thestimuluswillcontainaprinciplethatwillbeusedtosupportthecorrectanswer.Becausetheprincipleisprovidedinastimulus,thisisanApplythePrinciplequestion.Andthecorrectanswerwillbedirectlysupportedbythatprinciple,makingthissimilartoanInferencequestion.

Step2:UntangletheStimulus

TheprincipleisjustonebigpieceofFormalLogic:Ifit'snotdifficulttoreturnaborroweditemontimeandtheitem'sownerdidn'tsayyoucouldreturnitlate,thenyoushouldreturntheitemwhenyoupromised.

If~difficultAND~permission→returnitontime

If~returnitontime→difficultORpermission

Step3:MakeaPrediction

Theruleisprettystraightforward.Therearetwothingstonote:1)Thisruleonlyappliestopeoplewhopromisedtoreturnitbyacertaintime,thusifnopromiseismade,thentheprincipledoesn'tapply;and2)donot

Page 77: LSAT PrepTest 81 Unlocked · Similarly, PT 81 was the first test since June '07 (PT June '07) to require at least 94 questions correct to get a 172 (99th percentile score). Number

simplynegatethelogic.Ifit'snotdifficulttoreturnitandyoudon'thavepermissiontokeeptheitemlate,thenreturntheitemontime.That'stherule.Ifitisdifficultoryoudohavepermission,theprincipledoesn'tapply.Itmightbeokaytoreturntheitemlate,butyoucan'tlogicallyconcludethatitabsolutelyis.Perhapsit'sstilltherightthingtosticktoyourpromiseandgetitbackontime.

Step4:EvaluatetheAnswerChoices

(E)fitstheprinciple.Apromisewasmade,returningtheitemontimeisnotdifficult,andthere'snopermissiontokeepitlate.Thus,astheFormalLogicdictates,theitemshouldbereturnedontime.

(A)doesnotmatch.EventhoughChristophergavepermissiontoreturnthebooklate,thatdoesn'tmeanthere'sanythingwrongwithreturningitearly.Theprincipledoesn'tdenythat.

(B)doesnotmatch.TheFormalLogiccontainsthewordand:Ifyoudon'thavepermissionanditisn'tdifficulttoreturntheitem,thenreturnitontime.Onlyoneconditionismethere(Wandadidn'tgivepermission).Ifitisdifficulttoreturnthebicycleontime,thenitmaystillbeokayforNicktoreturnitlate.

(C)doesnotmatch.Onlyoneconditionismethere:It'snotdifficulttoreturnthecar.However,Tedgavepermissiontoreturnthecarlate,sotheprinciplenolongerapplies.

(D)doesnotmatch.Yeseniadidnotpromisetoreturnthecomputerbyacertaindate,andtheprincipleonlyappliestopeoplewhodomakesuchapromise.WhileitseemsreasonabletosuggestYeseniashouldreturnthe

Page 78: LSAT PrepTest 81 Unlocked · Similarly, PT 81 was the first test since June '07 (PT June '07) to require at least 94 questions correct to get a 172 (99th percentile score). Number

computerontime,itdoesnotconformtotheconfinesoftheprincipleinquestion.

11. (B)Assumption(Necessary)Step1:IdentifytheQuestionType

Thequestiondirectlyasksforanassumption,andonethattheargumentrequires,makingthisaNecessaryAssumptionquestion.

Step2:UntangletheStimulus

Theauthorpresentsevidenceoftwogaseoussubstances.Theybothattractmosquitoes,butabarearmattractsmosquitoesmorethaneitherone.Theauthorconcludes([t]herefore)thatthehumanarmmustgiveoffadifferentgaseoussubstance.

Step3:MakeaPrediction

ThisisaclassiccaseofOverlookedPossibilities.Theauthorhasruledouttwopossiblegaseoussubstancesandthenconcludesthatitmustbeanothergaseoussubstance.Whydoesithavetobeagaseoussubstanceatall?Whycan'ttherebesomeotheraspectofthehumanarmthatisattractingmosquitoes?Theauthordoesnotconsiderthatandassumestherearenootherfactors.Thecorrectanswerwillstatethatgenerallyorintroduceaspecificalternativethattheauthorassumesisnon-existent.

Step4:EvaluatetheAnswerChoices

(B)mustbeassumed.Theauthorassumesthemosquitoesareattractedbyagaseoussubstanceandnothingelse—notevenbodyheat.UsingtheDenialTest,ifmosquitoeswereattractedbybodyheat,thentheauthor'spersistencewithgaseoussubstanceswouldbeseriouslyquestioned.

(A)isnotnecessary.Itdoesn'tmatterwhethermosquitoescommunicatewitheachotherornot.Theargumentisaboutwhatattractsthemtothe

Page 79: LSAT PrepTest 81 Unlocked · Similarly, PT 81 was the first test since June '07 (PT June '07) to require at least 94 questions correct to get a 172 (99th percentile score). Number

humanarminthefirstplace.

(C)isanIrrelevantComparison.Theauthordoesclaimthatmosquitoesareattractedtoabarearm"evenincompletedarkness,"butthatstillcouldmeanthey'reequallyattractedinbroaddaylight.Whenthearmgivesoffmoresubstanceshasnoeffectontheauthor'sclaims.

(D)isanIrrelevantComparison.Theargumentisaboutwhatattractsthemosquitoes,notwhenthey'remostsuccessful.

(E)isExtremeandOutofScope.Theargumentisaboutwhatattractsmosquitoes.Whetherornotourskincouldeverrepelmosquitoeshasnothingtodowithwhathappenswhenmosquitoesareattracted.

12. (A)ParadoxStep1:IdentifytheQuestionType

Thequestionasksforsomethingthatwould"resolvetheapparentdiscrepancy"described,makingthisaParadoxquestion.

Step2:UntangletheStimulus

TwoanalysesweredoneonanItalianpainting,onein1955andanotherin2009.Bothanalysesfoundcobaltinthepaint,apigmentnotuseduntil1804.Basedonthat,the1955analysislogicallyconcludedthepaintingwasproducedafter1804,butthe2009analysissaidotherwise—itwasproducedbefore1804.

Step3:MakeaPrediction

Ifcobaltwasn'tuseduntil1804,whywouldthe2009analysissuggestthepaintingwasolderthanthat?Theauthormusthaveomittedsomethingcriticalaboutthat2009analysis.Itmaybedifficulttopredictanexact

Page 80: LSAT PrepTest 81 Unlocked · Similarly, PT 81 was the first test since June '07 (PT June '07) to require at least 94 questions correct to get a 172 (99th percentile score). Number

explanation,butknowthatthecorrectanswerwillprovideareasonwhythe2009analysisdatedthepaintingtosometimeearlierthancobaltwasfirstused.

Step4:EvaluatetheAnswerChoices

(A)offersanexplanation.The2009analysisshowedcobaltwasonlyfoundinupperlayersthatwereaddedtoolder,damagedlayers.Ifthat'strue,thenthetoplayerwithcobaltwouldbefromafter1804,buttheoriginal,olderlayerscouldbefromanytimebeforethat.

(B)doesnothelp.Evenifthenewtechnologyismoresophisticated,itstillfoundtracesofcobalt,sothere'snoexplanationwhyanalyststhoughthepaintingwasproducedbeforetheuseofcobalt.

(C)isirrelevant.Itdoesn'tmatterhowmanysamplesweretaken.Cobaltwasfoundinbothcases,sothere'snoexplanationwhyanalystswouldsuggestthepaintingwasolderthanthefirstuseofcobalt.

(D)isirrelevant.Regardlessofwhattheexpertsthink,thepaintingstillhadcobalt,whichwasn'tuseduntil1804.There'snothingaboutthepaintanalysisthatsuggestsitshouldbeanyearlierthanthat.So,ifthere'sartisticevidencethatthepaintingisolderthan1804,thenthemysteryaboutthecobaltstillremains.

(E)isa180.ThissaysthattheuseofcobaltinItaly,thesourceofthepainting,wasrareinthefirstfewyearsafter1804.Thatwouldmakeitmorelikelythepaintingwasproducedevenlaterthanissuggested,whencobaltusewasperhapsmorecommon.

13. (A)Strengthen

Page 81: LSAT PrepTest 81 Unlocked · Similarly, PT 81 was the first test since June '07 (PT June '07) to require at least 94 questions correct to get a 172 (99th percentile score). Number

Step1:IdentifytheQuestionType

Thequestiondirectlyasksforinformationthat"strengthenstheargument,"makingthisaStrengthenquestion.

Step2:UntangletheStimulus

Toreducethespreadofinfluenza,acampaignwasrunforsixmonthstoencouragefrequenthand-washingandavoidingpublicplaceswhensick.Inthatsixmonths,therewerefewerincidencesofinfluenza,leadingtheauthortoconcludethecampaignwasasuccess.

Step3:MakeaPrediction

ThisisaprimeexampleofCorrelationvs.Causation.Fewerpeoplegotthefluduringthecampaign,sotheauthorarguesthecampaigncausedthefluratetodrop.However,theauthormayhaveidentifiedthewrongcause.Perhapstherewassomeotherreasonpeopleweren'tgettingtheflu.Theauthorsuggestsotherwise,assumingthatpeoplewere,indeed,justencouragedbythecampaigntowashtheirhandsmoreandstayawayfromthepublicwhensick.Thecorrectanswerwillconfirmthis,makingitmorelikelythatthecampaignwasresponsible.

Step4:EvaluatetheAnswerChoices

(A)iscorrect.Thisoffersmorereasontobelievepeoplewerewashingtheirhandsmoreoftenandthuslisteningtothecampaignmessages.

(B)isa180atworst.It'snotmentionedwhatcouldhelppreventthecommoncold.Ifwashinghandsandstayinghomewhensickcouldreducetheriskofgettingacold,thenthissuggestspeopleweren'tdoingthat.Inthatcase,fluratesweredownforanotherreason,notbecauseofthecampaign.

Page 82: LSAT PrepTest 81 Unlocked · Similarly, PT 81 was the first test since June '07 (PT June '07) to require at least 94 questions correct to get a 172 (99th percentile score). Number

(C)isa180.Thissuggeststhecampaignmayhavebeenirrelevant.Therejustmighthavebeenfeweropportunitiesforpeopletobeatlargegatheringswheretheinfluenzaviruscouldbeeasilyshared.

(D)isanother180.Thisdirectlyoffersanalternativeexplanationforthereductioninfluincidences.Thecampaignmayhavebeenirrelevantifpeoplewerejustwatchingthenews.

(E)doesnothelp.Thissuggeststhatpeoplerecognizedtheimportanceofreducingtheincidenceoftheflu,butitstilldoesn'tshowthatthecampaignwasthefactorthatfinallymadepeopletakeaction.

14. (C)MainPointStep1:IdentifytheQuestionType

Thequestionasksforthe"conclusiondrawnintheargument,"makingthisaMainPointquestion.

Step2:UntangletheStimulus

Thefirsttwosentencesprovidefactualresultsfromastudy.Fromtheseresults,theauthorconcludesthatmeetingsneedtohaveaclear,less-than-30-minutetimeframetobetrulyproductive.

Step3:MakeaPrediction

Thecorrectanswerwillbeaparaphraseoftheconclusioninthelastsentence,definingthecircumstancesneededtoachievemaximumproductivity.

Step4:EvaluatetheAnswerChoices

(C)matchestheauthor'sclaimthataclear,below-30-minutetimeframeisneededformaximumproductivity.

Page 83: LSAT PrepTest 81 Unlocked · Similarly, PT 81 was the first test since June '07 (PT June '07) to require at least 94 questions correct to get a 172 (99th percentile score). Number

(A)isaDistortion.Thisconfusesthelogicoftheconclusion,whichclaimsthatameeting"needstohave"aclear,less-than-30-minutetimeframe.Thatmakesitnecessary,butnotsufficient.Inotherwords,havingthattimeframedoesn'tnecessarilyguaranteethatanysuchmeetingwillbemaximallyproductive.

(B)isafactfromthestudy,andthefactsaremerelyevidencetosupporttheconclusion,nottheconclusionitself.

(D)isalsoafactfromthestudy,andfactsareevidencetosupporttheconclusion,nottheconclusionitself.

(E)ispartofthefacts,i.e.,theevidence.Thosefactsareusedtosupporttheconclusion.

15. (D)InferenceStep1:IdentifytheQuestionType

Thestimuluscontainsasetofstatements,andthecorrectanswerwillbe"stronglysupportedby"thosestatements.ThatmakesthisanInferencequestion.

Step2:UntangletheStimulus

Thenutritionistprovidesaninterestingcontrast.Mostfaddietsprescribethesamenutrientstoeveryone,butnoteveryonehasthesamedietaryneeds.Thenutritionistthentossesoutarandomrecommendationtoeatyourfruitsandvegetables.

Step3:MakeaPrediction

Theopeningcontrastsuggeststhatfaddietswon'tbeappropriateforeveryone.Asforthefruitsandvegetablescomment,don'treadtoomuch

Page 84: LSAT PrepTest 81 Unlocked · Similarly, PT 81 was the first test since June '07 (PT June '07) to require at least 94 questions correct to get a 172 (99th percentile score). Number

intoit.It'sjustarecommendation,buttherecouldstillbeplentyofotherfoodsthatareequallyhelpful.Thecorrectanswerwillconformtothislimitedinformation.Justbewaryofanswersthatgobeyondwhat'smentionedorexaggeratethenutritionist'sclaims.

Step4:EvaluatetheAnswerChoices

(D)issupported.Ifdifferentpeoplehaveverydifferentdietaryneeds,thenthosefaddietsthattelleveryonetoeatthesamefewnutrientsarenotgoingtosatisfyeveryone'sneeds;hence,somepeoplewillnotgetwhattheyneed.

(A)isaDistortion.Therecommendationtoeatfruitsandvegetableshasnologicalconnectiontothefaddiets,sothere'snowaytoconcludewhetherthosedietsincludefruitsandvegetablesornot.

(B)isExtreme.Thenutritionistrecommendsfruitsandvegetables,butnevergoessofarastosaytheyaretheonlyfoodstoprovidewidespreadhealthbenefits.Therecouldbeothersuchfoods.

(C)isalsoExtreme.Noteverybodyhasthesamedietaryneeds,butthatdoesn'tmeaneverysinglepersonisentirelydifferent.Therecouldbeagroupofpeoplewhoallhaveonesetofneeds,butthoseneedsarecompletelydifferentfromthoseofanothergroupofpeople,whoseneedsaredifferentfromanothergroup's,etc.

(E)isOutofScope.Thereisnoinformationaboutwhatfoodscontainwhatkindsofnutrients,noristhereinformationaboutwhichnutrientscanbefoundinanygivenfood.

16. (A)Strengthen/Weaken(EvaluatetheArgument)

Page 85: LSAT PrepTest 81 Unlocked · Similarly, PT 81 was the first test since June '07 (PT June '07) to require at least 94 questions correct to get a 172 (99th percentile score). Number

Step1:IdentifytheQuestionType

Thecorrectanswerherewillhelpin"evaluatingtheargument,"whichmakesthisanEvaluatetheArgumentvariationofaStrengthen/Weakenquestion.Thecorrectanswerwilltestthevalidityoftheargumentbyquestioningtheauthor'sassumption.

Step2:UntangletheStimulus

Thecaffeineincoffeecanproduceirritatingstomachacid,butdarkerroasts(i.e.,coffeeproducedbyroastingthebeanslonger)havemoreNMPthanlighterroasts,andNMPissomethingthathelpsreducestomachacidproduction.(Don'tworryaboutwhatNMPactuallyis—allthatmattersforthisargumentiswhatitdoes.)Therefore,theauthorconcludesthatdarkerroastsarelessirritating.

Step3:MakeaPrediction

It'sgoodtoknowthatdarkerroastscontainsomethingthathelpsreduceacidproduction.However,theauthorassumesthere'snothingelsedifferentaboutdarkerroaststhatcouldcounteractthatbenefit.Perhapsthereissomethingelseaboutdarkerroaststhatcouldactuallystimulatemoreacidproduction,despitetheaddedNMP.ThecorrectanswerwillquestionwhethertheNMPisenoughtoreduceacidlevelsoverallorwhetherthere'ssomeoverlookedfactor.

Step4:EvaluatetheAnswerChoices

(A)iscorrect.Ifthelongerbrewingtimedoesincreasethecaffeine,thenthatcouldeasilybalanceouttheNMP,makingdarkerroastsequallyirritable.However,ifthecaffeinelevelisthesame,thenitislikelythatdarkerroastsarebetterforyourstomach.

(B)isagreatquestiontoponder,butnotrelevanttotheargument.Theauthormerelyclaimsthedarkerroastswillbelessirritatingbyreducing

Page 86: LSAT PrepTest 81 Unlocked · Similarly, PT 81 was the first test since June '07 (PT June '07) to require at least 94 questions correct to get a 172 (99th percentile score). Number

acidproduction.Whetherornotthiscausesotherstomachfunctionproblemsisanentirelydifferentconcernanddoesnothingtoquestiontheauthor'sclaims.

(C)isOutofScope.Thereisnomentionofcoffeesthatcontainlesscaffeine,andtheauthor'sargumentisaboutreducingacidproduction,notreducingcaffeineintake.

(D)isclever,butdoesnotaffecttheauthor'sargument.Theauthormaywellagreethatmorecoffee(andtherebycaffeine)couldbeconsumediftheswitchtodarkroastwasmade,andifitwastoomuchifmightoffsetthebenefitsofdarkroast.However,theconclusionwasmerelyabouttheeffectofdarkerroastsversuslighterroasts.Anychangeinhabitsbeyondthatareirrelevant.

(E)isOutofScope.Theargumentisentirelyfocusedonacidproduction.Otherhealthbenefitsareinteresting,butnotrelevanttothisargument.

17. (D)InferenceStep1:IdentifytheQuestionType

Thecorrectanswerwillbe"stronglysupportedbythestatements"provided,makingthisanInferencequestion.

Step2:UntangletheStimulus

Theauthormentionshowdifficultitisforfilmhistorianstodeterminehowtypicalaudiencemembersrespondtocertainfilms.Twopossiblesourcesofinformationarepresentedasunhelpful:boxofficefigures—whichcan'tprovidedetailsaboutwhatpeopleactuallylikedaboutafilm—andmoviereviews.

Page 87: LSAT PrepTest 81 Unlocked · Similarly, PT 81 was the first test since June '07 (PT June '07) to require at least 94 questions correct to get a 172 (99th percentile score). Number

Step3:MakeaPrediction

Insayingthatboxofficefigures"helplittle"andnewspaperandmagazinereviews"failtoprovidemuchinsight,"theauthorisreferringtohowunhelpfultheyareintryingtodeterminethetypicalresponsefromaudiencemembers.There'snoindicationwhatwouldbehelpful,orifit'sevenpossible.Thecorrectanswershouldmerelybeconsistentindescribingthedifficultyhistoriansface,withoutbringinginoutsideinformationorexaggeratingtheclaims.

Step4:EvaluatetheAnswerChoices

(D)issupported.Thelastsentencesayssuchreviews"failtoprovidemuchinsight,"andthatinsightreferstohistorian'sattempttodeterminethetypicalaudiencemember'sview.

(A)isOutofScope.Theauthormakesnomentionwhyhistoriansdon'tfindsuchreviewsinsightful,andthere'snosuggestionwhetherthereviewsweregenerallywrittenbeforeorafterafilm'srelease.

(B)isaDistortion.Theauthormentionsthatit'sespeciallydifficulttodetermineaudienceviewsforearly20th-centuryfilms,butthatdoesn'tmakeiteasytodetermineaudienceviewsoflate20th-centuryfilms.Theymaystillbedifficulttodetermine,justalittlelessso.

(C)isaDistortion.Thisconfusesthedetailthatboxofficefiguresdonotindicatewhatpeoplefindfunny,frightening,ormoving.However,thatjustmeansthefigureswon'trevealthespecificcomponentsthatpeopleenjoyed.It'sstillpossiblethosecomponentsarecriticaltoamovie'ssuccess,eveniftheycan'tbeidentifieddirectly.

Page 88: LSAT PrepTest 81 Unlocked · Similarly, PT 81 was the first test since June '07 (PT June '07) to require at least 94 questions correct to get a 172 (99th percentile score). Number

(E)isnotsupported.Thehistoriansdon'thappentofindsuchreviewsinsightful,butthatdoesn'tmeantheyweren'tcommonlywritten.

18. (A)RoleofaStatementStep1:IdentifytheQuestionType

Thequestionstemprovidesaclaimfromthestimulusandasksforits"roleintheargument,"makingthisaRoleofaStatementquestion.

Step2:UntangletheStimulus

Theclaiminquestion(thecorewouldhaveapositivecharge)isinthefirsthalfofthelastsentence.Beforedealingwiththat,considertheauthor'soverallargument.Ingeneral,astronomersbelievepulsarsaregiantballsofneutrons.(Don'tgettoocaughtupinthescience.Simplifyingthedetailsinto"giantballsofneutrons"isenoughtostayfocusedonthestructureoftheargument.)AtthecontrastKeyword[h]owever,theauthorarguesthatthisdescriptionalsoworksforpulsarsthataregiantballsofquarks.Asevidence,theauthornoteshowaquark-filledpulsarwouldhaveapositivecharge,whichwouldattractparticlesthatcould"supportacrustofneutrons"—thuscreatingsomethingthatmightbemistakenfortheaforementioned"giantballofneutrons."

Step3:MakeaPrediction

It'seasytogetdistractedbyallofthescience.However,boiltheargumentdowntoasimplestructure.Scientistshaveadefinitionofsomethingcalledapulsar(it'saballofneutrons).Theauthorarguesthatdefinitionappliesevenwhenitappearsotherwise(whenit'saballofquarksinstead).Theauthorprovidesevidencethatexplainswhytheoddballpulsarsstillfittheoriginaldefinition(aballofquarkswouldhaveaneutronshell).Theclaiminquestionisinthelastpart,soitsroleisto

Page 89: LSAT PrepTest 81 Unlocked · Similarly, PT 81 was the first test since June '07 (PT June '07) to require at least 94 questions correct to get a 172 (99th percentile score). Number

provideevidencetoshowwhyquark-filledpulsarsstillcanstillhaveanoutercoatingofneutrons.

Step4:EvaluatetheAnswerChoices

(A)iscorrect,identifyingtheclaimasevidencetoexplainhownon-neutron-filledpulsars(i.e.,thosefilledwithquarks)couldstillattractneutrons.

(B)isaDistortionanda180.Theauthorneverchallengestheideathatpulsarscanbemadeofquarks.Theyabsolutelycan.Andtheclaiminquestionexplainshowtheywork.

(C)isOutofScope.Theauthorsaysnothingaboutsuchpulsarsgoingunrecognizedbyastronomers.

(D)isa180.Theclaimactuallyshowshowquark-filledpulsarsconformtotheconsensusview,notchallengeit.

(E)isaDistortion.Theauthorneverquestionsthemassofpulsars.

19. (C)InferenceStep1:IdentifytheQuestionType

Thestimuluswillcontainasetofstatements,andthosestatementswillbeusedto"stronglysupport"thecorrectanswer,makingthisanInferencequestion.

Step2:UntangletheStimulus

Theanalystprovidesfourrequirementsforthelocationofaparticulargenerationstation:1)Itneedstobenearanatural-gaspipeline;2)itneedstobenearalargebodyofwater;3)itneedstobeneartransmission

Page 90: LSAT PrepTest 81 Unlocked · Similarly, PT 81 was the first test since June '07 (PT June '07) to require at least 94 questions correct to get a 172 (99th percentile score). Number

lines;and4)residentswon'topposeit.Asofnow,theanalyst'scountryhasextensivetransmissionlines,sothethirdrequirementshouldbefine.Theproblemisthereareonlythreelargebodiesofwaterneargaspipelines,butresidentswouldopposeallthreelocations.

Step3:MakeaPrediction

Withtheresidentsbeingsostubborn(theauthorsaystheywouldopposeanyconstructionprojectnearthethreebodiesofwater),thecountryisatanimpasse.Theonlychoiceleftistofindanotherbodyofwater,butnoneofthemarecurrentlynearnatural-gaspipelines.Thatleavesoneviablealternativeifthecountrywantstobuildoneofthesestations:buildnewpipelinesnearanotherlargebodyofwater.

Step4:EvaluatetheAnswerChoices

(C)isthesupportedinferencefromtheanalyst'sinformation.Thecurrentsitesdon'tmeettherequirements,andbuildingnewpipelinesistheonlylogicalcourseofactioniftheywanttobuildthistypeofstation.

(A)isnotsupported.Newpipelinescanstillbebuilt,sothere'snoneedtogiveuponnatural-gas-poweredgenerationjustyet.Furthermore,perhapsexistingnatural-gas-poweredgenerationstationswouldbesufficienttomeetfutureelectricalneeds—it'sonlytheconstructionofnewonesthatarecitedasapotentialproblem.

(B)isnotsupported.Theresidentsdoopposethecurrentlyavailablesites.Ifthestationisbuiltanyway,theresidentsmightprotest,butthere'snoindicationthey'djustpackupandmove.

(D)isnotsupported.It'spossiblethatsuchstationswerealreadybuiltinthepast.Theresidentsjustwon'tapproveofanynewprojects.Theold

Page 91: LSAT PrepTest 81 Unlocked · Similarly, PT 81 was the first test since June '07 (PT June '07) to require at least 94 questions correct to get a 172 (99th percentile score). Number

stationscouldhavebeenbuiltbeforethepublicstateditsviews.

(E)isa180.Theanalystclaimsresidentswouldopposeanysignificantconstructionprojectinthoseareas,notjustelectricalstations.

20. (A)FlawStep1:IdentifytheQuestionType

Thequestionaskswhythe"reasoningisquestionable"intheargumentgiven,makingthisaFlawquestion.

Step2:UntangletheStimulus

Theauthorconcludesthateachgenerationofcitizensisbecomingmoredisinterestedinpoliticsthanthenextgeneration.Theevidenceisthatpeopleover65votealotwhileyoungadultsdon't.

Step3:MakeaPrediction

Therearesomerepresentativenessissueswiththeauthor'sargument;thisisnotanapplestoapplescomparison.Whatistrueofagenerationinthelaterstagesoftheirlivesmaynothavebeentrueaboutthemintheirearlieryears.Perhapstheyoungergenerationwillbehavemoreliketheoldergenerationwhentheyreachthatpointoftheirlives.Likewise,theoldergeneration'scurrentvotingrecordmaynotberepresentativeoftheirvotingrecordwhentheywereyoungadults.

Step4:EvaluatetheAnswerChoices

(A)iscorrect.Onegenerationisatanearlystageoftheirlivesandtheotheratalaterstage.Itisunknownhowtheoldergenerationbehavedwhentheywereyounger,anditisunknownhowtheyoungergenerationwillbehavewhentheyareolder.So,theauthormakesafaultypredictionthattheyoungadultscurrentbehaviorisindicativeofwhattheirfuturebehaviorwillbe.

Page 92: LSAT PrepTest 81 Unlocked · Similarly, PT 81 was the first test since June '07 (PT June '07) to require at least 94 questions correct to get a 172 (99th percentile score). Number

(B)isnotanissue.Theargumentisaboutpercentagesandrates,soactualnumbersarenotrelevant.

(C)isaccurateinthattheauthordoesnotexplainwhypeoplearebecomingdisconnectedfrompolitics.However,that'snotthepurposeoftheargument,sothere'snothingflawed(i.e.,questionable)aboutomittingthat.

(D)isOutofScope.Theauthorneveraddressesthecauseoftheproblem,sothere'snothingtoconfuse.

(E)isa180.Thepointoftheauthor'sargumentisthatvotingpatternsarechanging,andthatfuturepatternsarelikelytoshowevenmoreofadisconnect.However,theauthorfailstoconsiderwhethertheoldergenerationhasalwayshadahighpercentageofvoters,orwhetherthey'vematuredintothatbehavior.Ifthey'vematured,thenthecurrentyoungergenerationmaydothesame.Thus,theauthoroverlooksthepossibilitythatthevotingpatternsamongagegroupsarenotchanging—it'sjustthatolderpeoplemayalwaysbemorelikelytovote.

21. (B)Principle(Parallel)Step1:IdentifytheQuestionType

Accordingtothequestion,thereisaprincipletobeidentifiedfromtheargumentgiven.However,thecorrectanswerwillnotdescribethatprinciple.Instead,itwillre-applythatprincipletoanewsituation.ThatmakesthisarelativelyuncommonParallelPrinciplequestion.

Step2:UntangletheStimulus

Page 93: LSAT PrepTest 81 Unlocked · Similarly, PT 81 was the first test since June '07 (PT June '07) to require at least 94 questions correct to get a 172 (99th percentile score). Number

Theauthorconcludes(therefore)thatthecityshouldnotallowtheofficecomplextobebuiltjustyet.Theevidenceisthatbuildingitwouldrequiredrainingalocalmarsh,andthatraisespotentialproblemsthathaveyettobeassessed.

Step3:MakeaPrediction

Inprinciple,theauthorisadvocatingnottotakeanyrashactionswhentherearepotentialproblemsthatshouldbestudiedfirst.Thecorrectanswerwillapplythisprincipletoanothersituation.

Step4:EvaluatetheAnswerChoices

(B)matchestheprinciple.Liketheoriginalargument,therearepotentialproblems(recallsandlawsuitsduetodefects)thathavenotbeenstudiedyet.Basedonthat,it'srecommendednottotakeactionandsellthenewproductjustyet.

(A)doesnotmatch.Thisoutrightrejectstakingactionbecauseofthehighcostofperformingtheneededassessment.That'snotthesameastemporarilyholdingbackuntiltheassessmentisdone.

(C)doesnotmatch.Inthissituation,thesuggestionistonotrevealtheresultsoftheassessmentjustyet.That'snotthesameasaskingthecompanytowaitbeforesellingthegrills.Infact,it'spossiblethecompanyhasalreadystartedsellinggrillsandsentsomeinfortestingafterthefact,whichwouldgocontrarytotheoriginalauthor'sprinciple.

(D)isaDistortion.Thistriestomimictheoriginalargument'sconcernfortheenvironment.However,theguidingprincipleoftheoriginalargumentwasnot"dowhat'slessdamaging."Itwasallaboutassessingtheproblembeforeacting,andthisargumentleavesthatoutentirely.

Page 94: LSAT PrepTest 81 Unlocked · Similarly, PT 81 was the first test since June '07 (PT June '07) to require at least 94 questions correct to get a 172 (99th percentile score). Number

(E)doesnotmatch.Thissimplymakesajudgmentthatsolvingfutureproblemsoverridesthecostsinvolved.Thisdoesnotcomparetotheoriginalargument,whichwasbasedonassessingproblemsfirst.Further,thisargumentrecommendstakingacourseofactionwhiletheoriginalrecommendedtemporarilyholdingoff.

22. (D)WeakenStep1:IdentifytheQuestionType

Thequestiondirectlyasksforsomethingthat"mostweakens"thegivenargument,makingthisaWeakenquestion.

Step2:UntangletheStimulus

Theauthordescribesastudywithtwogroupsofpeople.Thefirstgroupwatchedrecordingsofthemselvesonatreadmill.(Thatmusthavebeenexciting.)Thesecondgroupwatchedrecordingsofotherpeopleonatreadmill.(Evenmoreexciting.)Whenlateraskedhowmuchtheyexercise,peopleinthefirstgroupreportedanaverageofonehourlonger.Basedonthat,theauthorconcludesthatwatchingyourselfexercisingcanmotivateyoutoexercisemore.

Step3:MakeaPrediction

Thereisalotwrongwiththisentiresituation,butlet'ssticktothepoorlogicoftheargument.Firstoff,thisisaclassiccaseofCorrelationvs.Causation.Theauthormerelyassumesthatthevideosmotivatedpeople,andnothingelse.What'smore,there'salwaysafundamentalerrorwhentheauthorsayssomethinghappenedmoreoftenbecausepeopleinastudysaidtheydiditmoreoften.Inthiscase,thepeopleinthefirstgroupsaidtheydidmoreexercise.Butdidtheyreally?Whoknows?Maybetheyjustwatchedthevideoandthought,"Wow—IbetIexercisemorethanIthought.I'lljusttelltheseresearchfolksthatIexercisealot."Ifthe

Page 95: LSAT PrepTest 81 Unlocked · Similarly, PT 81 was the first test since June '07 (PT June '07) to require at least 94 questions correct to get a 172 (99th percentile score). Number

correctanswerdoesn'tshowanalternativereasonwhypeopleexercisemore,itwilllikelyshowthatpeoplearen'tactuallyexercisingmore;they'rejustmakingupnumbers.

Step4:EvaluatetheAnswerChoices

(D)weakenstheargument,albeitinaveryoffbeatway.Inthestudydescribedinthischoice,peoplewatchedvideosoftheiridenticaltwinreading.Inotherwords,theywerewatchingpeoplewholookedjustlikethem.Afterdoingso,theyoverreportedhowmuchtimetheyspentreading.So,theydidn'tactuallyreadmore,theyjustsaidtheydid.Thatsuggeststhesamemightbehappeningwiththetreadmillstudy,andthusitislesslikelythatpeopleareactuallymotivatedtoexercisemore.Isthatthebestwaytoweakenthisargument?Notbyalongshot.However,it'stheonlychoicethataddresseseithertheassumptionthattherewerenoalternativecausesoftheincreasedexerciseortheassumptionthatthepeople'sself-assessmentsofmoreexercisewereaccurate.

(A)isanIrrelevantComparison.Theauthorisnotconcernedwithfindingthemosteffectivemotivator.Ifpeopleexercisedmoreafterwatchingthemselvesonatreadmill,theauthor'sargumentisstillvalid,evenifwatchingthemselvesliftweightswouldhavebeenmoreeffective.

(B)isOutofScope.Thisinvolveshearingsecond-handstoriesaboutotherpeople.Evenifthateffectivelymotivatespeople,itdoesn'tweakentheideathatwatchingyourselfexerciseonvideocouldalsobemotivating.

(C)isirrelevant.Howmanysuchparticipantswerethere?Didtheymakeupalargeportionofthestudy?Andwhatgroupweretheyin?Without

Page 96: LSAT PrepTest 81 Unlocked · Similarly, PT 81 was the first test since June '07 (PT June '07) to require at least 94 questions correct to get a 172 (99th percentile score). Number

moredetails,afewstrayhealthnutsarenotgoingtohaveanyeffectontheauthor'sclaim.

(E)isa180atworst.Thissuggeststhatpeopleareactuallyinfluencedbywatchingthemselvesonvideo.Watchyourselfonatreadmill?Youexercisemore!Watchyourselfsittingonacouch?(Themostexcitingvideoyet!)Yousitaroundmore!

23. (A)Assumption(Necessary)Step1:IdentifytheQuestionType

Thequestionasksforsomethingtheargument"requiresassuming,"makingthisaNecessaryAssumptionquestion.

Step2:UntangletheStimulus

Theenvironmentalistisarguingthatconvincingpeopletoreducetheirpersonaluseoffossilfuelsisnotgoingtoreducecarbonusageoverall.Theevidenceisthatreducingcarbonusagerequireslarge-scalegovernmentpolicies.

Step3:MakeaPrediction

There'samajorOverlookedPossibilityhere.Theenvironmentalistisonlylookingatdirecteffects.Individualchangeswon'tdirectlyreducecarbonusageenough.However,it'spossiblethatindividualchangescouldindirectlyleadtobiggerchanges,whichmayultimatelyleadtothecentralrequirement:governmentpolicies.Theenvironmentalistassumesthiswouldn'thappenandthatweneedtostartdirectlyatgovernmentpolicies.

Step4:EvaluatetheAnswerChoices

(A)mustbeassumed.Afterall,usingtheDenialTest,ifpersonalchangesdidpersuadepeopletogetthegovernmentinvolved,thenthe

Page 97: LSAT PrepTest 81 Unlocked · Similarly, PT 81 was the first test since June '07 (PT June '07) to require at least 94 questions correct to get a 172 (99th percentile score). Number

environmentalist'sargumentisunsound.Focusingonindividualeffortscouldpayoffinthelongrun.Theenvironmentalistmustbeassumingthatwon'thappen.

(B)isOutofScope.Thedifficultyinperformingsuchcalculationsisirrelevanttotheargument.Theenvironmentalists'spointisthatpeople'seffortswouldn'tbeenough,eveniftheydidgothroughtheeffortofdeterminingthebestcourseofaction.

(C)isExtreme.Thepeopleencouragingpersonalreductioninfossilfuelusagedon'thavetobecurrentlyuninvolvedinframinggovernmentpolicies.Theycouldallbeinvolvedingovernment,butstillfail,astheenvironmentalistsuggests,byfocusingonindividualsinsteadofgettingthegovernmentinvolved.

(D)isanIrrelevantComparison.Theargumentdoesnotdependononecourseofactionbeingeasier.Itdependsononebeingmoreeffective.

(E)isOutofScope.Itdoesn'tmatterwhichcandidatespeoplesupport.Allthatmattersiswhetherornottheneededgovernmentalpoliciescanbeenacted.

24. (C)ParallelFlawStep1:IdentifytheQuestionType

Thecorrectanswerwillhaveanargumentthatis"similartothat"inthestimulus.Moreover,thatreasoningisdescribedasquestionable,makingthisaParallelFlawquestion.

Step2:UntangletheStimulus

Page 98: LSAT PrepTest 81 Unlocked · Similarly, PT 81 was the first test since June '07 (PT June '07) to require at least 94 questions correct to get a 172 (99th percentile score). Number

Theauthorpresentstwopossiblesourcesofapainting'saestheticvalue:thepainting'sformalqualitiesoritsmeaning.Theauthorthenarguesthatthere'snovalidsupportforsayingit'sintheformalqualities,soitmustbeinthepainting'smeaning.

Step3:MakeaPrediction

Theremaynotbesupportforformalqualities,butwhosaidthere'sanysupportforthepainting'smeaning?Theauthormerelyrejectsoneoptionwithoutprovidingconvincingevidenceinfavorofthesecond.Thecorrectanswerwillfollowthesameflawedformat:presenttwooptions,rejectonefornothavingsupportiveevidence,andillogicallyclaimthesecondoptioniscorrect.

Step4:EvaluatetheAnswerChoices

(C)isamatch.Theauthorpresentstwooptions(economicorpoliticalforces),rejectsonefornothavingsupportiveevidence(economicforces),andillogicallyclaimsthesecondoptioniscorrect.

(A)doesnotmatch.Theauthordoesrejectoneoftwooptions.However,theoriginalauthorclaimedtherewereonlytwoviableoptions.Thisauthorclaimsthattherearemultiple"othermethods."So,thisargumentisflawedinthatitfailstoconsidertheotheroptions,butthat'snotthesameastheoriginalargument.Furthermore,thisauthordoesgiveaviablereasontorejectoneofthetwooptionsspecificallymentioned.

(B)doesnotmatch.Thetwooptionspresentedarerequirementsifanoutcomeoccurs(thecompanybeingoutbid).Theauthorclaimsoneoptionwon'thappen,sotheoutcomewon'toccur.However,thiscompletelyignoresthesecondrequirement.Andtheoriginalargument

Page 99: LSAT PrepTest 81 Unlocked · Similarly, PT 81 was the first test since June '07 (PT June '07) to require at least 94 questions correct to get a 172 (99th percentile score). Number

wasnotbasedonanynecessaryconditions.TheFormalLogicisflawed,forsure.However,it'snotthesameflawastheoriginal.

(D)doesnotmatch.Theauthorpresentstwooutcomesifasituationoccurs.Theauthorthenconcludesthatoneoutcomewon'thappenbecausetheotherwon'thappen.ThisdisplayssomepoorFormalLogic,forsure,butit'snotthesameasrejectingoneoptionandsayingtheotheroptionmustbecorrect.

(E)doesnotmatch.Iftherearetwooptionstoconsiderhere,it'swhetherthepartychangesitspoliciesornot.However,theconclusiondoesn'trejectoneandfavortheother.Itjustsayssomethingbadwillinevitablyhappen.

25. (D)FlawStep1:IdentifytheQuestionType

Thequestiondirectlyasksforadescriptionoftheargument'sflaw.

Step2:UntangletheStimulus

Thephrase"mustbe"indicatessomeFormalLogic.Ifthereistobeeconomicgrowth,thentheremustbetechnologicalinnovations.Theauthorthenclaimsthatabanonfossilfuelswillspurtechnologicalinnovations,andusesthattoconcludethateconomicgrowthisimminent.

Step3:MakeaPrediction

WhenFormalLogicappearsinaFlawquestion,there'sagoodchancetheauthorisgoingtocommitthecommonlytestedflawofNecessityvs.Sufficiency.Sureenough,theFormalLogicdictatesthattechnologicalinnovationsmustbeinplacefirst(i.e.,they'renecessary).

Page 100: LSAT PrepTest 81 Unlocked · Similarly, PT 81 was the first test since June '07 (PT June '07) to require at least 94 questions correct to get a 172 (99th percentile score). Number

Ifsubstantialeconomicgrowth→techinnovations

However,thatdoesn'tmeantechinnovationswillguarantee(i.e.,aresufficientfor)economicgrowth,astheauthorasserts.

Iftechinnovations→substantialeconomicgrowth

Theauthorreversed,butfailedtonegate.Thecorrectanswerwilldescribethismistakentreatmentofanecessaryconditionasifitweresufficient.

Step4:EvaluatetheAnswerChoices

(D)iscorrect.

(A)describestheflawofCircularReasoning,butthatdoesn'thappenhere.Theauthormisinterpretstheevidence.It'snotjustaboutamererestatementoftheevidence.

(B)describestheflawofadhominem,whichinvolvesattackingpeoplepersonallyratherthanaddressingtheirclaims.However,theauthordoesnotpersonallyattackthecriticsoftheban.Theauthortriestoaddressestheirclaim,butfailstofollowtherulesofFormalLogic.

(C)isnotevenaflaw.Thisissuggestingthattheauthor'sevidenceistoogood.When'sthelasttimeyoutoldsomeone,"Idon'tbelieveyou—yourevidenceisjusttooconvincing"?

(E)isaDistortion.Thissuggeststheauthorconcludesthatinnovationalwaysbringsabouteconomicgrowthjustbecauseinnovationsometimeshappensbeforeeconomicgrowth.Thatwouldbeflawedlogic,butit'snot

Page 101: LSAT PrepTest 81 Unlocked · Similarly, PT 81 was the first test since June '07 (PT June '07) to require at least 94 questions correct to get a 172 (99th percentile score). Number

whattheauthordoeshere.Theauthorsayssubstantialeconomicgrowthmustbeprecededbyinnovation,so,there'snosometimesabouttheauthor'sevidence.

26. (E)PointatIssueStep1:IdentifytheQuestionType

AswithmostPointatIssuequestions,therearetwospeakers,andthecorrectanswerwilladdresssomethingaboutwhichtheybothhaveanopinion.However,readthequestioncarefully.UnlikemostPointatIssuequestions,thecorrectanswerwillbesomethingthespeakersagreewitheachotherabout.

Step2:UntangletheStimulus

WinstonisunhappywiththerulesforawardingNobelPrizes.Eachawardcangotoonlythreepeople,butmanywinningscienceresultsaretheworkoffourormorepeople.Sanjayisalsounhappy,butwithanotherrestriction:Winnershavetobeliving.Thatignoresinfluentialscientistswhodiedbeforetheirresultswererecognized.

Step3:MakeaPrediction

WinstonandSanjayarebothunhappywiththerulesforawardingNobelPrizes,particularlyinscience.Inbotharguments,theruleshavetheeffectofdenyingcredittocertainscientists(thosewhowereleftoutafterthefirstthreepeopleonaprojectwereselected,andthosewhodiedbeforetheprojectwontheaward).Thecorrectanswerwilladdressthisagreed-upondispleasurewithpeoplegettingignored.

Step4:EvaluatetheAnswerChoices

(E)iscorrect.ToWinston,theprizesareinaccuratebecausetheyonlylistthreepeopleandleaveoutotherpotentialcontributors.ToSanjay,the

Page 102: LSAT PrepTest 81 Unlocked · Similarly, PT 81 was the first test since June '07 (PT June '07) to require at least 94 questions correct to get a 172 (99th percentile score). Number

prizesareinaccuratebecausetheydon'trecognizecontributorswhomayhavedied.

(A)onlyaddressesSanjay'sconcerns.It'spossiblethatWinstonwouldagree,butthere'snothinginhisstatementstodirectlysuggestthat.

(B)isaDistortion.It'slikelythatbothauthorswanttoseesomechangestothesciencerules,butthatdoesn'tmeansciencehastohaveitsownuniquerules.Whilebothauthorsonlytalkaboutsciencehere,theymayhavesimilarcomplaintsaboutotherdisciplinesaswell.Perhapstheywouldbothliketoseeuniversalchangessothattherulesareconsistent,butmoreinclusive,foralldisciplines.

(C)isaDistortion.Neitherauthorarguesagainsttheawardingofprizestoparticularresults.Theirconcernsareaboutthepeoplebeingrecognized.

(D)isOutofScope.Neitherauthoraddresseswhethertheawardsarebasedonsubjectiveorobjectivecriteria.

Page 103: LSAT PrepTest 81 Unlocked · Similarly, PT 81 was the first test since June '07 (PT June '07) to require at least 94 questions correct to get a 172 (99th percentile score). Number

LSATPrepTest81

Page 104: LSAT PrepTest 81 Unlocked · Similarly, PT 81 was the first test since June '07 (PT June '07) to require at least 94 questions correct to get a 172 (99th percentile score). Number

SECTIONIII:LOGICALREASONING

Q# QuestionType Correct Difficulty

1 PointatIssue E Checkyouronlineresources.

2 Paradox C Checkyouronlineresources.

3 MainPoint A Checkyouronlineresources.

4 Assumption(Necessary) E Checkyouronlineresources.

5 Inference D Checkyouronlineresources.

6 Paradox E Checkyouronlineresources.

7 Flaw B Checkyouronlineresources.

8 MainPoint B Checkyouronlineresources.

9 Flaw B Checkyouronlineresources.

10 Inference B Checkyouronlineresources.

11 ParallelFlaw A Checkyouronlineresources.

12 Strengthen A Checkyouronlineresources.

13 Flaw B Checkyouronlineresources.

14 Inference A Checkyouronlineresources.

15 Flaw A Checkyouronlineresources.

16 Weaken C Checkyouronlineresources.

17 RoleofaStatement D Checkyouronlineresources.

18 Assumption(Necessary) D Checkyouronlineresources.

Page 105: LSAT PrepTest 81 Unlocked · Similarly, PT 81 was the first test since June '07 (PT June '07) to require at least 94 questions correct to get a 172 (99th percentile score). Number

19 Inference C Checkyouronlineresources.

20 Assumption(Sufficient) E Checkyouronlineresources.

21 MethodofArgument C Checkyouronlineresources.

22 Weaken D Checkyouronlineresources.

23 Flaw B Checkyouronlineresources.

24 Inference B Checkyouronlineresources.

25 ParallelReasoning E Checkyouronlineresources.

1. (E)PointatIssueStep1:IdentifytheQuestionType

Thequestionasksforsomethingthattwospeakers"disagreeover,"makingthisaPointatIssuequestion.

Step2:UntangletheStimulus

Joefindsvampirestoriesabsurd,arguingthattheseimmortalcreaturesshouldhavealmostnopreyremainingasthey'vebeenaroundforagesandalloftheirvictimsturnintovampires,too.MariapointsoutaflawinJoe'sanalysis.Instoriesshe'sread,vampiresonlyturnsomeotherpeopleintovampires,notalloftheirvictims.

Step3:MakeaPrediction

Thepointatissuehereiswhether,invampirestories,vampiresturneveryoneintovampiresorjustaselectfew.

Step4:EvaluatetheAnswerChoices

(E)iscorrect.Joeclaimsthisiscorrect,whileMariaarguesotherwise—insomestories,mostvictimsaremerelykilled,notturnedintomore

Page 106: LSAT PrepTest 81 Unlocked · Similarly, PT 81 was the first test since June '07 (PT June '07) to require at least 94 questions correct to get a 172 (99th percentile score). Number

vampires.

(A)isOutofScopeforMaria.Joementionsvampires'immortality,butMariamakesnomentionofit,nordoessheseemtodisputethatclaim.

(B)isOutofScopeforMaria.Joementionsvampiresexistingsinceancienttimes,butMarianeitheraddressesnordisputesthatclaim.

(C)isaDistortionofJoe'sclaims.Joefindsthestoriesabsurdinthattheyhaveridiculousconsequences.However,thatdoesn'tmeanthey'reincoherent(i.e.,confusingorunclear).

(D)isOutofScopeforbothspeakers.Joearguesthatthepremisesofsuchstoriesimplythatalmosteveryoneshouldbeavampirebynow.However,heneverclaimsthestoriesactuallydepictthislarge-scalevampirepopulation.AndMarianeveraddresseshowlargethevampirepopulationissaidtobe.

2. (C)ParadoxStep1:IdentifytheQuestionType

Thecorrectanswerwillhelp"accountfor"thesituationpresented,makingthisaParadoxquestion.

Step2:UntangletheStimulus

Acompanywantedtohelpitssalespeoplebyscanningalloftheirpaperworkandstoringitinadatabasethatcanbeeasilyaccessedbycomputer.Theyexpectedthesalespeoplewouldbethrilled.Nomorecarryingaroundpilesofpapers!However,theresultwasnotasexpected.Salespeopleresistedthedatabaseandrefusedtogettheirpaperworkscanned.

Page 107: LSAT PrepTest 81 Unlocked · Similarly, PT 81 was the first test since June '07 (PT June '07) to require at least 94 questions correct to get a 172 (99th percentile score). Number

Step3:MakeaPrediction

Itdefinitelyseemsstrangethatthesalespeoplewerenotinterestedinsomethingthatseemssohelpful.Whydidtheyresistthischange?Themostlikelyexplanationsarethattherewassomethingremarkablyinconvenientaboutthedatabase,orthereissomethinghighlybeneficialaboutkeepingworkinpaperform.Thecorrectanswerwilladdressone,ifnotboth,ofthesepossibleexplanations.

Step4:EvaluatetheAnswerChoices

(C)iscorrect.Thispointstoabenefitofpaperforms—clientconfidentiality.Thatexplainswhysalespeopleresistedthedatabaseanddidn'twanttosubmittheirpaperworkforscanning.

(A)doesnothelp.Thisconfirmsthatsomesalespeopledidn'tsubmitalotofpaperwork.However,itdoesn'tofferareasonwhy,sothere'sstillnoaccountingfortheirresistance.

(B)isa180.Ifthesalespeopledidn'thaveportablecomputers,thatmightexplainwhytheyresistedthedatabase.However,iftheyalreadyhadportablecomputers,it'sevenmoreunusualthattheywouldn'ttakeadvantageofthedatabase.

(D)isa180,atworst.Ifthetrainingwasinconvenientandthedatabasewasoverlycomplicated,thatmightexplaintheresistance.However,ifthesalespeoplefoundthedatabasesoftwaresoeasytouse,it'sevenhardertounderstandwhytheywouldn'tuseit.

(E)isOutofScope.Theparadoxhasnothingtodowiththebuildingofthedatabase.Theparadoxisallaboutwhyemployeesdidn'tusethedatabase,nomatterhowmuchtimeormoneyitcosttobuild.

Page 108: LSAT PrepTest 81 Unlocked · Similarly, PT 81 was the first test since June '07 (PT June '07) to require at least 94 questions correct to get a 172 (99th percentile score). Number

3. (A)MainPointStep1:IdentifytheQuestionType

Thequestionasksyouwhat"thepoliticianargues,"whichmeansthecorrectanswershouldexpressthepointthepoliticianisadvocating,i.e.,themainpoint.

Step2:UntangletheStimulus

Thepoliticianismakingacommonclaimthat"freespeech"doesn'timplyeverythingyousayisprotected.Whatfollowsaresomecommonexamplesofunacceptableformsofspeech,whichallleadtotheultimateconclusion:Someformsofspeechcanleaddirectlytoharmandarethusokaytomakeillegal.

Step3:MakeaPrediction

Thecorrectanswerwillexpressthepolitician'sclaimattheendthatcriminalizationofsomespeechisokaybecausethatspeechcancauseharm.

Step4:EvaluatetheAnswerChoices

(A)iscorrect.

(B)isExtreme.Thepoliticianusesexamplesofspeechthatcancauseharmandconcludesthatsuchspeechcanberestricted.However,theauthordoesn'tclaimthisistheonlykindofspeechthatcanberestricted.Theremaybeotherreasonstorestrictotherkindsofspeech.

(C)isaDistortion.Theonlyharmmentionedbythepoliticianisthatcausedbycertainformsofspeech.Theauthorneversaysanythingaboutharmbeingcausedbyrestrictingspeech.

(D)isExtreme.Thepoliticianarguesthatcertainformsofspeechcanlead

Page 109: LSAT PrepTest 81 Unlocked · Similarly, PT 81 was the first test since June '07 (PT June '07) to require at least 94 questions correct to get a 172 (99th percentile score). Number

directlytoharm,butneversaysthatanyformofspeechcandoso.

(E)isOutofScope.Theauthornevermentionsanysituationinwhichrestrictingfreedomisunjustified.

4. (E)Assumption(Necessary)Step1:IdentifytheQuestionType

Thequestiondirectlyasksforanassumptiononwhichtheargumentdepends,makingthisaNecessaryAssumptionquestion.

Step2:UntangletheStimulus

Accordingtotheartcritic,peoplewhogotomuseumslookatanartworkforunderaminute,takeaphoto,andmoveon.Thatleadsthecritictoconcludethatpeoplearelesswillingtoengagewithartwork.

Step3:MakeaPrediction

ThisisaperfectexampleofMismatchedConcepts.Theevidenceisallaboutthebrieftimespentlookingatartwork,buttheconclusionraisestheconceptofbeingengaged.Theartcriticisassumingthere'saconnectionbetweenthosetwoconcepts,i.e.,thattimespentlookingatanartworksomehowindicateshowengagedoneiswiththatartwork.

Step4:EvaluatetheAnswerChoices

(E)iscorrect.BytheDenialTest,iftimespentwasnotareliablemeasureofengagement,thenthecritic'sconclusioniscompletelyunsupported.Theartcriticmustassumethattimespenthassomeconnectiontoengagement.

(A)isOutofScope.Itdoesn'tmatterwhetherpeopleseeonepieceofartor100.Theargumentisaboutwhetherpeopleareengagedwiththatart,andthismakesnoconnectiontothat.

Page 110: LSAT PrepTest 81 Unlocked · Similarly, PT 81 was the first test since June '07 (PT June '07) to require at least 94 questions correct to get a 172 (99th percentile score). Number

(B)isirrelevant.Whypeoplemovesoquicklydoesn'tmatter.Whatmattersiswhetherornotpeoplearelosingtheirwillingnesstoengageintheart.

(C)isOutofScope.Theargumentisnotaboutenjoyingthemuseum-goingexperience.It'saboutengagingwiththeartwork,whichisnotnecessarilythesameconcept.

(D)isOutofScope.Thismaystrengthentheartcritic'sevidencethatpeopledon'tspendmuchtimewithasinglepieceofart—theydon'tevenlookatthephotooftheart!However,regardlessofthetimespentwiththeartworkoritsphoto,thisoffersnoconnectiontowhetherornotpeoplefeelengagedwiththeartwork.

5. (D)InferenceStep1:IdentifytheQuestionType

Thecorrectanswerwillbe"supportedbytheinformation"given,makingthisanInferencequestion.

Step2:UntangletheStimulus

Accordingtotheauthor,heavytapestryfabricsshouldn'tbeusedtocreateitemsthatneedtobefrequentlylaundered,suchasclothing.Instead,itshouldbeusedforitemssuchaswindowtreatments.

Step3:MakeaPrediction

ThewordonlyindicatesFormalLogic:Ifit'sappropriatetouseheavyfabric,thentheitemshouldnotbefrequentlylaundered.Bycontrapositive,ifanitemisgoingtobefrequentlylaundered,thenit'snotappropriatetouseheavytapestryfabrics.

Page 111: LSAT PrepTest 81 Unlocked · Similarly, PT 81 was the first test since June '07 (PT June '07) to require at least 94 questions correct to get a 172 (99th percentile score). Number

Ifheavyfabricappropriate→~launderedfrequently

Iflaunderedfrequently→~heavyfabricappropriate

Thecorrectanswerwillfollowthislogicwithoutimproperlynegatingorreversingthelogic.

Step4:EvaluatetheAnswerChoices

(D)iscorrect,essentiallyusingthecontrapositive.Becauseskirtsandjacketsaresaidtobefrequentlylaunderedclothing,thenheavyfabricwouldnotbeappropriate.

(A)isaDistortion.Heavyfabricsareappropriateforswags,buttherecouldbeotherfabricsnotmentionedthatarealsoappropriate.

(B)isExtreme.Theauthorsaysthatappropriateapplicationsincludeswagsandballoonvalances.However,therearelikelyplentyofotheracceptableapplicationsforheavyfabrics.

(C)isalsoExtreme.Appropriateapplicationsincludethewindowtreatmentslisted,butthatdoesn'tmeanallappropriateapplicationsmustbewindowtreatments.

(E)isa180.Theauthorspecificallymakesnoteofskirtsandjackets,butthegeneralclaimisthatheavytapestryfabricsarenotappropriatefor"anytypesofclothing."

6. (E)ParadoxStep1:IdentifytheQuestionType

Page 112: LSAT PrepTest 81 Unlocked · Similarly, PT 81 was the first test since June '07 (PT June '07) to require at least 94 questions correct to get a 172 (99th percentile score). Number

Thequestionasksforsomethingthat"helpstoexplain"adiscrepancy,makingthisaParadoxquestion.

Step2:UntangletheStimulus

NewapartmentsinBrewstervillelogicallyincreasedthesupplyofavailablehousing.However,whilethatusuallyleadstolowerrentsforexistingapartments,theoppositehappened:thoserentswentup.

Step3:MakeaPrediction

Whydidrentsforexistingapartmentsgoupwhentheyusuallygodown?Thecorrectanswerwillanswerthatquestion.Thecorrectanswerwilllikelyshowwhyeventheexistingapartments(insteadofjustthenewones)aresuddenlymoredesirable.

Step4:EvaluatetheAnswerChoices

(E)solvesthemystery.Ifthepopulationstayedthesame,thenthegeneraltrendwouldbeexpected:highpricesforthenewapartmentsandlowerpricesfortheoldones.However,iflotsofpeoplearelookingtomoveintothearea,thentherewouldbeincreaseddemandforboththenewandtheexistingapartments,whichwouldlogicallyleadtothehigherprices.

(A)doesnothelp.Evenifthereweresupposedtobemorenewapartments,there'snoindicationwhytheolderapartmentsaremoredesirableandworthmorerent.

(B)isa180.Ifthenewapartmentsaremoredesirable,thentheyshouldhavehigherrents.Itwouldn'tmakesensetoraisetherentfortheoldapartments,whicharelessdesirable.

Page 113: LSAT PrepTest 81 Unlocked · Similarly, PT 81 was the first test since June '07 (PT June '07) to require at least 94 questions correct to get a 172 (99th percentile score). Number

(C)isOutofScope.TheeffectinotherareashasnoimpactonexplainingtherentincreaseonapartmentsinBrewsterville.

(D)doesnothelp.Thisjustsuggeststhatthereweremoreolderapartmentsavailableaspeoplemovedout.However,withmoreapartmentsavailable,thatdoesn'texplainwhytherentwouldincrease.

7. (B)FlawStep1:IdentifytheQuestionType

Thecorrectanswerwilldescribewhytheargumentis"vulnerabletocriticism,"afrequentlyusedphrasethatindicatesaFlawquestion.

Step2:UntangletheStimulus

Theauthorarguesthatpoliticianspushformoreeconomicproductivitybutignorethenegativeconsequences.Theauthorthenprovidesanexampleofhowacompanycouldattempttoincreaseproductivitybyincreasingprofits,butthatoftenleadstoreducingemployment.Thus,theauthorconcludesthattryingtoincreaseeconomicproductivitywouldleadtounemployment.

Step3:MakeaPrediction

Theauthorprovidesagreatexampleofhowfocusingonproductivitycanhaveundesirableconsequences.However,theauthorthenusesthedetailsofthatoneexample(aboutacorporationlosingemployees)andsuggeststheexactsameconsequenceswillhappenifpoliticiansfocusonincreasingeconomicproductivityasawhole.Whileit'spossiblethatfocusingtoomuchonproductivitycouldbeproblematic,there'snoreasontosuggestthattheresultwoulddefinitelybeunemploymentbasedononehypotheticalexample.Thisisacommonflawofbasingabroadconclusiononanunrepresentativesample.

Page 114: LSAT PrepTest 81 Unlocked · Similarly, PT 81 was the first test since June '07 (PT June '07) to require at least 94 questions correct to get a 172 (99th percentile score). Number

Step4:EvaluatetheAnswerChoices

(B)correctlydescribesthecommonlytestedflaw.Theauthorassumesthatwhathappensinonesinglecaseisgoingtohappenwhenaddressingtheeconomyoverall.

(A)isOutofScope.Theauthoriscertainlyconcernedaboutpotentialdrawbacks,particularlyunemployment.However,theauthorneverarguesthatthegoalofincreasingproductivityshouldbeabandoned.Perhapstheauthorjustfeelsthatpoliticiansshouldexercisemorecaution.

(C)isaDistortion.Theauthordoescriticizepoliticiansingeneral,buttheevidenceisthattheydo,ingeneral,failtoconsiderthedrawbacks.It'snotsaidtobejustafewpoliticiansthatmakethismistake.

(D)isaDistortion.Theauthormakesnocomparisonastowhoseinterestsaremoreimportant.Besides,theauthorclaimsthatincreasingproductivitywouldbebeneficialtobusinessowners,sothere'snoassumptionthatproductivityismoreimportantthantheowners'interests.

(E)isaDistortion.Theauthor'sargumentisjustthattherecanbedrawbacks,notthatthedrawbacksoutweighthebenefits.Toclaimthattherearedrawbacks,theauthormerelyneedstoshowtheyexist.There'snoneedtomentionalldrawbacksoranybenefits.

8. (B)MainPointStep1:IdentifytheQuestionType

Thequestionasksforthe"overallconclusion,"i.e.,themainpointoftheargument.

Page 115: LSAT PrepTest 81 Unlocked · Similarly, PT 81 was the first test since June '07 (PT June '07) to require at least 94 questions correct to get a 172 (99th percentile score). Number

Step2:UntangletheStimulus

Theauthorstartswiththeopinionthatgoodmoviereviewersshouldbeabletogivepositivereviewstomoviestheydon'tpersonallylike.Thisopinionissupportedbytwofacts:1)Moviereviewers'tastesareoftenverydifferentfromthoseofmostmoviegoers;and2)theroleofamovierevieweristohelppeopledecidewhichmoviestheymightenjoy.

Step3:MakeaPrediction

Inanargument,theconclusionisanopinionthatissupportedbyevidence,whichusuallyconsistsoffacts.Inthisargument,theonlytrueopinionisthefirstsentence,andthatclaimissupportedbythefactsprovided.So,theconclusionisthatgoodmoviereviewersshouldbeabletogivepositivereviewstomoviestheydon'tpersonallylike.

Step4:EvaluatetheAnswerChoices

(B)iscorrect,providinganaccurateparaphraseoftheconclusioninthefirstsentence.

(A)isasubsidiaryconclusionpresentedintheargumentasevidencetosupportthemainconclusion;itsnotthemainconclusionitself.

(C)isafactpresentedintheargument,andfactsarepartoftheevidence,nottheconclusion.

(D)isafact,thatisusedtosupportthesubsidiaryconclusionthatmoviereviewershavetastesthataretypicallydifferentandbetterinformedthanmostmoviegoers.

(E)isafactpresentedasevidencetosupporttheconclusion,nottheconclusionitself.

Page 116: LSAT PrepTest 81 Unlocked · Similarly, PT 81 was the first test since June '07 (PT June '07) to require at least 94 questions correct to get a 172 (99th percentile score). Number

9. (B)FlawStep1:IdentifytheQuestionType

Thequestiondirectlyasksfortheflawintheargument.

Step2:UntangletheStimulus

Theauthorpresentsacorrelation:acertainpartofthebraintendstobelargerinskilledmusiciansthaninpeoplewhodon'treallyplaymusic.Thisleadstheauthortoconcludethatplayinganinstrumentchangesthebrain'sstructure.

Step3:MakeaPrediction

Thisisaprimeexampleoftheflawofcorrelationvs.causation.Thebrainareahappenstobelargerinmusicians(acorrelation),sotheauthorassumesthatplayingmusicisthecauseofthatareabeinglarger.Therearethreeproblemswithsuchcausalarguments:1)Theauthoroverlooksothercauses,i.e.,otherfactorsthatcontributetothesizeofthebrainarea;2)theauthormayhavereversedthecausality,i.e.,alreadyhavingalargerbrainareamayberesponsibleforpeoplechoosingtoplaymusic,nottheotherwayaround;and3)it'sjustacoincidence,i.e.,theresultsarecorrelatedbutneitheronedirectlyaffectstheother.Thecorrectanswerwillexpressoneofthesethreeproblems.

Step4:EvaluatetheAnswerChoices

(B)iscorrect,identifyingtheoverlookedpossibilitythattheauthorhasthecausalityreversed,i.e.,thathavingalargerbrainareacausespeopletoplaymusic,nottheotherwayaround.

(A)isanIrrelevantComparison.Theauthormentionspianosounds,butattributestheabilitytoallmusiciansequally.Thereisnocomparisonmadeorassumedaboutpianistsversusothermusicians.

Page 117: LSAT PrepTest 81 Unlocked · Similarly, PT 81 was the first test since June '07 (PT June '07) to require at least 94 questions correct to get a 172 (99th percentile score). Number

(C)isaOutofScope.Theauthorindicateshighlyskilledmusicianshaveaspecificareaoftheirbrainthatislarger.Theauthorclaimsthisiscausedbyplayinganinstrument.Theauthordoesnotthentakethissupposedphenomenonandapplyitbroadlytootheractivitiesthatcouldalso(allegedly)changebrainstructure.

(D)isOutofScope.Theauthorismerelysuggestingthatplayinganinstrumentcanaffectoneparticularareaofthebrain.Thatdoesn'tmeanlisteningtomusiccan'taffectanotherarea.Thathasnobearingontheauthor'sargument,sotheauthorhasnoneedtoaddressit.

(E)isalsoOutofScope.Theargumentisabouthowplayingmusicaffectsaparticularpartofthebrain.Whatmakessomeoneahighlyskilledmusicianorhowmuchpracticeisinvolvedisentirelyirrelevant.

10. (B)InferenceStep1:IdentifytheQuestionType

Thestimuluswillprovideasetofstatementsthatwill"stronglysupport"thecorrectanswer,makingthisanInferencequestion.

Step2:UntangletheStimulus

Accordingtotheresearcher,hearingjustonesideofacell-phoneconversationcanbedistractingfortworeasons:1)Thelistenerstartstoguesswhattheothersideissaying;and2)thecell-phoneuserspeaksveryloudly.

Step3:MakeaPrediction

Thereisverylittletoworkwithhere,andthusverylittletopredict.Thecorrectanswerwillbeconsistentwiththedistractingqualityofhearing

Page 118: LSAT PrepTest 81 Unlocked · Similarly, PT 81 was the first test since June '07 (PT June '07) to require at least 94 questions correct to get a 172 (99th percentile score). Number

onesideofacell-phoneconversation.Watchoutforanswersthatexaggerateordistorttheseclaims.

Step4:EvaluatetheAnswerChoices

(B)iscorrect.Accordingtothefirstclaim,overhearingacell-phoneconversationcandivertattentionfromwhateversomeoneisdoing.Thatwouldincludeanactivitysuchasdriving.

(A)isaDistortion.Peoplearesaidtobedistractediftheyhearonesideofaconversation,i.e.,theyhearsomebodyelseonacell-phone.Ifadriveristalkingonthephone,thedriverishearingbothsides,notjustone.While,inreallife,thisstatementisprobablyverytrue,itisnotsupportedbytheinformationprovided.

(C)isa180,atworst.Thestatementsonlysupportwhathappenswhenpeoplehearonesideofacallonacell-phone,notatraditionalphone.Besides,thefirstdistraction(listenersguessingwhattheothersideissaying)couldstillapplytotraditionalphones,whichwouldlikelymakehearingonesideofatraditionalphoneconversationsimilarlydistracting.

(D)isExtreme.Overhearingonesideofacell-phonecallmightdivertone'sattention,butperhapsjusttemporarily.Thatdoesn'tnecessarilymeanpeoplewillcompletelylosetrackoftheirthoughts.

(E)isOutofScope.Thesituationdescribedisguessingwhatpeoplearesayingwhenyoucan'thearthem,notguessingwhatpeoplemeanwhenyoudohearthem.Thisalsomakesanunsupportedcomparisonbetweencell-phoneconversationsandotherformsofconversations,whichareneveraddressed.

Page 119: LSAT PrepTest 81 Unlocked · Similarly, PT 81 was the first test since June '07 (PT June '07) to require at least 94 questions correct to get a 172 (99th percentile score). Number

11. (A)ParallelFlawStep1:IdentifytheQuestionType

Thecorrectanswerwilluse"parallelreasoning"toindicatethe"flawednature"oftheargumentprovided,makingthisaParallelFlawquestion.

Step2:UntangletheStimulus

Theauthormentionsthatstudiesshowedpositiveresultsforapromisingnewpaintreatment.However,therewassomethingwrongwiththemethodforeachstudy,sotheauthorconcludesthepaintreatmentisprobablynogood.

Step3:MakeaPrediction

Thestudymethodsmayhavebeenflawedinsomeway,buttheresultscouldstillhavebeenaccurate.Theauthordoesn'tconsiderthat,andthecorrectanswerwilldescribeasituationthatcommitsthesameflaw:Concludingthatsomethingassessedasgoodislikelybadbecauseofsomeproblemwithhowthatitemwasassessed.

Step4:EvaluatetheAnswerChoices

(A)isamatch.Thecakewasassessedasgood(itwonthecontest),buttheauthorarguesthatit'sprobablybadbecauseofsomeproblemwithhowthecakewasassessed(thecriteriawasnotconsistent).Evenwithinconsistentcriteria,thecakecouldstillbegood,justasthepaintreatmentintheoriginalargumentcouldstillbeeffective,despitetheflawedmethodsinthestudies.

(B)doesnotmatch.Thereisnojudgingthequalityofanythingorquestioningthemethodofassessment.

(C)doesnotmatch.Nomethodofassessmentisaddressed,andthisauthorshiftsfromadiscussionofnutritionalvaluetoaconclusionof

Page 120: LSAT PrepTest 81 Unlocked · Similarly, PT 81 was the first test since June '07 (PT June '07) to require at least 94 questions correct to get a 172 (99th percentile score). Number

beingmalnourished,ashiftinscopethatwasneverfoundintheoriginalargument.

(D)doesnotmatch.Nomethodofassessmentisaddressed,andtheauthordoesnotclaimthatapositivejudgmentislikelywrong.

(E)doesnotmatch.Thisdoesnotaddressanymethodofassessment,andtheauthordoesnotsaysomethingclaimedtobegoodislikelybad.

12. (A)StrengthenStep1:IdentifytheQuestionType

Thequestionasksforsomethingthat"stronglysupportstheargument"given,makingthisaStrengthenquestion.

Step2:UntangletheStimulus

Theconclusionisaconditionalprediction:Ifcomputersimulationscantestsafetyfeaturesaseffectivelyastestcrashes,thencompanieswillusefewertestcrashes.Theevidenceisthatcomputersimulationswouldcostalotless.

Step3:MakeaPrediction

ThisisacaseofOverlookedPossibilities.Iftestcrashesaremoreexpensive,thatcouldcertainlyprovideanincentivetocutbackonusingthem.However,theauthor'spredictionisbasedoncomputersbeingequallyeffectiveinprovidinginformationaboutsafetyfeatures.Whatiftestcrashesareusedtoproducemorethanjustsafetyinformation?Theauthordoesn'tconsiderthatandassumestherearenootherbenefitstotestcrashesthatwouldwarrantkeepingthemaround,evenifcomputersimulationsweretoprovideequallyreliablesafetyinformation.Thecorrectanswerwillvalidatethisassumption.

Page 121: LSAT PrepTest 81 Unlocked · Similarly, PT 81 was the first test since June '07 (PT June '07) to require at least 94 questions correct to get a 172 (99th percentile score). Number

Step4:EvaluatetheAnswerChoices

(A)iscorrect,confirmingthatmostoftheimportantinformationgleanedfromtestcrashesis,indeed,safety-related.

(B)isaDistortion.Theauthor'spredictionisbasedontheconditionthatcomputersimulationsbecomemoreinformative.However,evenifthatwerelikely,asthischoicesuggests,thatdoesn'thelpverifythatthepredictionisanymorevalid.Thesameassumptionsandoverlookedpossibilitiespersist.

(C)isOutofScope.Theauthor'sargumentisnotaboutcreatingsafercars.Theauthorjustseemsmoreintentonfindingacheaperwaytotestthem.

(D)isa180.Thecostofdesigningthefeatureshasnoimpactontheargument,astheargumentissolelyabouttestingthefeatures.Nonetheless,ifthecostoftestingisdecreasingandispredictedtodecreasefurther,thenitmayeventuallybejustascheapascomputersimulations.Inthatcase,there'dbenoneedtocutbackontestcrashes.

(E)isanIrrelevantComparison.Whattheaviationindustryneedsisnotnecessarilycomparabletowhattheautomobileindustryneeds.Fortheautoindustry,therecouldstillbeparticularneedsservedbytestcrashesthatwouldnotbeservedbycomputersimulations.

13. (B)FlawStep1:IdentifytheQuestionType

Thecorrectanswerwilldescribehowtheargumentis"vulnerabletocriticism,"commonwordingusedtoindicateaFlawquestion.

Page 122: LSAT PrepTest 81 Unlocked · Similarly, PT 81 was the first test since June '07 (PT June '07) to require at least 94 questions correct to get a 172 (99th percentile score). Number

Step2:UntangletheStimulus

Thelegislatorconcludes([s]o)thatacertainactshouldbeapproved.Theevidenceisthatacolleaguerecommendsrejectingtheactbecauseitwoulddeterinvestment.However,thelegislatorquestionsthatreasoningbecausethecolleaguefavoredotheractsinthepastthatdeterredinvestment.

Step3:MakeaPrediction

Thelegislatorismakinganadhominemattack.Thelegislatorisquestioningthecolleaguemerelyonherpreviousactions.It'spossiblethatthecolleagueisagainstdeterringinvestment,buttherewasanoverridingconcerntotheearlierlegislation.Insteadofattackingthecolleaguepersonallyforherpreviousvotingrecord,thelegislatorshouldhavefocusedonherreasoningforrejectingthisact.Thecorrectanswerwilldescribethisflaw.

Step4:EvaluatetheAnswerChoices

(B)iscorrect,describingthelegislator'sfailuretoaddressthecolleague'sreasoning,insteadconcentratingonherpreviousvotingrecord.

(A)isaDistortion.Attackingone'scharactertraitsisaformofadhominemattack,butthelegislatorisnotdoingthat.Thelegislatorisattackingthecolleague'sprioractions,nothercharactertraits.

(C)isOutofScope.Thelegislatordoesnotaddresswhichpositionismorepopular,anddoesnotassumeeitherway.

(D)isOutofScope.Thelegislatordoesnotassumeanythingaboutvoters.Thisargumentissolelyaboutthecolleague'sopinionandthereasontobeskepticalofthat.

Page 123: LSAT PrepTest 81 Unlocked · Similarly, PT 81 was the first test since June '07 (PT June '07) to require at least 94 questions correct to get a 172 (99th percentile score). Number

(E)isalsoOutofScope.Ifanything,thelegislatorwouldwelcomethisinformationasitwouldshowareasonwhythecolleague'sreasoningisnotpersuasive.Thecolleaguedoesn'treallycareaboutinvestment;she'sjusttryingtoplacateherconstituents.

14. (A)InferenceStep1:IdentifytheQuestionType

Thecorrectanswerwouldlogicallyfillintheblankattheendoftheargumentgiven.TheblankisprecededbytheconclusionKeywordso,indicatingthattheblankwillcontainaconclusiondirectlysupportedbythegivenevidence.ThatmakesthisanInferencequestion.

Step2:UntangletheStimulus

ThefirstclaimisFormalLogic:Toincreaseefficiencysignificantly,acomputersystemneedstomakeemployeesadoptanewwayofworking.

Ifefficiencysignificantlyup→adaptnewproductiveways

Then,theauthorclaimsthatthenewcomputersystemfortheMinistryofTransportationwillfitthewayemployeescurrentlywork.

Step3:MakeaPrediction

ThenewsystemfortheMinistryofTransportationfailstomeetthenecessaryconditionforincreasingefficiency.Ifitmerelyfitswithexistingwaysofworking,thenemployeesdon'tneedtoadoptanewwayofworking.Thesufficienttermofthecontrapositiveistriggered.

If~adaptnewproductiveways→efficiency~significantlyup

Page 124: LSAT PrepTest 81 Unlocked · Similarly, PT 81 was the first test since June '07 (PT June '07) to require at least 94 questions correct to get a 172 (99th percentile score). Number

Thus,bytheFormalLogicinthefirststatement,thelogicalconclusionisthatthenewsystemwillnotincreaseefficiency.

Step4:EvaluatetheAnswerChoices

(A)iscorrect,presentingthelogicalresultoftheFormalLogicbasedontheinformationgivenaboutthenewsystem.

(B)isOutofScope.Thereisnothingintheargumenttosupportwhythesystemwouldn'tfunctionproperly.

(C)isaDistortion.Perhapstheministryisabsolutelyconcernedwithproductivity,butaremisguidedorunawarethattheirdecisiontomaintainexistingwaysofworkingwon'tincreaseproductivity,despitethenewcomputersystem.Alternatively,perhapstheministryfeelsthatemployeesarealreadyworkingatpeakproductivityandthere'sjustnoreasontochangethat.

(D)isOutofScope.Theauthormakesnoargumentaboutwhetherthesystemwouldbeworthwhileornot,andthere'snoinformationtosupportswitchingprocessesfrommanualtoautomated.

(E)isOutofScope.Theauthor'sargumentrevolvesonwhat'snecessaryforthesystemtoincreaseefficiency.Theeaseofusingthesystemisofnoconcerntotheauthor.

15. (A)FlawStep1:IdentifytheQuestionType

Thecorrectanswerwilldescribedhowtheargumentis"vulnerabletocriticism,"acommonindicationofaFlawquestion.

Page 125: LSAT PrepTest 81 Unlocked · Similarly, PT 81 was the first test since June '07 (PT June '07) to require at least 94 questions correct to get a 172 (99th percentile score). Number

Step2:UntangletheStimulus

Thecolumnistconcludes([s]o)thatcarmanufacturersareprobablyexaggeratingtheircars'normalfueleconomy.Thisisbasedontherelativelyweakfuelperformanceofthethreecarsownedbythecolumnist.

Step3:MakeaPrediction

Therearetwopotentialproblems.Thefirstisthattheadvertisedfueleconomyissaidtooccur"undernormaldrivingconditions."Perhapsthecolumnistdrivesinabnormalconditions.However,evenassumingthecolumnistdoesdriveundernormalconditions,theconclusionaboutcarsingeneralisbasedonwhathappenedwithjustthreecars.Thatisfartoosmallasamplesize,makingthisadirecttestoftheflawofrepresentativeness.

Step4:EvaluatetheAnswerChoices

(A)iscorrect,identifyingthecommonlytestedflawofusingapotentiallyunrepresentativesample.

(B)isExtreme.Thecolumnistdoesn'thavetoassumeeveryregionhasthesamedrivingconditions.Someregionscanhaveunusualconditions,andthatwouldhavenoeffectonthecolumnist'sargument.

(C)isa180.Thecolumnistisaccusingthemanufacturersofbeingunreliable,notoverlookingthatpossibility.

(D)isaDistortion.Theauthormightbeaccusingmanufacturersofknowinglyinflatingthefueleconomynumbers,butthatdoesn'tmeanthecarsfailtomeetefficiencystandards.Theycouldbewellabovestandards,butthemanufacturersjustmarketthemasevenbetter.

Page 126: LSAT PrepTest 81 Unlocked · Similarly, PT 81 was the first test since June '07 (PT June '07) to require at least 94 questions correct to get a 172 (99th percentile score). Number

(E)isnotaccurate.Themeaningoffueleconomydoesnotchangeintheargument.Itreferstothedistanceacarwilltravelgivenacertainamountoffuel(e.g.,thecommonlyadvertised"milespergallon").

16. (C)WeakenStep1:IdentifytheQuestionType

Thequestiondirectlyasksforsomethingthat"weakenstheargument,"makingthisaWeakenquestion.

Step2:UntangletheStimulus

Accordingtotheauthor,tenantsdon'thaveanincentivetoconserveelectricitywhentheydon'tpaytheelectricbill.Thus,theauthorconcludesthatinstallingmetersandmakingthetenantspaytheelectricbillwillleadtoenergyconservation.

Step3:MakeaPrediction

Theauthor'sconclusionisaprediction.Predictionsgenerallyhavethesameassumption:nothingrelevantisgoingtochangethatmightaffecttheexpectedresults.Inthiscase,theauthorassumesthatmakingtenantspayfortheelectricityisnotgoingtoleadtosomeoverlookedsituationthatwouldactuallymakeenergyconversationlesslikely.Thecorrectanswerwillpointoutapotentialchangethatcouldpreventthepredictedoutcome.

Step4:EvaluatetheAnswerChoices

(C)iscorrect.Thissuggeststhatmakingthetenantspaywouldtakeawayastrongincentivefromlandlords.Iftheystopsupplyingtenantswithenergyefficientappliances,thatcouldmaketheenergyconservationsituationworse,notbetter.

(A)isirrelevant,asitdoesnotaddresswhatwouldhappeniftenants

Page 127: LSAT PrepTest 81 Unlocked · Similarly, PT 81 was the first test since June '07 (PT June '07) to require at least 94 questions correct to get a 172 (99th percentile score). Number

dopaytheelectricbillsandwhateffectanyofthishasonenergyconservation.

(B)ispotentiallya180.Ifpeopleareeducatedaboutenergyconservation,thenit'sevenmorelikelythey'dstartconservingenergymoreiftheysuddenlybecameresponsibleforpayingtheelectricbill.

(D)isOutofScope.Theargumentisnotaboutthelikelihoodorfeasibilityofinstallingelectricmeters.Theargumentisaboutwhatwouldhappeniftheywereinstalled,regardlessofthecost.

(E)isalsoOutofScope.Evenifthereareotherwaystogetpeopletoconserveenergy,thatdoesnotmeantheauthor'splanwon'twork.

17. (D)RoleofaStatementStep1:IdentifytheQuestionType

Thequestionstempresentsaclaimfromthestimulusandasksfortheroleitplaysintheargument,makingthisaRoleofaStatementquestion.

Step2:UntangletheStimulus

Startbyidentifyingtheclaiminquestion,whichappearsinthesecondsentence.Then,breakdowntheargument.Theauthorstartsbynegatingaposition,whichisoftenthesignofaconclusion.Sureenough,theauthor'sconclusionisthatyoucan'thavepunishmentsbeproportionaltoacrime'sseriousnessandgiveharsherpunishmentstorepeatoffenders.Asevidence,theauthorindicatesanimplicationofthisineffectiveplan:Itsuggestsyears-oldactionsarerelevanttonewoffenses.Inthat,allactionswouldbeconsideredrelevant,andthatwouldmaketheproportionalpunishmentconceptimpossibletoapply.

Page 128: LSAT PrepTest 81 Unlocked · Similarly, PT 81 was the first test since June '07 (PT June '07) to require at least 94 questions correct to get a 172 (99th percentile score). Number

Step3:MakeaPrediction

Thephrase"[i]timplies"indicatesthattheclaiminquestionisanimplicationofaposition.Thatpositionistheonetheauthorcallsunsustainable.Thecorrectanswerwillidentifytheclaiminquestionasanimplicationofaplantheauthorargueswon'twork.

Step4:EvaluatetheAnswerChoices

(D)iscorrect.Itisanimplication(i.e.,consequence)oftheviewtheauthorrejects.

(A)isaDistortion.Theclaiminquestionisusedtosupporttheconclusion,buttheauthorprovidesno"groundstoaccept"thatclaim.Theauthorjustpresentsitasfactandexpectsthereadertoacceptitwithoutevidence.

(B)isa180.Thepositionthatimpliestheclaiminquestionisbeingrejectedbytheauthor,notdefended.

(C)isaDistortion.Theconclusionisinthefirstsentence("[t]heposition...isunsustainable),andtheauthoroffersnoevidencetosupporttheclaiminquestion.

(E)isaDistortion.Theclaiminquestionismerelypartofastringofevidence,butthereisnointermediateconclusionforwhichthisclaimprovidessupport.

18. (D)Assumption(Necessary)Step1:IdentifytheQuestionType

Thequestiondirectlyasksforanassumption,andonethatis"requiredby"theargument.ThatmakesthisaNecessaryAssumptionquestion.

Page 129: LSAT PrepTest 81 Unlocked · Similarly, PT 81 was the first test since June '07 (PT June '07) to require at least 94 questions correct to get a 172 (99th percentile score). Number

Step2:UntangletheStimulus

Thebloggerdescribeshowthemediahaschangedfromfocusingonobjectivitytoembracingpartisanreporting.Thebloggerarguesthatthischangeisbasedonchangingbusinessstrategies.Inthepast,newspapershadnoseriousrivals,sotheirbiggestgoalwastoavoidbeingoffensive.

Step3:MakeaPrediction

ThisisacaseofMismatchedConcepts.Ifthegoalofnewspaperswastoavoidbeingoffensive,whatdoesthathavetodowithbeingobjective?Thebloggermustassumethatobjectivereportingwasconsideredinoffensive—attheveryleast,it'snotasoffensiveasthepartisanreportingthatismoreprominentintoday'smediawithitsnewerbusinessstrategies.

Step4:EvaluatetheAnswerChoices

(D)iscorrect,makingtherequisiteconnectionbetweenobjectivereportingandthelikelihoodofbeingoffensive.

(A)isaDistortion.Theargumentisaboutthepartisanshipofthejournalismandthereporting,notofthejournaliststhemselves.

(B)isOutofScope.Theargumentisaboutthestandardsusedbyjournalistsandwhatmayormaynotoffendreaders.That'snotnecessarilythesameasthepreferencesofreaders.Objectivitymaybelesspreferred,butalsolessoffensive.

(C)isOutofScope.Theargumentisnotabouthowpopularthemediais.It'saboutthechangeinstylefromobjectivetopartisan,andwhetherthat'sbasedonachangeinviewsregardingoffendingreaders.

Page 130: LSAT PrepTest 81 Unlocked · Similarly, PT 81 was the first test since June '07 (PT June '07) to require at least 94 questions correct to get a 172 (99th percentile score). Number

(E)isExtremeanda180.Thebloggerdoesnotarguethatthereisnobasisforbeingobjective.Ifanything,theauthorpresentsabasisusedinthepast:tryingtoavoidoffendingthereader.

19. (C)InferenceStep1:IdentifytheQuestionType

Thequestionasksforsomethingthatcanbe"properlyinferred"fromthestatementsprovided,makingthisanInferencequestion.

Step2:UntangletheStimulus

TheauthorbeginswithapieceofFormalLogic:Agovernmentpracticethatcouldleadtoabuseofpowershouldnotbeperformedunlessthere'sacompellingreasontodoso.Theauthorprovidesanexampleofkeepingsecrets,whichcanbejustified.However,whenthereasonsarenotcompellingorwheneventheexistenceofthesecretisnotrevealed,thatcanleadtoanabuseofpower.

Step3:MakeaPrediction

IthelpstotranslatetheopeningFormalLogicanditscontrapositive.

Ifundertakepracticethatcouldleadtoabuse→compellingreason

If~compellingreason→~undertakepracticethatcouldleadtoabuse

Theauthorclaimsthatkeepingsecretscanbejustified,inwhichcasetheremustbeacompellingreasonfordoingso.However,theauthorthensayssecretsareoftenkeptforinsubstantialreasons,inwhichcasesitisnotjustifiedtokeepthosesecrets.Theauthoralsosaysthatconcealingtheexistenceofasecretcouldalsoleadtoabuseofpower.Again,bythe

Page 131: LSAT PrepTest 81 Unlocked · Similarly, PT 81 was the first test since June '07 (PT June '07) to require at least 94 questions correct to get a 172 (99th percentile score). Number

logic,therewouldneedtobeacompellingreasonforconcealingthatfact.Otherwise,there'snojustification.

Step4:EvaluatetheAnswerChoices

(C)issupported.Bythestatements,concealingasecretcouldleadtoabuseofpower,andthelogicdictatessuchactionshouldnotbeundertakenunlessthere'sacompellingreason.

(A)isExtreme.Iftheactisnotjustified,it'sprobablybecausethere'snocompellingreasontodoit.However,there'snoindicationthatthishappensinmostcases.Atworst,theauthorsaysthatinsubstantialreasoninghappens"toooften,"butthatdoesn'tnecessarilymeanmostofthetime.

(B)isaDistortion.Ifthere'sacompellingreasontokeepasecret,thatjustmeansthekeepingofthatsecretmaybejustified.Thatdoesn'tmeanitwon'tfacilitateabuseofpower.

(D)isExtreme.Iftheydon'thaveacompellingreasontoconcealinformation,thentheyshouldnotconcealthatinformation...ifitwouldleadtoabuseofpower.However,there'snocertaintythatallsuchinformationabsolutelywillleadtoanabuseofpower.

(E)isaDistortion.Therequirementforkeepingasecretisthatthere'sacompellingreasontodoso.Evenifkeepingasecretdoesmakeiteasiertoabusepower,acompellingreasoncanoverridethatconcern.

20. (E)Assumption(Sufficient)Step1:IdentifytheQuestionType

Page 132: LSAT PrepTest 81 Unlocked · Similarly, PT 81 was the first test since June '07 (PT June '07) to require at least 94 questions correct to get a 172 (99th percentile score). Number

Thephrase"ifassumed"indicatesthecorrectanswerwillbeanassumptionthatguaranteestheconclusion,makingthisaSufficientAssumptionquestion.

Step2:UntangletheStimulus

Accordingtotheauthor,somemusiciansembracethetheorythatmusicisjustsoundswithnomeaning.Theauthorconcludes([t]hus)thattheirmusicdoesnotconformtothistheory,i.e.,thereissomemeaning—it'snotjustabunchofsounds.Theevidenceforthisisthatthesemusiciansexplaintheirintentionsbeforeperforming.

Step3:MakeaPrediction

AswithalmostallargumentsinSufficientAssumptionquestions,thisonerestsonMismatchedConcepts.Theconclusionimpliesthatthesongshavemeaning,whiletheevidencemerelytalksabouthowthemusiciansexplaintheirintentions.Theassumptionconnectsthoseconcepts:Explainingtheintentionsindicatesthatthemusichassomemeaning.

Step4:EvaluatetheAnswerChoices

(E)iscorrect.Ifmusicwithnomeaningisnotexplained,thenbycontrapositive,ifmusicisexplained,itmusthavemeaning,confirmingtheauthor'sargument.

(A)isOutofScope.Theabilitytothinksymbolicallyhasnothingtodowiththeauthor'sargument.Also,theargumentisfocusedonwhetherornotthemusichasmeaningatall,nothowdifficultitistocreatemusicwithmeaning.

(B)isaDistortion.Theauthorclaimsthatmusicians"encourageaudienceacceptance,"butthatdoesn'tmeanacceptanceisnecessaryfor

Page 133: LSAT PrepTest 81 Unlocked · Similarly, PT 81 was the first test since June '07 (PT June '07) to require at least 94 questions correct to get a 172 (99th percentile score). Number

musictohavenomeaning.Infact,themusiciansprobablybelievetheirmusichasnomeaningwithorwithoutaudienceacceptance.

(C)isaDistortion.Thiscombinesalotofideasfromtheargument(e.g.,randomseriesofsounds,meaning,audienceacceptance).However,thisonlyindicateswhatwouldmakesomemusicappealing.Thisdoesnotverifytheauthor'sconclusionaboutwhetherornotmusicdoeshavemeaning.

(D)isaDistortion.Theargumentisnotaboutwhetherornotpeoplewillenjoythemusic.Theargumentisfocusedonwhetherornotthemusichasmeaning.

21. (C)MethodofArgumentStep1:IdentifytheQuestionType

Thequestionasksfortheauthor's"techniqueofreasoning,"makingthisaMethodofArgumentquestion.

Step2:UntangletheStimulus

Theauthorstartsoffbyarguingthatevolutiondoesnotalwaysmaximizethepotentialforsurvival.Therestoftheargumentisanextendedexampleofmoose,whichevolvedlargerantlerstobetterfightoffcompetition,butalsomakesthemmorevisibleandvulnerabletopredators.

Step3:MakeaPrediction

Thebulkoftheargumentisanexampleusedtoshowhowevolutiondoesnotalwaysimprovethesurvivalrateofanorganism.Thecorrectanswerwilldescribethistechniqueofcounteringanideaviaexample.

Step4:EvaluatetheAnswerChoices

Page 134: LSAT PrepTest 81 Unlocked · Similarly, PT 81 was the first test since June '07 (PT June '07) to require at least 94 questions correct to get a 172 (99th percentile score). Number

(C)iscorrect.Theauthorchallengestheideathatevolutionisallaboutsurvivalbypresentingthecounterexampleofmooseandtheirantlers.

(A)isOutofScope.Theauthoriscounteringageneralideaaboutevolution,butthereisnospecificcompetingargumentthattheauthorisattacking.

(B)isaDistortion.Ananalogyisusedwhenanauthorcomparesonespecificcircumstancetoadifferentbutsimilarspecificcircumstance.However,theauthoronlyraisesonespecificcircumstancetoaddressageneralclaim.That'sanexampleorcounterexample,notananalogy.

(D)isaDistortion.Theexampleraisedisaboutmoose,andit'sentirelyrelevanttothediscussionofevolution.Theauthorwouldnotdisputeherownexample.

(E)isaDistortion.Theauthorusesanexampletoundermineaclaim,buttheclaimisnotshowntobeself-contradictory.Infact,theclaim(evolutionsupportssurvival)maybejustfineinsomecircumstances,justnotinthemooseexample.

22. (D)WeakenStep1:IdentifytheQuestionType

Thequestiondirectlyasksforsomethingthat"weakensthebiologist'sargument,"makingthisaWeakenquestion.

Step2:UntangletheStimulus

Thebiologistdescribeshow,whenexposedtovariouscolorsoflight,aparticularspeciesofbacteriagravitatestowardashadeofred,alightcolorthataidsitschlorophyllinproducingenergy.Thebiologist

Page 135: LSAT PrepTest 81 Unlocked · Similarly, PT 81 was the first test since June '07 (PT June '07) to require at least 94 questions correct to get a 172 (99th percentile score). Number

concludesthatthebacteriadetecttheredcolorbymonitoringitsenergylevels.

Step3:MakeaPrediction

Thebiologistiscommittingacorrelationvs.causationflaw.Thereisacorrelationbetweenenergylevelsandtheredcolor,butthebiologistassumestheincreasedenergypotentialiswhat'scausingthebacteriatomovetowardtheredlight.Thiscouldbeweakenedbyshowinganalternativeexplanation(i.e.,theyaremovingtowardtheredlightforadifferentreason)orbyshowingthatit'sjustacoincidence(i.e.,theenergylevelhasnoeffectonwhichcolorischosen).

Step4:EvaluatetheAnswerChoices

(D)iscorrect.Ifbluewouldspuranequallevelofenergycreation,thatsuggeststhere'sanotherreasonthebacteriaaregravitatingtowardredonly.

(A)isa180.Ifthebacteriastopgravitatingtowardredwhentheydon'thavechlorophyll,thatsuggeststheenergyproducedbychlorophylldoesindeedinfluencethebacteria'sbehavior,confirmingratherthanweakeningthebiologist'sassertion.

(B)isalsoa180.Thissuggeststhebacteriaareseekingoutmaximumenergyproduction,whichonlyconfirmsthebiologist'sargument.

(C)isyetanotherstrengthener.Iftheredareawaswarmer,thatmightbeanalternativeexplanationforthebacteria'sbehavior.However,ifthetemperatureisthesame,asthischoicesuggests,thenwarmthisnotafactor,makingitmorelikelythebiologist'sclaimiscorrect.

Page 136: LSAT PrepTest 81 Unlocked · Similarly, PT 81 was the first test since June '07 (PT June '07) to require at least 94 questions correct to get a 172 (99th percentile score). Number

(E)isanIrrelevantComparison.There'snoindicationherewhyotherbacteriagravitatetowardothercolors.Perhapstheycontainsomeothersubstancethatproducesenergybetterunderothercolors.Inthatcase,thatwouldconfirm,notweaken,thebiologist'sclaimthatenergyproductionisamajorfactor.

23. (B)FlawStep1:IdentifytheQuestionType

Thequestiondirectlyasksforananswerthatdescribestheflawoftheargument.

Step2:UntangletheStimulus

TheargumentbeginswithFormalLogic:Iflegislationistheproductofgroupsnegotiatingandcompromising,thennoneofthosegroupswillbesatisfied.Theauthorthenconcludesthat,becauseallofthegroupsinvolvedinthenewtradeagreementareunsatisfied,compromisesmusthavebeenmade.

Step3:MakeaPrediction

FormalLogicinaFlawquestion?Theflawofnecessityvs.sufficiencyishighlyprobable.Sureenough,inthisargument,theFormalLogicdictatesthatcompromiseissufficienttoguaranteeunhappyparticipants.

Ifcompromise→unhappyparticipants

Theauthorthenconcludesthatthepresenceofunhappygroupsindicatestheremusthavebeencompromises,suggestingthatcompromiseisanecessaryconditionforproducingunhappyparties.

Ifunhappyparticipants→compromise

Page 137: LSAT PrepTest 81 Unlocked · Similarly, PT 81 was the first test since June '07 (PT June '07) to require at least 94 questions correct to get a 172 (99th percentile score). Number

Thatisnotlogicallysound,andthecorrectanswerwilldescribethiscommonlytestedflaw.

Step4:EvaluatetheAnswerChoices

(B)iscorrect.Theauthorconcludesthatcomprisewasnecessary(i.e.,itmusthavehappened)foraresult(i.e.,unhappygroups)merelyfromtheclaimthatcompromiseleadstounhappygroups.Thegroupscouldhavebeenunhappyformanyotherreasons.

(A)isaDistortion.Theconclusiondoesn'tmerelyrestatetheevidence.Itgetsthelogicoftheevidencebackward.

(C)isnotaccurate.Thissuggeststheflawofequivocation,butalltermsintheargumentareusedconsistentlyandneverchangemeaning.

(D)isExtreme.Theauthoronlyarguesthatlegislationinvolvingcompromiseswillensureunhappyparties.However,ifthere'snoneedforcompromise,thenit'spossibleforallpartiestobesatisfied.Theauthorneverassumesotherwise.

(E)isaDistortion.Atradeagreementwouldbeapieceoflegislation,andthere'snothingaboutthetradeagreementthatwouldsuggestitdoesn'tapplytotheprincipleathand.

24. (B)InferenceStep1:IdentifytheQuestionType

Thecorrectanswerwillbe"stronglysupportedbytheinformation"given,makingthisanInferencequestion.

Step2:UntangletheStimulus

Page 138: LSAT PrepTest 81 Unlocked · Similarly, PT 81 was the first test since June '07 (PT June '07) to require at least 94 questions correct to get a 172 (99th percentile score). Number

Followinganaccidentatapowerplant,researchersfoundthreeradioactiveisotopes(callthemI,Te,andCs),butnoheavyisotopes.Thereareonlytwopossiblesources:spentfuelrodsortheplant'score.However,isotopeTeisneverfoundinspentfuelrods(insignificantquantities),andradioactivematerialreleaseddirectlyfromthecorewouldhavecontainedheavyisotopes.SowhereareisotopesI,Te,andCscomingfrom?Theauthorprovidesonemoreclue:Steamwasreleasedthatmayhavecontactedthecore,eventhoughitcaneasilydissolvethosethreeradioactiveisotopes.

Step3:MakeaPrediction

Thekeyhereisnottogettoocaughtupinthescientificterms.Insimpleterms,researchersfoundthreechemicalitemsintheair.Onepossiblesource?Fuelrods.Butfuelrodsdon'tcontainoneofthechemicalsinsignificantquantities.Thatleavestheotherpossiblesource:thecore.However,iftheycamefromthecoredirectly,therewouldalsohavebeenheavyisotopes.So,theymusthavebeenreleasedindirectly.Andthat'swherethesteamcomesin.Thechemicalsmusthaveescapedfromthecoreindirectlythroughthesteam.

Step4:EvaluatetheAnswerChoices

(B)issupported.Withonlytwopossiblesources(spentfuelrodsorthecore)andonesourceeliminated(spentfuelrodsdon'tcontainenoughTe),thechemicalsmusthavecomefromthesecondsource:thecore.Andtheywouldn'tbeejecteddirectly,sotheymusthavetakenanindirectroute:thesteam.

(A)isaDistortion.Becausedirectejectionwouldhaveincludedheavyisotopes,it'ssuggestedthatTe(andtheothernon-heavyisotopes)was

Page 139: LSAT PrepTest 81 Unlocked · Similarly, PT 81 was the first test since June '07 (PT June '07) to require at least 94 questions correct to get a 172 (99th percentile score). Number

ejectedindirectly.However,ifdirectejectionhadoccurred,there'snoreasontobelieveTewouldn'thaveappearedthen,too.

(C)isnotsupported.Thespentfuelrodscouldn'tbethesourceoftheTe,buttheystillcouldhavebeenbroken.

(D)isOutofScope.Theauthorimpliesthatthematerialfounddidnotcomefromspentfuelrodsordirectlyfromthecore.Whileit'spossiblethatotheritemswerefoundthatcamefromthesesources,there'snothinginthestatementsthatsuggestassuch.

(E)isnotsupported.It'sonlystatedthatthespentfuelrodsdonotcontainTe,butthere'snoindicationwhattheydocontain,whetheritbealotofotherheavyisotopesornot.

25. (E)ParallelReasoningStep1:IdentifytheQuestionType

Thecorrectanswerwillbeacompleteargumentwithreasoning"mostsimilar"tothatintheargumentgiven.ThatmakesthisaParallelReasoningquestion.

Step2:UntangletheStimulus

TheargumentgivenisbasedonsomebasicFormalLogic.Iftwosciences(ecologyandphysics)wereevaluatedequally,ecologywouldnotbeasuccessfulscience.

Ifevaluatedbysamecriteria→ecologyfails

However,itissuccessful.Therefore,theauthorconcludesthatthetwosciencesarenotevaluatedequally.

Page 140: LSAT PrepTest 81 Unlocked · Similarly, PT 81 was the first test since June '07 (PT June '07) to require at least 94 questions correct to get a 172 (99th percentile score). Number

Ifecology~fail→~evaluatedbysamecriteria

Step3:MakeaPrediction

TheargumentpresentsapieceofFormalLogicandthenreachesitsconclusionbyusingthecontrapositive.Ingenericterms,theargumentisstructuredasso:IfXweretrue,thenYwouldbetrue.HoweverYisnottrue,soXisnottrue.Thecorrectanswerwillconformtothisexactsamestructure.

Step4:EvaluatetheAnswerChoices

(E)iscorrect,usingthesameargument-by-contrapositivestructure.Ifanyeconomictheorywereadequate,accurateforecastscouldbemade.Accurateforecastscan'tbemade,soeconomictheoriesarenotadequate.

Ifadequatedescription→accurateeconomicforecasts

If~accurateeconomicforecasts→~adequatedescription

(A)doesnotmatch.Here,therearetwoconsequences(connectedbyor)iftaxesincrease.Oneofthoseconsequencescan'thappen,sotheauthorconcludestheotheronemust.However,there'snoindicationthattheconditionofasalestaxincreasewillhappen.There'salsonouseofthecontrapositive.And,theconclusionisaprediction,whichissomethingtheoriginalauthornevermakes.

(B)doesnotmatch.TheFormalLogichereis:Ifthegalleryborrowssomeworks,thenitsexhibitwouldbethelargestever.However,unliketheoriginalargument,thisargumentshiftstonewtopicssuchasthedemandforlargerexhibitsandthewillingnessofgalleriestolendouttheirworks.

Page 141: LSAT PrepTest 81 Unlocked · Similarly, PT 81 was the first test since June '07 (PT June '07) to require at least 94 questions correct to get a 172 (99th percentile score). Number

Plus,theconclusionisaprediction,whichdoesnotmatchtheconclusionoftheoriginalargument.

(C)doesnotmatch.ThissimplyappliestheFormalLogicasitiswrittenwithoutusingthecontrapositive.Ingenericterms,itsays:IfXweretrue,Ywouldbetrue.Xwillbetrue,soYwillbetrue,too.Whilethelogicissound,itdoesnotmatchtheoriginal.Further,itmakesaprediction,whichisnotlogicallyequivalenttotheoriginalargument.

(D)doesnotmatch,anditcommitsalogicalflaw.ItsimplynegatestheFormalLogicwithoutreversingit.Ingenericterms,thisissaying:IfXweretrue,Ywouldbetrue.However,Xisnotusuallytrue,soYisnotusuallytrue.That'snotlogicallysound,anditdoesnotmatchthestructureoftheoriginalargument.

Page 142: LSAT PrepTest 81 Unlocked · Similarly, PT 81 was the first test since June '07 (PT June '07) to require at least 94 questions correct to get a 172 (99th percentile score). Number

LSATPrepTest81

Page 143: LSAT PrepTest 81 Unlocked · Similarly, PT 81 was the first test since June '07 (PT June '07) to require at least 94 questions correct to get a 172 (99th percentile score). Number

SECTIONIV:LOGICGAMES

Games

Game1:RuralandUrbanPhotoEssays

Game2:ConcertMusicians

Game3:AmusementCenterObstacleCourse

Game4:ManagersinManila,Sydney,andTokyo

GAME1:RURALANDURBANPHOTOESSAYS

Q# QuestionType Correct Difficulty

1 PartialAcceptability E Checkyouronlineresources.

2 CouldBeTrue B Checkyouronlineresources.

3 “If”/CouldBeTrueEXCEPT C Checkyouronlineresources.

4 MustBeTrue D Checkyouronlineresources.

5 CouldBeTrueEXCEPT A Checkyouronlineresources.

6 RuleSubstitution B Checkyouronlineresources.

GAME2:CONCERTMUSICIANS

Page 144: LSAT PrepTest 81 Unlocked · Similarly, PT 81 was the first test since June '07 (PT June '07) to require at least 94 questions correct to get a 172 (99th percentile score). Number

Q# QuestionType Correct Difficulty

7 CouldBeTrue D Checkyouronlineresources.

8 “If”/CouldBeTrueEXCEPT A Checkyouronlineresources.

9 MustBeFalse(CANNOTBeTrue) E Checkyouronlineresources.

10 “If”/MustBeFalse(CANNOTBeTrue) D Checkyouronlineresources.

11 CompletelyDetermine E Checkyouronlineresources.

GAME3:AMUSEMENTCENTEROBSTACLECOURSE

Q# QuestionType Correct Difficulty

12 Acceptability D Checkyouronlineresources.

13 “If”/MustBeTrue C Checkyouronlineresources.

14 CompleteandAccurateList B Checkyouronlineresources.

15 “If”/MustBeTrue B Checkyouronlineresources.

16 “If”/MustBeTrue A Checkyouronlineresources.

GAME4:MANAGERSINMANILA,SYDNEY,ANDTOKYO

Q# QuestionType Correct Difficulty

17 Acceptability C Checkyouronlineresources.

18 CompletelyDetermine B Checkyouronlineresources.

19 MustBeTrue D Checkyouronlineresources.

Page 145: LSAT PrepTest 81 Unlocked · Similarly, PT 81 was the first test since June '07 (PT June '07) to require at least 94 questions correct to get a 172 (99th percentile score). Number

20 CouldBeTrue A Checkyouronlineresources.

21 “If”/MustBeTrue D Checkyouronlineresources.

22 “If”/CouldBeTrue A Checkyouronlineresources.

23 RuleSubstitution E Checkyouronlineresources.

GAME1:RURALANDURBANPHOTOESSAYSStep1:Overview

Situation:Amagazineassigningphotoessaysforupcomingissues

Entities:Fivephotographers(Fetter,Gonzalez,Howland,Jordt,Kim)andtwothemes(ruralandurban)

Action:Sequencing/MatchingHybrid.Determinetheorderinwhicheachphotographer'sessaywillappear(Sequencing),anddeterminethethemeofeachphotographer'sessay(Matching).

Limitations:Eachessayisassignedtoadifferentphotographer,makingthesequencingstandardone-to-onesequencing.Forthematching,threeessayswillberuralandtwowillbeurban.

Step2:Sketch

Drawtworowsoffiveslotslabeled1through5.Thetoprowwillbeusedtodeterminetheorderofthephotographers,solistthembyinitialnexttothatrow.Thebottomrowwillbeusedtodeterminethetheme,sodrawthreer'sandtwou'snexttothatrow.

Page 146: LSAT PrepTest 81 Unlocked · Similarly, PT 81 was the first test since June '07 (PT June '07) to require at least 94 questions correct to get a 172 (99th percentile score). Number

Youcouldalsodrawasinglerowofslotsandusethetopofeachslotforthephotographerandthebottomofeachslotforthetheme.

Step3:Rules

Rule1establishesthefirstessayasrural.Draw"r"inslot1ofthebottomrow,andcrossone"r"offthelistnexttothatrow.

Rule2createsaBlockofEntities.KimandFetter,inthatorder,willbeconsecutive.

Rule3dictatesthatFetterandKimhavedifferentthemes.Eithermakeanotetothesidefornow,orsomehownotateitundertheblockfromthepreviousrule.Perhaps"x"underoneand"y"undertheother,or"r/u"underoneand"u/r"undertheother,withanotethattheyaredifferent.

Rule4assignsGonzaleztothethirdessay.Draw"G"inslot3ofthetoprow,andcross"G"offthelistnexttothatrow.

Rule5establishesthatthethemeforJordt'sessayisurban.

Step4:Deductions

Numbersareimportantinthisgame.Threephotographerswillhaveruralthemes,whileonlytwohaveurbanthemes.ByRule3,FetterandKimhavedifferentthemes,sooneofthemmusthavearuralthemeandthe

Page 147: LSAT PrepTest 81 Unlocked · Similarly, PT 81 was the first test since June '07 (PT June '07) to require at least 94 questions correct to get a 172 (99th percentile score). Number

otherwillhaveanurbantheme.So,eitherFetterorKimgetsoneofthetwourbanthemesand,byRule5,Jordtgetstheother.Theremainingphotographers,GonzalezandHowland,mustthenhaveruralthemes,alongwitheitherFetterorKim(whoeverdoesn'tgettheurbantheme).

Atthispoint,an"r"canbeplacedunderthe"G"inessay3.Thenthere'stheBlockofKimandFetter.WiththeEstablishedEntityofGonzaleztakingupthethirdessay,KimandFettercouldonlybeassignedessays1and2,respectively,oressays4and5,respectively.Thesetwooutcomeswouldeachestablishthreeofthefivephotographers,suggestingLimitedOptionsareworthwhile.

Inthefirstoption,KimandFetterareassignedtoessays1and2,respectively.Essay1isalreadyestablishedasrural,soessay2forFetterwillbeurban.HowlandandJordtwillbeassignedtoessays4and5,ineitherorder.NotethatHowland'sessaywillberuralandJordt'sessaywillbeurban.

Inthesecondoption,KimandFetterareassignedtoessays4and5,respectively.Itcannotbedeterminedwhichonewillberuralandwhichonewillbeurban.However,thatleavesHowlandandJordtforessays1and2.Essay1isruralandJordthastohaveanurbantheme.So,Howlandmustbeassignedtoessay1,andJordttoessay2.

Page 148: LSAT PrepTest 81 Unlocked · Similarly, PT 81 was the first test since June '07 (PT June '07) to require at least 94 questions correct to get a 172 (99th percentile score). Number

Step5:Questions

1. (E)PartialAcceptabilityStartbyusingstandardAcceptabilitytactics.Gothroughtherulesoneatatimeandeliminatechoicesthatviolatethoserules.Becausethechoicesdon'tlisttheessaythemes,somerulesmayhavetobetestedindirectlyorincombinationwithotherrules.

Withnothemeslisted,Rules1,3,and5cannotbetesteddirectly.(A)and(C)violateRule2bynothavingKimandFetterconsecutive.(B)violatesRule4byputtingGonzalezfirst,notthird.

CombiningRules1and5,thefirstessaymustberural,soitcannotbeJordt's,whichmustbeurban.(D)violatesthat,leaving(E)asthecorrectanswer.

NotethatLimitedOptionscouldhavebeenusedtosaveevenmoretime.Bythetwooptions,thefirstessayhastobeassignedtoHowlandorKim.Thatimmediatelynarrowsthechoicesdownto(C)and(E),and(C)doesnotmatchtheoptionwithHowlandfirstbecauseitsplitsupKimandFetter.

2. (B)CouldBeTrueThecorrectanswerwillbetheonethatispossible.Thefourwrongchoiceswillbeimpossible,i.e.,mustbefalse.

Page 149: LSAT PrepTest 81 Unlocked · Similarly, PT 81 was the first test since June '07 (PT June '07) to require at least 94 questions correct to get a 172 (99th percentile score). Number

IfFetter'sessaywereimmediatelybeforeJordt's,thatwouldcreateathree-personblockofKFJ.WithGonzalezassignedthethirdessay,therewouldbenoroomforsuchablock.Thateliminates(A).

Gonzalez'sessayisthird.IfGonzalez'sessaywereimmediatelybeforeHowland's,thenHowland'sessaywouldbefourth.ThatviolatesnorulesandisevenseenaspossibleinOptionI.Thus,thiscouldbetrue,making(B)correct.Fortherecord:

IfHowland'sorJordt'sessaywereimmediatelybeforeKim's,thatwouldcreateathree-personblockofeitherHKForJKF.WithGonzalezassignedthethirdessay,therewouldbenoroomforeitherblock.Thateliminates(C)and(D).

(E)directlycontradictsRule2,whichstatesthatKim'sessaymustbeimmediatelybeforeFetter's,notGonzalez's.

3. (C)“If”/CouldBeTrueEXCEPTForthisquestion,thefourthissuewillhaveanurbantheme.Thiscouldhappenineitheroption,sodrawbothout.InOptionI,Kim,Fetter,andGonzalez,inthatorder,areassignedtothefirstthreeessayswithrural,urban,andruralthemes,respectively.Ifthefourthessayisurban,thenthefifthessaymustbethefinalruralone.Jordtmusthaveanurbantheme,soJordtisassignedtoessay4,leavingHowlandforessay5.

InOptionII,thephotographersarealldetermined.Forthisquestion,it'snowestablishedthatessay4(Kim's)isurban,makingessay5(Fetter's)rural.

Page 150: LSAT PrepTest 81 Unlocked · Similarly, PT 81 was the first test since June '07 (PT June '07) to require at least 94 questions correct to get a 172 (99th percentile score). Number

Withthat,theseoptionsshowHowland'sessayasfirstorfifth,neverfourth.Thatmakes(C)impossibleandthusthecorrectanswer.Eachoftheremainingchoicesarepossibleinoneofthetwooptions.

4. (D)MustBeTrueThecorrectanswerforthisquestionmustbetrue,whichmeansthefourwrongchoicesmaynotbe,i.e.,couldbefalse.

Considerthemajordeductions.Therearetwourbanthemes.Jordthasone.And,becauseKimandFettermusthavedifferentthemes,oneofthemhastheother.Thatmeanstheremainingphotographers,GonzalezandHowland,musthaveruralthemes.Thatmakes(D)definitivelytrue,andthusthecorrectanswer.Fortherecord:

(A)iscertainlyfalse,asGonzalezisassignedthethirdessayandGonzalezgetsaruraltheme.Thefifthessaycouldberural,butitcouldalsobeurban.So,(B)couldbefalse.AndFetterandKimhavedifferentthemes,butit'spossiblethatFetterhasaruralthemeandKimhasanurbantheme.Thus,(C)and(E)couldbefalse.

5. (A)CouldBeTrueEXCEPTFourchoicesherecouldbethefourthessay.Thatmeansthecorrectanswerwillbetheexception:theonethatcannotbe,i.e.,mustbefalse.

Page 151: LSAT PrepTest 81 Unlocked · Similarly, PT 81 was the first test since June '07 (PT June '07) to require at least 94 questions correct to get a 172 (99th percentile score). Number

LimitedOptionshelpoutalothere.InOptionI,thefourthessaycouldbeeitherJordt'surbanessayorHowland'sruralessay.InOptionII,thefourthessaywillbeKim's,andcouldbeeitherruralorurban.Thatsumsupchoices(B),(C),(D),and(E),whichmeansthosearetheincorrect"couldbetrue"choices.Further,it'simpossibleforthefourthessaytobeFetter'sbecauseGonzalez'sisthirdandFetterhastocomeimmediatelyafterKim,notGonzalez.Thatmakes(A)impossibleandthusthecorrectanswer.

6. (B)RuleSubstitutionThecorrectanswerherewillbeanewrulethatcouldreplaceRule3,regardingFetterandKimhavingdifferentthemes,withoutaffectingthegameinanyway.Inotherwords,Rule3willbeeliminated,andthecorrectanswerhastore-establishthatexactsamerestrictionwithoutaddinganynewrestrictions.

SimplyestablishingHowlandwitharuralthemeisnotenough.WithRule5,Jordthasanurbantheme,butthatstillleavestworuralthemesandoneurbantheme.ThatwouldallowFetterandKimtobothhaveruralthemes.Thus,(A)isnotgoodenough.

However,ifbothGonzalezandHowlandareassignedruralthemes,thatwouldhelp.Then,JordtgetsanurbanthemebyRule5.ThatleavesoneurbanthemeandoneruralthemeforFetterandKim.Thatwouldforcethemtohavedifferentthemes,astheoriginalruledid.AndGonzalezandHowardwerealwaysassignedruralthemesoriginally,sotherearenonewrestrictions.Thatmakes(B)thecorrectanswer.Fortherecord:

(C)doesnotpreventFetterandKimfromhavingthesametheme,anditforcesFettertohavearuraltheme,whichwasnotalwaysthecase.

Page 152: LSAT PrepTest 81 Unlocked · Similarly, PT 81 was the first test since June '07 (PT June '07) to require at least 94 questions correct to get a 172 (99th percentile score). Number

GAME2:CONCERTMUSICIANS

(D)addsarestrictiontoJordt'sessaywhichhappenstobetruebasedonthedeductions.However,itdoesnothingtopreventKimandFetterfromhavingthesametheme.(E)wouldactuallyhavethecompleteoppositeeffect.IfKim'sessayhadthesamethemeasGonzalez'sorHowland's,butnotboth,thenGonzalezandHowlandwouldhavetohavedifferentthemes.So,oneofthemwouldgetanurbanthemealongwithJordt.ThatwouldleaveonlyruralthemesforKimandFetter,givingthemboththesametheme,notdifferent.

Step1:Overview

Situation:Musiciansperformingataconcert

Entities:Sevenmusicians(Lowe,Miller,Nadel,Otero,Parker,Sen,Thomas)

Action:StrictSequencing.Determinetheorderinwhichthemusicianswillperform.Althoughthefirsttworulesareloosesequencingstylerules,Rules3,4,and5makethisaStrictSequencinggame.

Limitations:Eachmusicianperforms,oneatatime.Thisisstandardone-to-onesequencing.

Step2:Sketch

Listthemusiciansbyinitialanddrawaseriesofsevenconsecutivelynumberedslots.

Page 153: LSAT PrepTest 81 Unlocked · Similarly, PT 81 was the first test since June '07 (PT June '07) to require at least 94 questions correct to get a 172 (99th percentile score). Number

Step3:Rules

Rules1and2setuptwoseparatelooserelationships:LoweatsomepointbeforeNadel,andMilleratsomepointbeforeThomas.

Rules3and4setuptwosimilarstrictrelationships.ThereisexactlyonespacebetweenLoweandOtero,andonespacebetweenMillerandParker,thougheachpaircanappearineitherorder.

Rule5presentstwooptions.Parkerwillbefirstorseventh(i.e.,last).Youcandraw"P"overthesketchwitharrowspointingtothefirstandlastspots.However,Parker'splacementdirectlyaffectsMiller'splacement,andthatwillhaveothereffects.So,drawingLimitedOptionsmightbeabettercourseofaction.

Step4:Deductions

Basedonthelastrule,drawtwosketches.Inthefirst,Parkerwillbefirst.ByRule4,Millerwillbethird.ByRule2,ThomasmustperformafterMiller,soThomascanbeanywherebutsecond.Nadelalsocannotbesecond,asNadelmustperformafterLowe(Rule1).ThatmeansthesecondperformercouldbeLoweorOtero,whichwouldmeanLoweandOterotakeuppositionstwoandfour,withMillerinbetween.Or,thesecondperformercouldbeSen,theFloaterofthegame.Also,becauseLowehastoperformbeforeNadel,Lowecannotperformlast.

Page 154: LSAT PrepTest 81 Unlocked · Similarly, PT 81 was the first test since June '07 (PT June '07) to require at least 94 questions correct to get a 172 (99th percentile score). Number

Inthesecondoption,Parkerwillbeseventh.ByRule4,Millerwillbefifth.ThatmeansThomasmustbesixth(Rule2).Thatleavesthefirstfourslotsopen.TheonlydefiniteorderisthatLowemustperformbeforeNadel,soLowecannotbefourthandNadelcannotbefirst.There'salsotherestrictionaboutLoweandOtero,buttheycouldbeinpositions1and3,orinpositions2and4(aslongasOteroisfourth).

Step5:Questions

7. (D)CouldBeTrueThecorrectanswerwillbetheonethatcouldbetrue.Theremainingchoicescannotbetrue,i.e.,theymustbefalse.

WithParkereitherfirstorseventh,Millercouldonlybethirdorfifth(Rule4),neverfourth.Thateliminates(A).

NadelhastoperformafterLowe,socanneverbefirst.Thateliminates(B).

IfOteroisfifth,thenMillercan'tbefifth,soParkercan'tbeseventh.Thus,ParkerwouldbefirstandMillerthird.ThisisOptionI.IfOteroisfifth,thenRule3requiresLowetobethirdorseventh.However,inthisoption,Miller

Page 155: LSAT PrepTest 81 Unlocked · Similarly, PT 81 was the first test since June '07 (PT June '07) to require at least 94 questions correct to get a 172 (99th percentile score). Number

isthird,andLowecannotbelastwithoutviolatingRule1.Thisisimpossible,whicheliminates(C).

ThereisnoruledirectlyrestrictingSen—aFloater—soitwouldseempossibleforSentoperformseventh.Inthatcase,ParkerwouldhavetoperformfirstwithMillerthird,asseeninOptionI.ThatleavesenoughoptionsforplacingLoweandOtero,aswellasNadelandThomas.Thisispossible,making(D)correct.Fortherecord:

IfThomasperformedsecond,Millerwouldhavetoperformfirst(Rule2).ThatwouldforceParkertoperformthird(Rule4),violatingRule5.Thateliminates(E).

8. (A)“If”/CouldBeTrueEXCEPTForthisquestion,OteroperformsearlierthanMiller.Thiscouldhappenineitheroption,sotestthemboth.Thecorrectanswerwillbetheonepersonwhocannotperformfifth.So,eliminateanymusicianwhocouldperformfifthineitherscenario.

InOptionI,forOterotobebeforeMiller,Oterowouldhavetoperformsecond.Inthatcase,Lowewouldhavetoperformfourth(Rule3).Theremainingmusicians,Nadel,Sen,andThomas,couldfillintheremainingpositionsinanyorder.Soanyofthosethreecouldbefifth,whicheliminates(C),(D),and(E).

Page 156: LSAT PrepTest 81 Unlocked · Similarly, PT 81 was the first test since June '07 (PT June '07) to require at least 94 questions correct to get a 172 (99th percentile score). Number

InOptionII,OteroisdefinitelybeforeMillerandMilleristhefifthperformerthere.Thateliminates(B).

Thatleaves(A)asthecorrectanswer,asLowecannotbefifthineitheroutcomewhenOteroisbeforeMiller.

9. (E)MustBeFalse(CANNOTBeTrue)Thecorrectanswerwillbeamusicianwhocannotperformthird,i.e.,itmustbefalsethatmusicianisthird.Theremainingchoiceswilllistmusicianswhocouldperformthird.

InOptionI,Milleristhird,sothateliminates(B).

InOptionII,Lowe,Nadel,Otero,andSenoccupythefirstfourslots,includingthethirdperformance.LoweandOteromustbeseparatedbyonespace.So,it'spossibleforLoweandOterotoperformfirstandthird,ineitherorder,aslongasNadelperformsafterLowe(e.g.,fourth).So,LoweandOterocouldeachperformthird,eliminating(A)and(D).

TheonlyotherpossibilityinOptionIIistohaveLoweandOteroperformsecondandfourth.ForLowetoperformbeforeNadel,LowewouldhavetobesecondwithNadelthird.Nadelcouldbethird,whicheliminates(C).

Page 157: LSAT PrepTest 81 Unlocked · Similarly, PT 81 was the first test since June '07 (PT June '07) to require at least 94 questions correct to get a 172 (99th percentile score). Number

Senistheonlymusicianwhocannotperformthird.AlthoughSenisnotdirectlyaffectedbytherules,placingSenthirdwouldforcetheremainingmusiciansintopositionsthatwindupviolatingtherules.Thus(E)isimpossible,makingitthecorrectanswer.

10. (D)“If”/MustBeFalse(CANNOTBeTrue)Forthisquestion,SenandThomas,inthatorder,areconsecutive.IfParkerwereseventh,Millerwouldbefifth(Rule4),forcingThomastobesixth(Rule2),makingitimpossibleforSenandThomastobeconsecutive.So,thiscouldonlyworkinOptionI,whenParkerisfirst.Inthatcase,Milleristhird.TheblockofSenandThomascannotfilltheonespacebetweenParkerandMiller,andNadelcannotperformsecondbecauseNadelhastoperformafterLowe.Thatleavestwoperformersforthesecondslot:LoweandOtero.IfLoweperformssecond,Oterowouldperformfourth,andviceversa.So,LoweandOteromustbesecondandfourth,ineitherorder.

ThatleavestheblockofSenandThomas,inthatorder,toperformfifthandsixth,orsixthandseventh.Nadelwillfillintheremainingspot.

Page 158: LSAT PrepTest 81 Unlocked · Similarly, PT 81 was the first test since June '07 (PT June '07) to require at least 94 questions correct to get a 172 (99th percentile score). Number

Withthat,Thomascanonlyperformsixthorseventh,notfifth.Thatmakes(D)impossible,andthusthecorrectanswer.

11. (E)CompletelyDetermineThecorrectanswerwillplacesomeoneinsuchawaythatallsevenmusicianscanbeplacedwithabsolutecertainty.

IfLoweperformsfourth,Oterocouldstillperformsecondorsixth,sothere'sstillsomeuncertainty.Thateliminates(A).

IfMillerperformsfifth,ParkerisseventhandThomasissixth.However,that'sthesetupforOptionII,whichstillleavesalotofuncertaintyaboutthefirstfourperformances.Thateliminates(B).

IfNadelisfourth,Lowemustperformearlier.ThatcouldhappeninOptionII.However,inthatcase,LowecouldbefirstwithOterothird,orLowecouldbethirdwithOterofirst.It'snotcompletelydetermined,sothateliminates(C).

IfOteroisthird,thatcouldonlyhappeninOptionIIwithMillerfifth,Thomassixth,andParkerseventh.Lowewouldhavetobefirst,butNadelandSencouldthenperformsecondandfourthineitherorder.That'stwopossibleoutcomes,notone.Thateliminates(D).

IfSenperformsfirst,Parkermustperformseventh.ThatmeansMillerperformsfifthandThomassixth.ThatleavesLowe,Nadel,andOteroforthesecond,third,andfourthperformances.LoweandOteroneedtobeseparated,soNadelmustperformthird,inbetweenthem.LowehastoperformbeforeNadel,soLowemustbesecondandOterofourth.All

Page 159: LSAT PrepTest 81 Unlocked · Similarly, PT 81 was the first test since June '07 (PT June '07) to require at least 94 questions correct to get a 172 (99th percentile score). Number

GAME3:AMUSEMENTCENTEROBSTACLECOURSE

sevenmusiciansareassignedwithcertainty,making(E)thecorrectanswer.

Inapinch,thisseemedthemostlikelyanswerasitplacesSen—theFloater—whowasotherwisenotdirectlyrestricted.

Step1:Overview

Situation:Amusementcenteroperatorsdesigninganobstaclecourse

Entities:Sixobstacles(ropebridge,spinningplatform,tunnel,vaultingapparatus,wall,zipline)

Action:StrictSequencing.Determinetheorderinwhichtheobstacleswillbeplaced.Aquickglanceattherulesrevealsthattheyareallstrictsequencingstylerules.

Limitations:Eachobstacleisincludedandseparate,sothisisstandardone-to-onesequencing.

Step2:Sketch

Listtheobstaclesbyinitialanddrawsixconsecutivelynumberedslots.(Note:Youcanusedualinitialsfortheobstacles,e.g.,"RB"fortheropebridge.However,justusingthefirstinitialofeachobstacleissufficientastheyaredistinctandpresentedalphabetically.Also,itcanbelessconfusingtoseesixindividuallettersratherthanamixtureofsingle-anddouble-letteritems).

Page 160: LSAT PrepTest 81 Unlocked · Similarly, PT 81 was the first test since June '07 (PT June '07) to require at least 94 questions correct to get a 172 (99th percentile score). Number

Step3:Rules

Rule1limitsthespinningplatformtooneoftwopositions:thirdorfourth.Draw"S"above/belowthesketchwitharrowspointingtothethirdandfourthslot.

Rule2createsaBlockofEntitieswiththewallandtheziplineconsecutive,inthatorder.

Rule3preventstheropebridgeandthevaultingapparatusfrombeingconsecutive,ineitherorder.

Step4:Deductions

ByRule2,thewallmustbebeforethezipline,sothewallcannotbelastandtheziplinecannotbefirst.It'salsoimpossibletoplacethewallandziplinethirdandfourth,respectively,asthatwouldleavenoplaceforthespinningplatform.

However,thatleavesfourpossibleplacementsforthewall/ziplineblock.Withonlyfivequestions,it'snotworthdrawingoutallfouroptions.

ItmightseemtemptingtosetupLimitedOptionsbasedonwherethespinningplatformgoes.However,ineitherposition,therearestillmultipleplacesforthewall/ziplineblockthatwouldalsoallowtheropebridgeandvaultingapparatustobeseparated.So,neitheroptionwouldproduceanyfruitfuldeductions.

Page 161: LSAT PrepTest 81 Unlocked · Similarly, PT 81 was the first test since June '07 (PT June '07) to require at least 94 questions correct to get a 172 (99th percentile score). Number

TherearenoNumbersdeductionsandnoDuplicationdeductions.Deductionsarerathersparsehere.However,it'shelpfultonotethatthetunnelisaFloater.Also,thegamethankfullycomeswithabunchofNew-"If"questions,andthatcanoftenindicatealackofmajordeductions.ThefinalMasterSketchshouldreflectwhat'sknownsofar:

Step5:Questions

12. (D)AcceptabilityAswithanyAcceptabilityquestion,gothroughtherulesoneatatimeandeliminateanswersthatviolatethem.

(C)violatesRule1byputtingthespinningplatformfifth.(A)violatesRule2byseparatingthewallandthezipline.(B)and(E)violateRule3byhavingtheropebridgeandvaultingapparatusconsecutive.Thatleaves(D)asthecorrectanswer.

13. (C)“If”/MustBeTrueForthisquestion,thetunnelwillbefirst.Thespinningplatformcouldstillgothirdorfourth,sotestthemboth.

Ifthespinningplatformisthird,thewall/ziplineblockwouldhavetogoafter,eitherin4/5or5/6—sonomatterwhatoneofthewall/ziplineblockwilloccupyslot5.Thatleaveseithertheropebridgeorthevaultingapparatusforthesecondobstacle.

Page 162: LSAT PrepTest 81 Unlocked · Similarly, PT 81 was the first test since June '07 (PT June '07) to require at least 94 questions correct to get a 172 (99th percentile score). Number

Ifthespinningplatformisfourth,thatleavestwosetsofspacesforthewall/ziplineblock:secondandthird,orfifthandsixth.However,ineithercase,thatwouldleaveconsecutivespacesfortheropebridgeandthevaultingapparatus,violatingRule3.Thisoptionisunacceptable.

So,ifthetunnelisfirst,thespinningplatformcanonlybethird,making(C)thecorrectanswer.Theremaininganswersareallpossible,butneednotbetrue.

14. (B)CompleteandAccurateListThecorrectanswerwilllisteverypossiblepositionforthetunnel.Thewrongchoiceswillleavesomethingoutorincludeapositionwherethetunnelcannotgo.

Answeringthisquestionefficientlywouldrequirefindingaverychallengingdeductionatthebeginning.Withoutthat,thisquestionisworthskippingandsavingforlast.Usingsketchesandoutcomesfromotherquestionscansavealotoftesting.TheanswertotheAcceptabilityquestionshowsthatthetunnelcanbesecond,andthesecondquestionofthesetisbasedonthepossibilitythatthetunnelcanbefirst.Unfortunately,everyanswerliststhosepositions,sothatdoesn'thelp.However,thesketchforthelastquestionplacesthetunnelsixth,sothateliminates(A),whichfailstolistsixth.

Page 163: LSAT PrepTest 81 Unlocked · Similarly, PT 81 was the first test since June '07 (PT June '07) to require at least 94 questions correct to get a 172 (99th percentile score). Number

Fromthere,withoutanincrediblededuction,it'sallabouttesting.Startbytestingwhetherthetunnelcouldbethird.Ifitwere,thenthespinningplatformwouldbefourth.Thatleavestwosetsofopenspaces:firstandsecond,andfifthandsixth.

However,thiscannothappen.Thewall/ziplineblockwouldhavetotakeuponesetofspaces,leavingtheropebridgeandthevaultingapparatustobeconsecutiveintheothersetofspaces.ThisviolatesRule3andisthusunacceptable.So,(C),(D),and(E)canallbeeliminatedforincludingtheimpossiblepositionofthird.

Atthispoint,(B)istheonlyanswerleftandisthuscorrect.Fortherecord,thetunnelcannotbefourthbecausethatwouldmakethespinningplatformthird,leadingtothesameproblemasplacingthetunnelthird.Youcoulddrawonemoresketchtoprovethatthetunnelcouldbefifth,butthat'snotnecessaryasalloftheremainingchoiceshavebeeneliminated.

15. (B)“If”/MustBeTrueForthisquestion,theropebridgeissecond.Thatmeansthevaultingapparatuscannotbefirstorthird.Thefirstobstaclealsocannotbethespinningplatform,norcanitincludethewall/ziplineblock.Thatleavesthetunnelasthefirstobstacle.Thatmakes(B)thecorrectanswer.

Page 164: LSAT PrepTest 81 Unlocked · Similarly, PT 81 was the first test since June '07 (PT June '07) to require at least 94 questions correct to get a 172 (99th percentile score). Number

Theremaininganswersareeithercompletelyfalseorarepossible,butnotdefinitivelytrue.

16. (A)“If”/MustBeTrueForthisquestion,theropebridgeandthevaultingapparatusarebothearlierthanthetunnel.Theropebridgeandthevaultingapparatuscannotbenexttooneanother,sotheremustbeatleastoneotherobstaclebeforethetunnel.Thus,thetunnelcannotbefirst,second,orthird.Itmustbefourth,fifth,orsixth,leavingnoroomafteritforthespinningplatform.Thus,thespinningplatformmustalsobebeforethetunnel.

Thatleavesthewall/ziplineblock.Ifthatcameafterthetunnel,thetunnelwouldbefourth,makingthespinningplatformthird,whichwouldforcetheropebridgeandthevaultingapparatustobeconsecutive,violatingRule3.

So,thewall/ziplineblockmustalsobebeforethetunnel.That'severything,whichmeansthetunnelmustbelast.Next,considerthespinningplatform.Ifthespinningplatformwerethird,thatwouldcreate

Page 165: LSAT PrepTest 81 Unlocked · Similarly, PT 81 was the first test since June '07 (PT June '07) to require at least 94 questions correct to get a 172 (99th percentile score). Number

GAME4:MANAGERSINMANILA,SYDNEY,ANDTOKYO

twosetsofspaces:firstandsecond,andfourthandfifth.Thewall/ziplineblockwouldtakeoneset,butthatwouldagainforcetheropebridgeandthevaultingapparatustobeconsecutive.Thatcan'thappen,sothespinningplatformmustbefourth.Thatmakes(A)thecorrectanswer.Thewall/ziplineblockwilleithergofirst/secondorsecond/third.Oneoftheropebridgeorvaultingapparatuswillbefifthwiththeotheroneeitherfirstorthird.

(B),(C),(D),and(E),areeithercompletelyfalseorarepossible,butnotdefinitivelytrue.

Step1:Overview

Situation:Acompanysendingproductmanagerstovisitsomecities

Entities:Fourmanagers(Fan,Gleeson,Haley,Ibañez)andthreecities(Manila,Sydney,Tokyo)

Action:Matching.Determinewhichmanagersareassignedtoeachcity.

Limitations:Eachmanagerisassignedatleastonce,andtwomanagersareassignedtoeachcity.That'satotalofsixassignmentsforfourmanagers,soeitheronemanagergoestoallthreecitiesortwomanagersgototwocitieseach.

Page 166: LSAT PrepTest 81 Unlocked · Similarly, PT 81 was the first test since June '07 (PT June '07) to require at least 94 questions correct to get a 172 (99th percentile score). Number

Step2:Sketch

Listthemanagersbyinitialandsetupatablewiththethreecitiesascolumnheadings.Drawtwoslotsundereachcolumn.

Step3:Rules

Rule1setsupaNumericRestriction.Ibañezgoestoexactlytwocities.Drawasecond"I"intheentitylist.Youcouldalsomakeanotetotheside(e.g.,"Exactly2I's").

Rule2preventsFanandHaleyfromvisitingthesamecity.

Rule3providessomeFormalLogic.IfGleesongoestoManila,HaleygoestoTokyo.Bycontrapositive,ifHaleydoesnotgotoTokyo,thenGleesoncannotgotoManila.

Rule4preventsGleesonfromgoingtoSydney.Draw"~G"undertheSydneycolumn.

Step4:Deductions

Gleesonisduplicatedinthelasttworules.GleesoncannotvisitSydney,whichleavesManilaandTokyo.ThekeyquestionisifGleesonvisits

Page 167: LSAT PrepTest 81 Unlocked · Similarly, PT 81 was the first test since June '07 (PT June '07) to require at least 94 questions correct to get a 172 (99th percentile score). Number

Manila.IfGleesondoesvisitManila,thattriggerstheFormalLogicofRule3.IfGleesondoesnotvisitManila,thenshemustvisitTokyo.Eitherway,somevaluabledeductionscanbemade.ItisworthsettingupLimitedOptions.

Inthefirstoption,GleesonvisitsManila.Inthatcase,HaleyvistsTokyo.ThatmeansFancannotvisitTokyo(Rule2).ThatleavesoneofGleesonandIbañeztobethesecondmanagerinTokyo.NotethatinOptionI,GleesoncanstillvisitTokyo,asmanagerscanbesenttomultiplecities.

Inthesecondoption,GleesondoesnotvisitManila.Inthatcase,shemustvisitTokyo.

Atthispoint,Numbersbecomeimportant.Eachcitywillbevisitedbytwomanagers.Inbothoptions,there'satleastonecitythatcannotbevisitedbyGleeson(SydneyinOptionI,ManilaandSydneyinOptionII).Ineachcase,thatleavesFan,HaleyandIbañez.However,FanandHaleycannotbetogether(Rule2).So,eachofthosecitiescanonlygetoneofFanorHaley,andthesecondmanagermustbeIbañez.

There'sonefinalNumbersdeductiontonote.Ibañezmustvisitexactlytwocities.ThisaffectsOptionII,asIbañezisalreadyassignedtwiceand

Page 168: LSAT PrepTest 81 Unlocked · Similarly, PT 81 was the first test since June '07 (PT June '07) to require at least 94 questions correct to get a 172 (99th percentile score). Number

cannolongerbeassignedtoTokyo.So,thelastspotinTokyomustgotoFanorHaley.

Further,ifIbañezgoestotwocitiesandeachotheremployeegoestoone,thatwouldbeatotaloffiveassignments.However,therearesixslots,soexactlyoneotheremployeemustbeassignedtoasecondcity.

Step5:Questions

17. (C)AcceptabilityThisisatypicalAcceptabilityquestion.Testeachrule,oneatatime,andeliminateanswersthatviolatethoserules.

(A)violatesRule1byassigningIbañeztojustonecity.(E)violatesRule2byassigningFanandHaleytogethertoTokyo.(D)violatesRule3byassigningGleesontoManilawithoutassigningHaleytoTokyo.(B)violatesRule4byassigningGleesontoSydney.Thatleaves(C)asthecorrectanswer.

18. (B)CompletelyDetermineThecorrectanswerwillestablishaconditionthatwillallowallsixassignmentstobedeterminedwithnouncertainty.Eliminateanychoicethatallowsformorethanoneoutcome.

Page 169: LSAT PrepTest 81 Unlocked · Similarly, PT 81 was the first test since June '07 (PT June '07) to require at least 94 questions correct to get a 172 (99th percentile score). Number

Fancanvisitanypairofcities,aslongasHaleyvisitsthethird.Similarly,Haleycanvisitanypairofcities,aslongasFanvisitsthethird.However,ineithercase,it'snotcertainwhichcitiesarevisitedbywhom.Thateliminates(A)and(C).

IfGleesonvisitstwocities,theycouldonlybeManilaandTokyo(Rule4).WithGleesoninManila,HaleymustvisitTokyo(Rule3).WithTokyofilled,Ibañezstillneedstovisittwocities.TheymustbeManilaandSydney.ThatleavesoneslotinSydney,whichmustbetakenupbyFan,whohasnowhereelselefttogo.Theentireoutcomeisdetermined,making(B)thecorrectanswer.

Fortherecord:IfFanandGleesonvisitTokyo,IbañezwouldbeleftwithManilaandSydney.HaleycouldvisitManilaorSydney,orboth.Withmultiplepossibilities,thateliminates(D).

IfGleesonandHaleyvisitTokyo,IbañezwouldbeleftwithManilaandSydney.FancouldvisitManilaorSydney,orboth.Withmultiplepossibilities,thateliminates(E).

19. (D)MustBeTrueThecorrectanswerhastobetruenomatterwhat.Thewrongchoicescouldbefalseoraredefinitelyfalse.

Inbothoptions,GleesoncannotvisitSydney.FanandHaleycannotbothvisitSydney,soonlyoneofthemcan.ThesecondmanagervisitingSydneymustbeIbañez,making(D)thecorrectanswer.

Page 170: LSAT PrepTest 81 Unlocked · Similarly, PT 81 was the first test since June '07 (PT June '07) to require at least 94 questions correct to get a 172 (99th percentile score). Number

20. (A)CouldBeTrueThecorrectanswertothisquestionistheonlyonethatcouldbetrue.Theremainingchoiceswillallbeimpossible,i.e.,mustbefalse.

InOptionII,itispossibleforFanandIbañeztovisitManilatogether.Thatmakes(A)thecorrectanswer.Fortherecord:

GleesoncannotvisitSydney,soIbañezmustvisitSydneytopreventFanandHaleyfrombeingtogether.IfGleesonandIbañezvisitTokyo,thatwouldbeIbañez'ssecondcity.ThatwouldleaveFan,Gleeson,andHaleytovisitManila.However,withoutHaleyinTokyo,GleesoncannotvisitManila.AndwithoutGleeson,thatwouldleaveFanandHaleytogether,violatingRule2.Thisultimatelyisimpossible,whicheliminates(B).

IbañezhastovisitSydney,butcanonlyvisittwocities.Thus,IbañezcannotvisitManilaandTokyo,too.Thateliminates(C).

NeitherFannorHaleycanvisitthreecities.WithIbañezvisitingtwocities,thatwouldmeanFanorHaleyvisitsthreecities,Ibañezvisitstwo,andoneothermanagervisitsonecity.Somebodywouldbeleftout.Thateliminates(D)and(E).

21. (D)“If”/MustBeTrueForthisquestion,GleesonandHaleyvisitacitytogether.GleesoncannotvisitSydney,soitmustbeManilaorTokyo.

IfGleesonandHaleyvisitManilatogether,HaleymustalsovisitTokyo(Rule3).IbañezwouldthenbelefttovisitSydneyandTokyo.ThatwouldleaveonlySydneyforFan.

Page 171: LSAT PrepTest 81 Unlocked · Similarly, PT 81 was the first test since June '07 (PT June '07) to require at least 94 questions correct to get a 172 (99th percentile score). Number

IfGleesonandHaleyvisitTokyotogether,IbañezwouldbelefttovisitManilaandSydney.FancouldvisitManila,Sydney,orboth.

Ineithercase,HaleyvisitsTokyo,making(D)thecorrectanswer.Theremainingchoicesareallpossible,butneednotbetrue.

22. (A)“If”/CouldBeTrueForthisquestion,IbañezvisitsTokyo(whichcanonlyhappeninOptionI).Ifyouhaven'tmadeLimitedOptionsthoughandneedtostartfromscratch,IbañezalsomustvisitSydneybecauseGleesoncannotvisitSydneyandFanandHaleycannotvisitacitytogether.

WithIbañezdone,thatleavesManilaopentoFan,Gleeson,andHaley.Again,FanandHaleycannotvisittheretogether.So,onlyoneofthemcanvisitManila,andGleesonmustbethesecondmanager.WithGleesonvisitingManila,HaleymustvisitTokyo.

Withthat,only(A)ispossible,andisthusthecorrectanswer.IbañezcannotvisitManilainthiscase,andmustvisitSydneyandTokyo,eliminating(B)and(C).WithbothHaleyandIbañezinTokyo,there'sno

Page 172: LSAT PrepTest 81 Unlocked · Similarly, PT 81 was the first test since June '07 (PT June '07) to require at least 94 questions correct to get a 172 (99th percentile score). Number

roomforFan,whicheliminates(D).Asfor(E),HaleycouldgotoManilaorSydney,butifshewenttoboth,there'dbenocityleftforFantovisit,so(E)iseliminated.

23. (E)RuleSubstitutionForthisquestion,Rule2isremovedfromthesetup.ThecorrectanswerwillprovideanewconditionthatreplicatesalloftheeffectsofRule2(i.e.,splittingupFanandHaley)withoutaddinganynewrestrictions.

TheoriginalrestrictionsdidnotrequireGleesonandIbañeztobesplitup.Also,thatwouldnotkeepFanandHaleyapart.Thus,(A)iseliminated.

HaleywasneverrequiredtovisitTokyoifFanvisitsSydney.AndifHaleydidhavetovisitTokyointhatcase,itwouldn'tstopHaleyfromalsovisitingSydneywithFan.(B)doesnotworkandcanbeeliminated.

RestrictingFanandHaleyfrombeingtogetherinTokyowouldnotpreventthemfrombeingtogetherinothercities.Thateliminates(C).

(D)setsupsomecleverFormalLogic.IfFandoesnotgotoaparticularcity,thenHaleymust.However,thatwasnotalwaysthecase.ItwaspossibleintheoriginalforacitytonothaveFan,butalsonothaveHaley.Inthatcase,thecitycouldhaveGleesonandIbañez.ThisFormalLogicwouldberestrictiveinawaytheoriginalruleswerenot,whichmakes(D)incorrect.

(E)alsohassomecleverFormalLogic,butitworks.Bythisrule,acitywithoutIbañezwouldhavetobevisitedbyGleeson.Bycontrapositive,ifacitydidnothaveGleeson,itmusthaveIbañez.Inshort,ifoneofthemisn'tthere,theotheroneis,i.e.,eachcityhastobevisitedbyatleastone

Page 173: LSAT PrepTest 81 Unlocked · Similarly, PT 81 was the first test since June '07 (PT June '07) to require at least 94 questions correct to get a 172 (99th percentile score). Number

ofthem.It'spossibletohaveboth,butyoucan'tgetridofbothGleesonandIbañez.Bydoingthat,itpreventsFanandHaleyfrombeingtogether,establishingtheoriginalrule.Andthiswasalwaystruewiththeoriginalrule,becausesplittingupFanandHaleymadeitnecessarytoincludeGleesonorIbañez(orboth)ineachcity.Theoriginalconditionsarerestored,andnonewrestrictionsareadded.Thatmakes(E)thecorrectanswer.

Page 174: LSAT PrepTest 81 Unlocked · Similarly, PT 81 was the first test since June '07 (PT June '07) to require at least 94 questions correct to get a 172 (99th percentile score). Number

GLOSSARY

Page 175: LSAT PrepTest 81 Unlocked · Similarly, PT 81 was the first test since June '07 (PT June '07) to require at least 94 questions correct to get a 172 (99th percentile score). Number

Glossary

GLOSSARY

LogicalReasoningQuestionTypes

Argument-BasedQuestions

MainPointQuestionAquestionthatasksforanargument’sconclusionoranauthor’smainpoint.Typicalquestionstems:

Whichonethefollowingmostaccuratelyexpressestheconclusionof

theargumentasawhole?

Whichoneofthefollowingsentencesbestexpressesthemainpointof

thescientist’sargument?

RoleofaStatementQuestionAquestionthataskshowaspecificsentence,statement,orideafunctionswithinanargument.Typicalquestionstems:

Whichoneofthefollowingmostaccuratelydescribestheroleplayed

intheargumentbythestatementthatautomationwithinthesteel

industryallowedsteelmillstoproducemoresteelwithfewer

workers?

LOGICALREASONINGLOGICALREASONING

Page 176: LSAT PrepTest 81 Unlocked · Similarly, PT 81 was the first test since June '07 (PT June '07) to require at least 94 questions correct to get a 172 (99th percentile score). Number

Theclaimthatgovernmentaltransparencyisanation’sprimary

defenseagainstpublic-sectorcorruptionfiguresintheargumentin

whichoneofthefollowingways?

PointatIssueQuestionAquestionthatasksyoutoidentifythespecificclaim,statement,orrecommendationaboutwhichtwospeakers/authorsdisagree(or,rarely,aboutwhichtheyagree).Typicalquestionstems:

ApointatissuebetweenTomandJerryis

ThedialoguemoststronglysupportstheclaimthatMarilynandBilly

disagreewitheachotheraboutwhichoneofthefollowing?

MethodofArgumentQuestionAquestionthatasksyoutodescribeanauthor’sargumentativestrategy.Inotherwords,thecorrectanswerdescribeshowtheauthorargues(notnecessarilywhattheauthorsays).Typicalquestionstems:

Whichoneofthefollowingmostaccuratelydescribesthetechniqueof

reasoningemployedbytheargument?

Julian’sargumentproceedsby

Inthedialogue,AlexanderrespondstoAbigailinwhichoneofthe

followingways?

ParallelReasoningQuestionAquestionthatasksyoutoidentifytheanswerchoicecontaininganargumentthathasthesamelogicalstructureandreachesthesametypeofconclusionastheargumentinthestimulusdoes.Typicalquestionstems:

Thepatternofreasoninginwhichoneofthefollowingargumentsis

mostparalleltothatintheargumentabove?

Page 177: LSAT PrepTest 81 Unlocked · Similarly, PT 81 was the first test since June '07 (PT June '07) to require at least 94 questions correct to get a 172 (99th percentile score). Number

Thepatternofreasoninginwhichoneofthefollowingargumentsis

mostsimilartothepatternofreasoningintheargumentabove?

Assumption-FamilyQuestions

AssumptionQuestionAquestionthatasksyoutoidentifyoneoftheunstatedpremisesinanauthor’sargument.Assumptionquestionscomeintwovarieties.

NecessaryAssumptionquestionsaskyoutoidentifyanunstatedpremiserequiredforanargument’sconclusiontofollowlogicallyfromitsevidence.Typicalquestionstems:

Whichoneofthefollowingisanassumptiononwhichtheargument

depends?

Whichoneofthefollowingisanassumptionthattheargument

requiresinorderforitsconclusiontobeproperlydrawn?

SufficientAssumptionquestionsaskyoutoidentifyanunstatedpremisesufficienttoestablishtheargument’sconclusiononthebasisofitsevidence.Typicalquestionstems:

Theconclusionfollowslogicallyifwhichoneofthefollowingis

assumed?

Whichoneofthefollowing,ifassumed,enablestheconclusionabove

tobeproperlyinferred?

Strengthen/WeakenQuestionAquestionthatasksyoutoidentifyafactthat,iftrue,wouldmaketheargument’sconclusionmorelikely(Strengthen)orlesslikely(Weaken)tofollowfromitsevidence.Typicalquestionstems:

Strengthen

Page 178: LSAT PrepTest 81 Unlocked · Similarly, PT 81 was the first test since June '07 (PT June '07) to require at least 94 questions correct to get a 172 (99th percentile score). Number

Whichoneofthefollowing,iftrue,moststrengthenstheargument

above?

Whichonethefollowing,iftrue,moststronglysupportstheclaim

above?

Weaken

Whichoneofthefollowing,iftrue,wouldmostweakentheargument

above?

Whichoneofthefollowing,iftrue,mostcallsintoquestiontheclaim

above?

FlawQuestionAquestionthatasksyoutodescribethereasoningerrorthattheauthorhasmadeinanargument.Typicalquestionstems:

Theargument’sreasoningismostvulnerabletocriticismonthe

groundsthattheargument

Whichofthefollowingidentifiesareasoningerrorintheargument?

Thereasoninginthecorrespondent’sargumentisquestionable

becausetheargument

ParallelFlawQuestionAquestionthatasksyoutoidentifytheargumentthatcontainsthesameerror(s)inreasoningthattheargumentinthestimuluscontains.Typicalquestionstems:

Thepatternofflawedreasoningexhibitedbytheargumentaboveis

mostsimilartothatexhibitedinwhichoneofthefollowing?

Whichoneofthefollowingmostcloselyparallelsthequestionable

reasoningcitedabove?

Page 179: LSAT PrepTest 81 Unlocked · Similarly, PT 81 was the first test since June '07 (PT June '07) to require at least 94 questions correct to get a 172 (99th percentile score). Number

EvaluatetheArgumentQuestionAquestionthatasksyoutoidentifyanissueorconsiderationrelevanttothevalidityofanargument.ThinkofEvaluatequestionsas“StrengthenorWeaken”questions.Thecorrectanswer,iftrue,willstrengthentheargument,andiffalse,willweakentheargument,orviceversa.Evaluatequestionsareveryrare.Typicalquestionstems:

Whichoneofthefollowingwouldbemostusefultoknowinorderto

evaluatethelegitimacyoftheprofessor’sargument?

Itwouldbemostimportanttodeterminewhichoneofthefollowing

inevaluatingtheargument?

Non-ArgumentQuestions

InferenceQuestionAquestionthatasksyoutoidentifyastatementthatfollowsfromthestatementsinthestimulus.ItisveryimportanttonotethecharacteristicsoftheonecorrectandthefourincorrectanswersbeforeevaluatingthechoicesinInferencequestions.Dependingonthewordingofthequestionstem,thecorrectanswertoanInferencequestionmaybetheonethat

-mustbetrueifthestatementsinthestimulusaretrue

-ismoststronglysupportedbythestatementsinthestimulus

-mustbefalseifthestatementsinthestimulusaretrue

Typicalquestionstems:

Ifallofthestatementsabovearetrue,thenwhichoneofthefollowing

mustalsobetrue?

Whichoneofthefollowingcanbeproperlyinferredfromthe

informationabove?

Page 180: LSAT PrepTest 81 Unlocked · Similarly, PT 81 was the first test since June '07 (PT June '07) to require at least 94 questions correct to get a 172 (99th percentile score). Number

Ifthestatementsabovearetrue,theneachofthefollowingcouldbe

trueEXCEPT:

Whichoneofthefollowingismoststronglysupportedbythe

informationabove?

Thestatementsabove,iftrue,mostsupportwhichoneofthe

following?

Thefactsdescribedaboveprovidethestrongestevidenceagainst

whichoneofthefollowing?

ParadoxQuestionAquestionthatasksyoutoidentifyafactthat,iftrue,mosthelpstoexplain,resolve,orreconcileanapparentcontradiction.Typicalquestionstems:

Whichoneofthefollowing,iftrue,mosthelpstoexplainhowboth

studies’findingscouldbeaccurate?

Whichonethefollowing,iftrue,mosthelpstoresolvetheapparent

conflictinthespokesperson’sstatements?

Eachoneofthefollowing,iftrue,wouldcontributetoanexplanation

oftheapparentdiscrepancyintheinformationaboveEXCEPT:

PrincipleQuestions

PrincipleQuestionAquestionthatasksyoutoidentifycorrespondingcasesandprinciples.SomePrinciplequestionsprovideaprincipleinthestimulusandcallfortheanswerchoicedescribingacasethatcorrespondstotheprinciple.Othersprovideaspecificcaseinthestimulusandcallfortheanswercontainingaprincipletowhichthatcasecorresponds.

Page 181: LSAT PrepTest 81 Unlocked · Similarly, PT 81 was the first test since June '07 (PT June '07) to require at least 94 questions correct to get a 172 (99th percentile score). Number

OntheLSAT,PrinciplequestionsalmostalwaysmirrortheskillsrewardedbyotherLogicalReasoningquestiontypes.AftereachofthefollowingPrinciplequestionstems,wenotethequestiontypeitresembles.Typicalquestionstems:

Whichoneofthefollowingprinciples,ifvalid,mosthelpstojustify

thereasoningabove?(Strengthen)

Whichoneofthefollowingmostaccuratelyexpressestheprinciple

underlyingthereasoningabove?(Assumption)

Thesituationdescribedabovemostcloselyconformstowhichofthe

followinggeneralizations?(Inference)

Whichoneofthefollowingsituationsconformsmostcloselytothe

principledescribedabove?(Inference)

Whichoneofthefollowingprinciples,ifvalid,mosthelpstoreconcile

theapparentconflictamongtheprosecutor’sclaims?(Paradox)

ParallelPrincipleQuestionAquestionthatasksyoutoidentifyaspecificcasethatillustratesthesameprinciplethatisillustratedbythecasedescribedinthestimulus.Typicalquestionstem:

Ofthefollowing,whichoneillustratesaprinciplethatismostsimilar

totheprincipleillustratedbythepassage?

UntanglingtheStimulus

ConclusionTypesTheconclusionsinargumentsfoundintheLogicalReasoningsectionoftheLSATtendtofallintooneofsixcategories:

Page 182: LSAT PrepTest 81 Unlocked · Similarly, PT 81 was the first test since June '07 (PT June '07) to require at least 94 questions correct to get a 172 (99th percentile score). Number

1)ValueJudgment(anevaluativestatement;e.g.,ActionXisunethical,orY’srecitalwaspoorlysung)

2)“If”/Then(aconditionalprediction,recommendation,orassertion;e.g.,IfXistrue,thensoisY,orIfyouanM,thenyoushoulddoN)

3)Prediction(Xwillorwillnothappeninthefuture)

4)Comparison(Xistaller/shorter/morecommon/lesscommon,etc.thanY)

5)AssertionofFact(XistrueorXisfalse)

6)Recommendation(weshouldorshouldnotdoX)

One-SentenceTestAtacticusedtoidentifytheauthor’sconclusioninanargument.Considerwhichsentenceintheargumentistheonetheauthorwouldkeepifaskedtogetridofeverythingexcepthermainpoint.

SubsidiaryConclusionAconclusionfollowingfromonepieceofevidenceandthenusedbytheauthortosupporthisoverallconclusionormainpoint.Considerthefollowingargument:

Thepharmaceuticalcompany’snewexperimentaltreatmentdidnot

succeedinclinicaltrials.Asaresult,thenewtreatmentwillnotreach

themarketthisyear.Thus,thecompanywillfallshortofitsrevenue

forecastsfortheyear.

Here,thesentence“Asaresult,thenewtreatmentwillnotreachthemarketthisyear”isasubsidiaryconclusion.Itfollowsfromtheevidencethatthenewtreatmentfailedinclinicaltrials,anditprovidesevidencefortheoverallconclusionthatthecompanywillnotmeetitsrevenueprojections.

Keyword(s)inLogicalReasoning

Page 183: LSAT PrepTest 81 Unlocked · Similarly, PT 81 was the first test since June '07 (PT June '07) to require at least 94 questions correct to get a 172 (99th percentile score). Number

Awordorphrasethathelpsyouuntangleaquestion’sstimulusbyindicatingthelogicalstructureoftheargumentortheauthor’spoint.HerearethreecategoriesofKeywordstowhichLSATexpertspayspecialattentioninLogicalReasoning:

Conclusionwords;e.g.,therefore,thus,so,asaresult,itfollowsthat,consequently,[evidence]isevidencethat[conclusion]

Evidenceword;e.g,because,since,afterall,for,[evidence]isevidencethat[conclusion]

Contrastwords;e.g.,but,however,while,despite,inspiteof,ontheotherhand(TheseareespeciallyusefulinParadoxandInferencequestions.)

ExpertsuseKeywordsevenmoreextensivelyinReadingComprehension.LearntheKeywordsassociatedwiththeReadingComprehensionsection,andapplythemtoLogicalReasoningwhentheyarehelpful.

MismatchedConceptsOneoftwopatternstowhichauthors’assumptionsconforminLSATarguments.MismatchedConceptsdescribestheassumptioninargumentsinwhichtermsorconceptsintheconclusionaredifferentinkindfromthoseintheevidence.Theauthorassumesthatthereisalogicalrelationshipbetweenthedifferentterms.Forexample:

Bobbyisachampionshipswimmer.Therefore,hetrainsevery

day.

Here,thewords“trainseveryday”appearonlyintheconclusion,andthewords“championshipswimmer”appearonlyintheevidence.Fortheauthortoreachthisconclusionfromthisevidence,heassumesthatchampionshipswimmerstraineveryday.

Anotherexample:

Page 184: LSAT PrepTest 81 Unlocked · Similarly, PT 81 was the first test since June '07 (PT June '07) to require at least 94 questions correct to get a 172 (99th percentile score). Number

Susandoesnoteathervegetables.Thus,shewillnotgrowbig

andstrong.

Inthisargument,notgrowingbigandstrongisfoundonlyintheconclusionwhilenoteatingvegetablesisfoundonlyintheevidence.Fortheauthortoreachthisconclusionfromthisevidence,shemustassumethateatingone’svegetablesisnecessaryforonetogrowbigandstrong.

SeealsoOverlookedPossibilities.

OverlookedPossibilitiesOneoftwopatternstowhichauthors’assumptionsconforminLSATarguments.MismatchedConceptsdescribestheassumptioninargumentsinwhichtermsorconceptsintheconclusionaredifferentindegree,scale,orlevelofcertaintyfromthoseintheevidence.Theauthorassumesthatthereisnofactororexplanationfortheconclusionotherthantheone(s)offeredintheevidence.Forexample:

Samsondoesnothaveaticketstubforthismovieshowing.Thus,

Samsonmusthavesneakedintothemoviewithoutpaying.

TheauthorassumesthatthereisnootherexplanationforSamson’slackofaticketstub.Theauthoroverlooksseveralpossibilities:e.g.,Samsonhadaspecialpassforthisshowingofthemovie;Samsondroppedhisticketstubbyaccidentorthrewitawayafterenteringthetheater;someoneelseinSamson’spartyhasallofthepartymembers’ticketstubsinherpocketorhandbag.

Anotherexample:

Jonah’smarketingplanwillsavethecompanymoney.Therefore,the

companyshouldadoptJonah’splan.

Page 185: LSAT PrepTest 81 Unlocked · Similarly, PT 81 was the first test since June '07 (PT June '07) to require at least 94 questions correct to get a 172 (99th percentile score). Number

Here,theauthormakesarecommendationbasedononeadvantage.Theauthorassumesthattheadvantageisthecompany’sonlyconcernorthattherearenodisadvantagesthatcouldoutweighit,e.g.,Jonah’splanmightsavemoneyonmarketingbutnotgenerateanynewleadsorcustomers;Jonah’splanmightdamagethecompany’simageorreputation;Jonah’splanmightincludeillegalfalseadvertising.WhenevertheauthorofanLSATargumentconcludeswitharecommendationorapredictionbasedonjustasinglefactintheevidence,thatauthorisalwaysoverlookingmanyotherpossibilities.

SeealsoMismatchedConcepts.

CausalArgumentAnargumentinwhichtheauthorconcludesorassumesthatonethingcausesanother.ThemostcommonpatternontheLSATisfortheauthortoconcludethatAcausesBfromevidencethatAandBarecorrelated.Forexample:

Inoticethatwheneverthestorehasapoorsalesmonth,employee

tardinessisalsohigherthatmonth.Therefore,itmustbethat

employeetardinesscausesthestoretolosesales.

Theauthorassumesthatthecorrelationintheevidenceindicatesacausalrelationship.Theseargumentsarevulnerabletothreetypesofoverlookedpossibilities:

1)Therecouldbeanothercausalfactor.Inthepreviousexample,maybethemonthsinquestionarethoseinwhichthemanagertakesvacation,causingthestoretolosesalesandpermittingemployeestoarrivelatewithoutfearoftheboss’sreprimands.

2)Causationcouldbereversed.Maybeinmonthswhensalesaredown,employeemoralesuffersandtardinessincreasesasaresult.

Page 186: LSAT PrepTest 81 Unlocked · Similarly, PT 81 was the first test since June '07 (PT June '07) to require at least 94 questions correct to get a 172 (99th percentile score). Number

3)Thecorrelationcouldbecoincidental.Maybethecorrelationbetweentardinessandthedipinsalesispurecoincidence.

SeealsoFlawTypes:CorrelationversusCausation.

Anotherpatternincausalarguments(lessfrequentontheLSAT)involvestheassumptionthataparticularcausalmechanismisorisnotinvolvedinacausalrelationship.Forexample:

Theairporthasreroutedtakeoffsandlandingssothattheywillnot

createnoiseovertheSunnysideneighborhood.Thus,therecentdrop

inSunnyside’spropertyvaluescannotbeexplainedbythe

neighborhood’sproximitytotheairport.

Here,theauthorassumesthattheonlywaythattheairportcouldbethecauseofdroppingpropertyvaluesisthroughnoisepollution.Theauthoroverlooksanyotherpossiblemechanism(e.g.,frequenttrafficjamsandcongestion)throughwhichproximitytotheairportcouldbecauseofSunnyside’swoes.

PrincipleAbroad,law-likerule,definition,orgeneralizationthatcoversavarietyofspecificcaseswithdefinedattributes.ToseehowprinciplesaretreatedontheLSAT,considerthefollowingprinciple:

Itisimmoralforapersonforhisowngaintomisleadanotherperson.

Thatprinciplewouldcoveraspecificcase,suchasasellerwholiesaboutthequalityofconstructiontogetahigherpriceforhishouse.Itwouldalsocorrespondtothecaseofateenagerwho,wishingtospendanightoutonthetown,tellshismom“I’mgoingovertoRandy’shouse.”HeknowsthathismombelievesthathewillbestayingatRandy’shouse,wheninfact,heandRandywillgoouttogether.

Page 187: LSAT PrepTest 81 Unlocked · Similarly, PT 81 was the first test since June '07 (PT June '07) to require at least 94 questions correct to get a 172 (99th percentile score). Number

Thatprincipledoesnot,however,covercasesinwhichsomeoneliessolelyforthepurposeofmakingtheotherpersonfeelbetterorinwhichonepersoninadvertentlymisleadstheotherthroughamistakeoffact.

Becarefulnottoapplyyourpersonalethicsormoralswhenanalyzingtheprinciplesarticulatedonthetest.

FlawTypes

NecessaryversusSufficientThisflawoccurswhenaspeakerorauthorconcludesthatoneeventisnecessaryforasecondeventfromevidencethatthefirsteventissufficienttobringaboutthesecondevent,orviceversa.Example:

Ifmorethan25,000usersattempttoaccessthenewappatthesame

time,theserverwillcrash.Lastnight,at11:15PM,theservercrashed,

soitmustbecasethatmorethan25,000userswereattemptingtouse

thenewappatthattime.

Inmakingthisargument,theauthorassumesthattheonlythingthatwillcausetheservertocrashistheusagelevel(i.e.,highusageisnecessaryfortheservertocrash).Theevidence,however,saysthathighusageisonethingthatwillcausetheservertocrash(i.e.,thathighusageissufficienttocrashtheserver).

CorrelationversusCausationThisflawoccurswhenaspeakerorauthordrawsaconclusionthatonethingcausesanotherfromevidencethatthetwothingsarecorrelated.Example:

Overthepasthalfcentury,globalsugarconsumptionhastripled.That

sametimeperiodhasseenasurgeintherateoftechnological

advancementworldwide.Itfollowsthattheincreaseinsugar

Page 188: LSAT PrepTest 81 Unlocked · Similarly, PT 81 was the first test since June '07 (PT June '07) to require at least 94 questions correct to get a 172 (99th percentile score). Number

consumptionhascausedtheaccelerationintechnological

advancement.

Inanyargumentwiththisstructure,theauthorismakingthreeunwarrantedassumptions.First,heassumesthatthereisnoalternatecause,i.e.,thereisnothingelsethathascontributedtorapidtechnologicaladvancement.Second,heassumesthatthecausationisnotreversed,i.e.,technologicaladvancementhasnotcontributedtotheincreaseinsugarconsumption,perhapsbymakingiteasiertogrow,refine,ortransportsugar.And,third,heassumesthatthetwophenomenaarenotmerelycoincidental,i.e.,thatitisnotjusthappenstancethatglobalsugarconsumptionisupatthesametimethatthepaceoftechnologicaladvancementhasaccelerated.

UnrepresentativeSampleThisflawoccurswhenaspeakerorauthordrawsaconclusionaboutagroupfromevidenceinwhichthesamplecannotrepresentthatgroupbecausethesampleistoosmallortooselective,orisbiasedinsomeway.Example:

Moviegoersinourtownpreferactionfilmsandromanticcomedies

overotherfilmgenres.LastFriday,wesentreporterstosurvey

moviegoersatseveraltheatersintown,andnearly90percentofthose

surveyedweregoingtowatcheitheranactionfilmoraromantic

comedy.

Theauthorassumesthatthesurveywasrepresentativeofthetown’smoviegoers,butthereareseveralreasonstoquestionthatassumption.First,wedon’tknowhowmanypeoplewereactuallysurveyed.Evenifthenumberofpeoplesurveyedwasadequate,wedon’tknowhowmanyothertypesofmovieswereplaying.Finally,theauthordoesn’tlimitherconclusiontomoviegoersonFridaynights.IfthesurveyhadbeenconductedatSundaymatinees,maybemostmoviegoerswouldhavebeen

Page 189: LSAT PrepTest 81 Unlocked · Similarly, PT 81 was the first test since June '07 (PT June '07) to require at least 94 questions correct to get a 172 (99th percentile score). Number

headingouttoseeananimatedfamilyfilmorahistoricaldrama.Whoknows?

ScopeShift/UnwarrantedAssumptionThisflawoccurswhenaspeaker’sorauthor’sevidencehasascopeorhastermsdifferentenoughfromthescopeortermsinhisconclusionthatitisdoubtfulthattheevidencecansupporttheconclusion.Example:

Averysmallpercentageofworkingadultsinthiscountrycan

correctlydefinecollateralizeddebtobligationsecurities.Thus,sadto

say,themajorityofthenation’sworkingadultscannotmakeprudent

choicesabouthowtoinvesttheirsavings.

Thisspeakerassumesthatprudentinvestingrequirestheabilitytoaccuratelydefineasomewhatobscurefinancialterm.Butprudenceisnotthesamethingasexpertise,andthespeakerdoesnotofferanyevidencethatthisknowledgeofthisparticulartermisrelatedtowiseinvesting.

PercentversusNumber/RateversusNumberThisflawoccurswhenaspeakerorauthordrawsaconclusionaboutrealquantitiesfromevidenceaboutratesorpercentages,orviceversa.Example:

Attheendoflastseason,CampSunnyDaylaidoffhalfoftheirsenior

counselorsandaquarteroftheirjuniorcounselors.Thus,Camp

SunnyDaymusthavemoreseniorcounselorsthanjuniorcounselors.

Theproblem,ofcourse,isthatwedon’tknowhowmanyseniorandjuniorcounselorswereonstaffbeforethelayoffs.Iftherewereatotalof4seniorcounselorsand20juniorcounselors,thenthecampwouldhavelaidoffonly2seniorcounselorswhiledismissing5juniorcounselors.

Equivocation

Page 190: LSAT PrepTest 81 Unlocked · Similarly, PT 81 was the first test since June '07 (PT June '07) to require at least 94 questions correct to get a 172 (99th percentile score). Number

Thisflawoccurswhenaspeakerorauthorusesthesamewordintwodifferentandincompatibleways.Example:

Ouropponentintheracehasaccusedourcandidate’sstaffmembers

ofbehavingunprofessionally.Butthat’snotfair.Ourstaffismadeup

entirelyofvolunteers,notpaidcampaignworkers.

Thespeakerinterpretstheopponent’suseofthewordprofessionaltomean“paid,”buttheopponentlikelymeantsomethingmorealongthelinesof“mature,competent,andbusinesslike.”

AdHominemThisflawoccurswhenaspeakerorauthorconcludesthatanotherperson’sclaimorargumentisinvalidbecausethatotherpersonhasapersonalflaworshortcoming.Onecommonpatternisforthespeakerorauthortoclaimtheotherpersonactshypocriticallyorthattheotherperson’sclaimismadefromself-interest.Example:

Mrs.Smitherstestifiedbeforethecitycouncil,statingthatthespeed

limitsontheresidentialstreetsnearherhomearedangerouslyhigh.

Butwhyshouldwegiveherclaimanycredence?Thewaysheeatsand

exercises,she’snotevenlookingoutforherownhealth.

TheauthorattemptstoundermineMrs.Smithers’stestimonybyattackinghercharacterandhabits.Hedoesn’tofferanyevidencethatisrelevanttoherclaimaboutspeedlimits.

Page 191: LSAT PrepTest 81 Unlocked · Similarly, PT 81 was the first test since June '07 (PT June '07) to require at least 94 questions correct to get a 172 (99th percentile score). Number

PartversusWholeThisflawoccurswhenaspeakerorauthorconcludesthatapartorindividualhasacertaincharacteristicbecausethewholeorthelargergrouphasthatcharacteristic,orviceversa.Example:

Patient:Ishouldhavenoproblemstakingthethreedrugsprescribed

tomebymydoctors.Ilookedthemup,andnoneofthethreeislisted

ashavinganymajorsideeffects.

Here,thepatientisassumingthatwhatistrueofeachofthedrugsindividuallywillbetrueofthemwhentakentogether.Thepatient’sflawisoverlookingpossibleinteractionsthatcouldcauseproblemsnotpresentwhenthedrugsaretakenseparately.

CircularReasoningThisflawoccurswhenaspeakerorauthortriestoproveaconclusionwithevidencethatislogicallyequivalenttotheconclusion.Example:

Allthosewhorunforofficeareprevaricators.Toseethis,just

considerpoliticians:theyallprevaricate.

Perhapstheauthorhastriedtodisguisethecircularreasoninginthisargumentbyexchangingthewords“thosewhorunforoffice”intheconclusionfor“politicians”intheevidence,butallthisargumentamountstois“Politiciansprevaricate;therefore,politiciansprevaricate.”OntheLSAT,circularreasoningisveryrarelythecorrectanswertoaFlawquestion,althoughitisregularlydescribedinoneofthewronganswers.

QuestionStrategies

DenialTest

Page 192: LSAT PrepTest 81 Unlocked · Similarly, PT 81 was the first test since June '07 (PT June '07) to require at least 94 questions correct to get a 172 (99th percentile score). Number

Atacticforidentifyingtheassumptionnecessarytoanargument.Whenyounegateanassumptionnecessarytoanargument,theargumentwillfallapart.Negatinganassumptionthatisnotnecessarytotheargumentwillnotinvalidatetheargument.Considerthefollowingargument:

Onlyhighschoolswhichproducedastatechampionathleteduring

theschoolyearwillberepresentedattheGovernor’sawardsbanquet.

Therefore,McMurtryHighSchoolwillberepresentedatthe

Governor’sawardsbanquet.

Whichoneofthefollowingisanassumptionnecessarytothatargument?

(1)McMurtryHighSchoolproducedmorestatechampionathletes

thananyotherhighschoolduringtheschoolyear.

(2)McMurtryHighSchoolproducedatleastonestatechampion

athleteduringtheschoolyear.

Ifyouareatallconfusedaboutwhichofthosetwostatementsreflectsthenecessaryassumption,negatethemboth.

(1)McMurtryHighSchooldidnotproducemorestatechampion

athletesthananyotherhighschoolduringtheschoolyear.

Thatdoesnotinvalidatetheargument.McMurtrycouldstillberepresentedattheGovernor’sbanquet.

(2)McMurtryHighSchooldidnotproduceanystatechampion

athletesduringtheschoolyear.

Here,negatingthestatementcausestheargumenttofallapart.Statement(2)isanassumptionnecessarytotheargument.

PointatIssue“DecisionTree”AtacticforevaluatingtheanswerchoicesinPointatIssuequestions.Thecorrectansweristheonlyanswerchoicetowhichyoucananswer“Yes”to

Page 193: LSAT PrepTest 81 Unlocked · Similarly, PT 81 was the first test since June '07 (PT June '07) to require at least 94 questions correct to get a 172 (99th percentile score). Number

allthreequestionsinthefollowingdiagram.

Doesspeaker#1haveanopinionaboutthisstatement?

Doesspeaker#2haveanopinion?

NO

Answeriswrong.

Dothespeakersdisagree?

YES

NOYES

NOYES

Answeriswrong.

CORRECT! Answeriswrong.

CommonMethodsofArgumentThesemethodsofargumentorargumentativestrategiesarecommonontheLSAT:

Analogy,inwhichanauthordrawsparallelsbetweentwounrelated(butpurportedlysimilar)situationsExample,inwhichanauthorcitesaspecificcaseorcasestojustifya

Page 194: LSAT PrepTest 81 Unlocked · Similarly, PT 81 was the first test since June '07 (PT June '07) to require at least 94 questions correct to get a 172 (99th percentile score). Number

WrongAnswerTypesinLR

OutsidetheScope(OutofScope;BeyondtheScope)Ananswerchoicecontainingastatementthatistoobroad,toonarrow,orbeyondthepurviewofthestimulus,makingthestatementinthechoiceirrelevant

180Ananswerchoicethatdirectlycontradictswhatthecorrectanswermustsay(forexample,achoicethatstrengthenstheargumentinaWeakenquestion)

ExtremeAnanswerchoicecontaininglanguagetooemphatictobesupportedbythestimulus;often(althoughnotalways)characterizedbywordssuchasall,never,every,only,ormost

Distortion

generalizationCounterexample,inwhichanauthorseekstodiscreditanopponent’sargumentbycitingaspecificcaseorcasesthatappeartoinvalidatetheopponent’sgeneralizationAppealtoauthority,inwhichanauthorcitesanexpert’sclaimoropinionassupportforherconclusionAdhominemattack,inwhichanauthorattacksheropponent’spersonalcredibilityratherthanattackingthesubstanceofheropponent’sargumentEliminationofalternatives,inwhichanauthorlistspossibilitiesanddiscreditsorrulesoutallbutoneMeans/requirements,inwhichtheauthorarguesthatsomethingisneededtoachieveadesiredresult

Page 195: LSAT PrepTest 81 Unlocked · Similarly, PT 81 was the first test since June '07 (PT June '07) to require at least 94 questions correct to get a 172 (99th percentile score). Number

Ananswerchoicethatmentionsdetailsfromthestimulusbutmanglesormisstateswhattheauthorsaidaboutthosedetails

IrrelevantComparisonAnanswerchoicethatcomparestwoitemsorattributesinawaynotgermanetotheauthor’sargumentorstatements

Half-Right/Half-WrongAnanswerchoicethatbeginscorrectly,butthencontradictsordistortsthepassageinitssecondpart;thiswronganswertypeismorecommoninReadingComprehensionthanitisinLogicalReasoning

FaultyUseofDetailAnanswerchoicethataccuratelystatessomethingfromthestimulus,butdoessoinamannerthatanswersthequestionincorrectly;thiswronganswertypeismorecommoninReadingComprehensionthanitisinLogicalReasoning

LOGICGAMES

GameTypes

StrictSequencingGameAgamethatasksyoutoarrangeentitiesintonumberedpositionsorintoasetschedule(usuallyhoursordays).StrictSequencingis,byfar,themostcommongametypeontheLSAT.InthetypicalStrictSequencinggame,thereisaone-to-onematchupofentitiesandpositions,e.g.,sevenentitiestobeplacedinsevenpositions,oneperposition,orsixentitiestobeplacedoversixconsecutivedays,oneentityperday.

Page 196: LSAT PrepTest 81 Unlocked · Similarly, PT 81 was the first test since June '07 (PT June '07) to require at least 94 questions correct to get a 172 (99th percentile score). Number

Fromtimetotime,theLSATwillofferStrictSequencingwithmoreentitiesthanpositions(e.g.,sevenentitiestobearrangedoverfivedays,withsomedaystoreceivemorethanoneentity)ormorepositionsthanentities(e.g.,sixentitiestobescheduledoversevendays,withatleastonedaytoreceivenoentities).

Other,lesscommonvariationsonStrictSequencinginclude:

DoubleSequencing,inwhicheachentityisplacedorscheduledtwotimes(therehavebeenrareoccurrencesofTripleorQuadrupleSequencing).Alternatively,aDoubleSequencinggamemayinvolvetwodifferentsetsofentitieseachsequencedonce.

CircularSequencing,inwhichentitiesarearrangedaroundatableorinacirculararrangement(NOTE:WhenthepositionsinaCircularSequencinggamearenumbered,thefirstandlastpositionsareadjacent.)

VerticalSequencing,inwhichthepositionsarenumberedfromtoptobottomorfrombottomtotop(asinthefloorsofabuilding)

LooseSequencingGameAgamethatasksyoutoarrangeorscheduleentitiesinorderbutprovidesnonumberingornamingofthepositions.TherulesinLooseSequencinggiveonlytherelativepositions(earlierorlater,higherorlower)betweentwoentitiesoramongthreeentities.LooseSequencinggamesalmostalwaysprovidethattherewillbenotiesbetweenentitiesintherank,order,orpositiontheytake.

CircularSequencingGameSeeStrictSequencingGame.

SelectionGame

Page 197: LSAT PrepTest 81 Unlocked · Similarly, PT 81 was the first test since June '07 (PT June '07) to require at least 94 questions correct to get a 172 (99th percentile score). Number

Agamethatasksyoutochooseorincludesomeentitiesfromtheinitiallistofentitiesandtorejectorexcludeothers.SomeSelectiongamesprovideoveralllimitationsonthenumberofentitiestobeselected(e.g.,“chooseexactlyfourofsevenstudents”or“chooseatleasttwoofsixentrees”)whileothersprovidelittleornorestrictiononthenumberselected(“chooseatleastonetypeofflower”or“selectfromamongsevenboardmembers”).

DistributionGameAgamethatasksyoutobreakuptheinitiallistofentitiesintotwo,three,or(veryrarely)fourgroupsorteams.InthevastmajorityofDistributiongames,eachentityisassignedtooneandonlyonegrouporteam.ArelativelycommonvariationonDistributiongameswillprovideasubdividedlistofentities(e.g.,eightstudents—fourmenandfourwomen—willformthreestudygroups)andwillthenrequirerepresentativesfromthosesubdivisionsoneachteam(e.g.,eachstudygroupwillhaveatleastoneofthemenonit).

MatchingGameAgamethatasksyoutomatchoneormoremembersofonesetofentitiestospecificmembersofanothersetofentities,orthatasksyoutomatchattributesorobjectstoasetofentities.UnlikeDistributiongames,inwhicheachentityisplacedinexactlyonegrouporteam,Matchinggamesusuallypermityoutoassignthesameattributeorobjecttomorethanoneentity.

Insomecases,thereareoveralllimitationsonthenumberofentitiesthatcanbematched(e.g.,“Inaschool’swoodshop,therearefourworkstations—numbered1through4—andeachworkstationhasatleastoneandatmostthreeofthefollowingtools—bandsaw,dremmeltool,electricsander,andpowerdrill”).InalmostallMatchinggames,furtherrestrictionsonthenumberofentitiesthatcanbematchedtoaparticularpersonorplacewillbefoundintherules(e.g.,Workstation4willhavemoretoolsthanWorkstation2has).

Page 198: LSAT PrepTest 81 Unlocked · Similarly, PT 81 was the first test since June '07 (PT June '07) to require at least 94 questions correct to get a 172 (99th percentile score). Number

HybridGameAgamethatasksyoutodotwo(orrarely,three)ofthestandardactions(Sequencing,Selection,Distribution,andMatching)toasetofentities.

ThemostcommonHybridisSequencing-Matching.AtypicalSequencing-MatchingHybridgamemightaskyoutoschedulesixspeakersataconferencetosixone-hourspeakingslots(from9AMto2PM),andthenassigneachspeakeroneoftwosubjects(economicdevelopmentortradepolicy).

NearlyascommonasSequencing-MatchingisDistribution-Sequencing.AtypicalgameofthistypemightaskyoutodividesixpeopleinatalentcompetitionintoeitheraDancecategoryoraSingingcategory,andthenrankthecompetitorsineachcategory.

ItismostcommontoseeoneHybridgameineachLogicGamessection,althoughtherehavebeentestswithtwoHybridgamesandtestswithnone.TodeterminethetypeofHybridyouarefacedwith,identifythegame’sactioninStep1oftheLogicGamesMethod.Forexample,agameaskingyoutochoosefourofsixrunners,andthenassignthefourchosenrunnerstolanesnumbered1through4onatrack,wouldbeaSelection-SequencingHybridgame.

MappingGameAgamethatprovidesyouwithadescriptionofgeographicallocationsand,typically,oftheconnectionsamongthem.Mappinggamesoftenaskyoutodeterminetheshortestpossibleroutesbetweentwolocationsortoaccountforthenumberofconnectionsrequiredtotravelfromonelocationtoanother.Thisgametypeisextremelyrare,andasofFebruary2017,aMappinggamewaslastseenonPrepTest40administeredinJune2003.

ProcessGame

Page 199: LSAT PrepTest 81 Unlocked · Similarly, PT 81 was the first test since June '07 (PT June '07) to require at least 94 questions correct to get a 172 (99th percentile score). Number

Agamethatopenswithaninitialarrangementofentities(e.g.,astartingsequenceorgrouping)andprovidesrulesthatdescribetheprocessesthroughwhichthatarrangementcanbealtered.Thequestionstypicallyaskyouforacceptablearrangementsorplacementsofparticularentitiesafterone,two,orthreestagesintheprocess.Occasionally,aProcessgamequestionmightprovideinformationaboutthearrangementafterone,two,orthreestagesintheprocessandaskyouwhatmusthavehappenedintheearlierstages.Thisgametypeisextremelyrare,andasofNovember2016,aProcessgamewaslastseenonPrepTest16administeredinSeptember1995.However,therewasaProcessgameonPrepTest80,administeredinDecember2016,thusendinga20-yearhiatus.

GameSetupsandDeductions

FloaterAnentitythatisnotrestrictedbyanyruleorlimitationinthegame

BlocksofEntitiesTwoormoreentitiesthatarerequiredbyruletobeadjacentorseparatedbyasetnumberofspaces(Sequencinggames),tobeplacedtogetherinthesamegroup(Distributiongames),tobematchedtothesameentity(Matchinggames),ortobeselectedorrejectedtogether(Selectiongames)

LimitedOptionsRulesorrestrictionsthatforceallofagame’sacceptablearrangementsintotwo(oroccasionallythree)patterns

EstablishedEntitiesAnentityrequiredbyruletobeplacedinonespaceorassignedtooneparticulargroupthroughouttheentiregame

Page 200: LSAT PrepTest 81 Unlocked · Similarly, PT 81 was the first test since June '07 (PT June '07) to require at least 94 questions correct to get a 172 (99th percentile score). Number

NumberRestrictionsRulesorlimitationsaffectingthenumberofentitiesthatmaybeplacedintoagrouporspacethroughoutthegame

DuplicationsTwoormorerulesthatrestrictacommonentity.Usually,theserulescanbecombinedtoreachadditionaldeductions.Forexample,ifyouknowthatBisplacedearlierthanAinasequenceandthatCisplacedearlierthanBinthatsequence,youcandeducethatCisplacedearlierthanAinthesequenceandthatthereisatleastonespace(thespaceoccupiedbyB)betweenCandA.

MasterSketchThefinalsketchderivedfromthegame’ssetup,rules,anddeductions.LSATexpertspreservetheMasterSketchforreferenceastheyworkthroughthequestions.TheMasterSketchdoesnotincludeanyconditionsfromNew-“If”questionstems.

LogicGamesQuestionTypes

AcceptabilityQuestionAquestioninwhichthecorrectanswerisanacceptablearrangementofalltheentitiesrelativetothespaces,groups,orselectioncriteriainthegame.Answerthesebyusingtherulestoeliminateanswerchoicesthatviolatetherules.

PartialAcceptabilityQuestionAquestioninwhichthecorrectanswerisanacceptablearrangementofsomeoftheentitiesrelativetosomeofthespaces,groups,orselectioncriteriainthegame,andinwhichthearrangementofentitiesnotincluded

Page 201: LSAT PrepTest 81 Unlocked · Similarly, PT 81 was the first test since June '07 (PT June '07) to require at least 94 questions correct to get a 172 (99th percentile score). Number

intheanswerchoicescouldbeacceptabletothespaces,groups,orselectioncriterianotexplicitlyshownintheanswerchoices.AnswerthesethesamewayyouwouldanswerAcceptabilityquestions,byusingtherulestoeliminateanswerchoicesthatexplicitlyorimplicitlyviolatetherules.

MustBeTrue/False;CouldBeTrue/FalseQuestionAquestioninwhichthecorrectanswermustbetrue,couldbetrue,couldbefalse,ormustbefalse(dependingonthequestionstem),andinwhichnoadditionalrulesorconditionsareprovidedbythequestionstem

New-“If”QuestionAquestioninwhichthestemprovidesanadditionalrule,condition,orrestriction(applicableonlytothatquestion),andthenaskswhatmust/couldbetrue/falseasaresult.LSATexpertstypicallyhandleNew-“If”questionsbycopyingtheMasterSketch,addingthenewrestrictiontothecopy,andworkingoutanyadditionaldeductionsavailableasaresultofthenewrestrictionbeforeevaluatingtheanswerchoices.

RuleSubstitutionQuestionAquestioninwhichthecorrectanswerisarulethatwouldhaveanimpactidenticaltooneofthegame’soriginalrulesontheentitiesinthegame

RuleChangeQuestionAquestioninwhichthestemaltersoneoftheoriginalrulesinthegame,andthenaskswhatmust/couldbetrue/falseasaresult.LSATexpertstypicallyhandleRuleChangequestionsbyreconstructingthegame’ssketch,butnowaccountingforthechangedruleinplaceoftheoriginal.Thesequestionsarerareonrecenttests.

RuleSuspensionQuestion

Page 202: LSAT PrepTest 81 Unlocked · Similarly, PT 81 was the first test since June '07 (PT June '07) to require at least 94 questions correct to get a 172 (99th percentile score). Number

Aquestioninwhichthestemindicatesthatyoushouldignoreoneoftheoriginalrulesinthegame,andthenaskswhatmust/couldbetrue/falseasaresult.LSATexpertstypicallyhandleRuleSuspensionquestionsbyreconstructingthegame’ssketch,butnowaccountingfortheabsentrule.Thesequestionsareveryrare.

CompleteandAccurateListQuestionAquestioninwhichthecorrectanswerisalistofanyandallentitiesthatcouldacceptablyappearinaparticularspaceorgroup,oralistofanyandallspacesorgroupsinwhichaparticularentitycouldappear

CompletelyDetermineQuestionAquestioninwhichthecorrectanswerisaconditionthatwouldresultinexactlyoneacceptablearrangementforalloftheentitiesinthegame

Supplythe“If”QuestionAquestioninwhichthecorrectanswerisaconditionthatwouldguaranteeaparticularresultstipulatedinthequestionstem

Minimum/MaximumQuestionAquestioninwhichthecorrectansweristhenumbercorrespondingtothefewestorgreatestnumberofentitiesthatcouldbeselected(Selection),placedintoaparticulargroup(Distribution),ormatchedtoaparticularentity(Matching).Often,Minimum/MaximumquestionsbeginwithNew-“If”conditions.

Earliest/LatestQuestionAquestioninwhichthecorrectansweristheearliestorlatestpositioninwhichanentitymayacceptablybeplaced.Often,Earliest/LatestquestionsbeginwithNew-“If”conditions.

Page 203: LSAT PrepTest 81 Unlocked · Similarly, PT 81 was the first test since June '07 (PT June '07) to require at least 94 questions correct to get a 172 (99th percentile score). Number

“HowMany”QuestionAquestioninwhichthecorrectansweristheexactnumberofentitiesthatmayacceptablybeplacedintoaparticulargrouporspace.Often,“HowMany”questionsbeginwithNew-“If”conditions.

READINGCOMPREHENSION

StrategicReading

RoadmapThetesttaker’smarkupofthepassagetextinStep1(ReadthePassageStrategically)oftheReadingComprehensionMethod.TocreatehelpfulRoadmaps,LSATexpertscircleorunderlineKeywordsinthepassagetextandjotdownbrief,helpfulnotesorparagraphsummariesinthemarginoftheirtestbooklets.

Keyword(s)inReadingComprehensionWordsinthepassagetextthatrevealthepassagestructureortheauthor’spointofviewandthushelptesttakersanticipateandresearchthequestionsthataccompanythepassage.LSATexpertspayattentiontosixcategoriesofKeywordsinReadingComprehension:

Emphasis/Opinion—wordsthatsignalthattheauthorfindsadetailnoteworthyorthattheauthorhaspositiveornegativeopinionaboutadetail;anysubjectiveorevaluativelanguageontheauthor’spart(e.g.,especially,crucial,unfortunately,disappointing,Isuggest,itseemslikely)

Contrast—wordsindicatingthattheauthorfindstwodetailsorideasincompatibleorthatthetwodetailsillustrateconflictingpoints(e.g.,but,yet,despite,ontheotherhand)

Page 204: LSAT PrepTest 81 Unlocked · Similarly, PT 81 was the first test since June '07 (PT June '07) to require at least 94 questions correct to get a 172 (99th percentile score). Number

Logic—wordsthatindicateanargument,eithertheauthor’sorsomeoneelse’s(e.g.,thus,therefore,because,itfollowsthat)

Illustration—wordsindicatinganexampleofferedtoclarifyorsupportanotherpoint(e.g.,forexample,thisshows,toillustrate)

Sequence/Chronology—wordsshowingstepsinaprocessordevelopmentsovertime(e.g.,traditionally,inthepast,today,first,second,finally,earlier,subsequent)

Continuation—wordsindicatingthatasubsequentexampleordetailsupportsthesamepointorillustratesthesameideaasthepreviousexample(e.g.,moreover,inaddition,also,further,alongthesamelines)

MarginNotesThebriefnotesorparagraphsummariesthatthetesttakerjotsdownnexttothepassageinthemarginofthetestbooklet

BigPictureSummaries:Topic/Scope/Purpose/MainIdeaAtesttaker’smentalsummaryofthepassageasawholemadeduringStep1(ReadthePassageStrategically)oftheReadingComprehensionMethod.LSATexpertsaccountforfouraspectsofthepassageintheirbigpicturesummaries:

Topic—theoverallsubjectofthepassage

Scope—theparticularaspectoftheTopicthattheauthorfocuseson

Purpose—theauthor’sreasonormotiveforwritingthepassage(expressthisasaverb;e.g.,torefute,tooutline,toevaluate,tocritique)

MainIdea—theauthor’sconclusionoroveralltakeaway;ifthepassagedoesnotcontainanexplicitconclusionorthesis,youcancombinetheauthor’sScopeandPurposetogetagoodsenseoftheMainIdea.

Page 205: LSAT PrepTest 81 Unlocked · Similarly, PT 81 was the first test since June '07 (PT June '07) to require at least 94 questions correct to get a 172 (99th percentile score). Number

PassageTypesKaplancategorizesReadingComprehensionpassagesintwoways,bysubjectmatterandbypassagestructure.

Subjectmattercategories

InthemajorityofLSATReadingComprehensionsections,thereisonepassagefromeachofthefollowingsubjectmattercategories:

Humanities—topicsfromart,music,literature,philosophy,etc.

NaturalScience—topicsfrombiology,astronomy,paleontology,physics,etc.

SocialScience—topicsfromanthropology,history,sociology,psychology,etc.

Law—topicsfromconstitutionallaw,internationallaw,legaleducation,jurisprudence,etc.

Passagestructurecategories

ThemajorityofLSATReadingComprehensionpassagescorrespondtooneofthefollowingdescriptions.Thefirstcategories—Theory/PerspectiveandEvent/Phenomenon—havebeenthemostcommononrecentLSATs.

Theory/Perspective—Thepassagefocusesonathinker’stheoryorperspectiveonsomeaspectoftheTopic;typically(thoughnotalways),theauthordisagreesandcritiquesthethinker’sperspectiveand/ordefendshisownperspective.

Event/Phenomenon—Thepassagefocusesonanevent,abreakthroughdevelopment,oraproblemthathasrecentlyarisen;whenasolutiontotheproblemisproposed,theauthormostoftenagreeswiththesolution(andthatrepresentsthepassage’sMainIdea).

Page 206: LSAT PrepTest 81 Unlocked · Similarly, PT 81 was the first test since June '07 (PT June '07) to require at least 94 questions correct to get a 172 (99th percentile score). Number

Biography—Thepassagediscussessomethingaboutanotableperson;theaspectoftheperson’slifeemphasizedbytheauthorreflectstheScopeofthepassage.

Debate—Thepassageoutlinestwoopposingpositions(neitherofwhichistheauthor’s)onsomeaspectoftheTopic;theauthormaysidewithoneofthepositions,mayremainneutral,ormaycritiqueboth.(ThisstructurehasbeenrelativelyrareonrecentLSATs.)

ComparativeReadingApairofpassages(labeledPassageAandPassageB)thatstandinplaceofthetypicalsinglepassageexactlyonetimeineachReadingComprehensionsectionadministeredsinceJune2007.ThepairedComparativeReadingpassagessharethesameTopic,butmayhavedifferentScopesandPurposes.OnmostLSATtests,amajorityofthequestionsaccompanyingComparativeReadingpassagesrequirethetesttakertocompareorcontrastideasordetailsfrombothpassages.

QuestionStrategies

ResearchCluesAreferenceinaReadingComprehensionquestionstemtoaword,phrase,ordetailinthepassagetext,ortoaparticularlinenumberorparagraphinthepassage.LSATexpertsrecognizefivekindsofresearchclues:

LineReference—AnLSATexpertresearchesaroundthereferencedlines,lookingforKeywordsthatindicatewhythereferenceddetailswereincludedorhowtheywereusedbytheauthor.

ParagraphReference—AnLSATexpertconsultsherpassageRoadmaptoseetheparagraph’sScopeandPurpose.

Page 207: LSAT PrepTest 81 Unlocked · Similarly, PT 81 was the first test since June '07 (PT June '07) to require at least 94 questions correct to get a 172 (99th percentile score). Number

QuotedText(oftenaccompaniedbyalinereference)—AnLSATexpertchecksthecontextofthequotedtermorphrase,askingwhattheauthormeantbyitinthepassage.

ProperNouns—AnLSATexpertchecksthecontextoftheperson,place,orthinginthepassage,askingwhethertheauthormadeapositive,negative,orneutralevaluationofitandwhytheauthorincludeditinthepassage.

ContentClues—Theseareterms,concepts,orideasfromthepassagementionedinthequestionstembutnotasdirectquotesandnotaccompaniedbylinereferences.AnLSATexpertknowsthatcontentcluesalmostalwaysrefertosomethingthattheauthoremphasizedoraboutwhichtheauthorexpressedanopinion.

ReadingCompQuestionTypes

GlobalQuestionAquestionthatasksfortheMainIdeaofthepassageorfortheauthor’sprimaryPurposeinwritingthepassage.Typicalquestionstems:

Whichoneofthefollowingmostaccuratelyexpressesthemainpoint

ofthepassage?

Theprimarypurposeofthepassageisto

DetailQuestionAquestionthataskswhatthepassageexplicitlystatesaboutadetail.Typicalquestionstems:

Accordingtothepassage,somecriticshavecriticizedGilliam’sfilms

onthegroundsthat

Thepassagestatesthatoneroleofamunicipality’scomptrollerin

budgetdecisionsbythecitycouncilisto

Page 208: LSAT PrepTest 81 Unlocked · Similarly, PT 81 was the first test since June '07 (PT June '07) to require at least 94 questions correct to get a 172 (99th percentile score). Number

Theauthoridentifieswhichoneofthefollowingasacommonlyheld

butfalsepreconception?

Thepassagecontainssufficientinformationtoanswerwhichofthe

followingquestions?

Occasionally,thetestwillaskforacorrectanswerthatcontainsadetailnotstatedinthepassage:

Theauthorattributeseachofthefollowingpositionstothe

FederalistsEXCEPT:

InferenceQuestionAquestionthatasksforastatementthatfollowsfromorisbasedonthepassagebutthatisnotnecessarilystatedexplicitlyinthepassage.SomeInferencequestionscontainresearchclues.ThefollowingaretypicalInferencequestionstemscontainingresearchclues:

Basedonthepassage,theauthorwouldbemostlikelytoagreewith

whichoneofthefollowingstatementsaboutunifiedfieldtheory?

Thepassagesuggestswhichoneofthefollowingaboutthebehaviorof

migratorywaterfowl?

Giventheinformationinthepassage,towhichoneofthefollowing

wouldradiocarbondatingtechniqueslikelybeapplicable?

OtherInferencequestionslackresearchcluesinthequestionstem.Theymaybeevaluatedusingthetesttaker’sBigPictureSummaries,ortheanswerchoicesmaymakeitclearthatthetesttakershouldresearchaparticularpartofthepassagetext.ThefollowingaretypicalInferencequestionstemscontainingresearchclues:

Itcanbeinferredfromthepassagethattheauthorwouldbemost

likelytoagreethat

Page 209: LSAT PrepTest 81 Unlocked · Similarly, PT 81 was the first test since June '07 (PT June '07) to require at least 94 questions correct to get a 172 (99th percentile score). Number

Whichoneofthefollowingstatementsismoststronglysupportedby

thepassage?

OtherReadingComprehensionquestiontypescategorizedasInferencequestionsareAuthor’sAttitudequestionsandVocabulary-in-Contextquestions.

LogicFunctionQuestionAquestionthataskswhytheauthorincludedaparticulardetailorreferenceinthepassageorhowtheauthorusedaparticulardetailorreference.Typicalquestionstems:

Theauthorofthepassagementionsdeclininginner-citypopulations

intheparagraphmostlikelyinorderto

Theauthor’sdiscussionofRimbaud’stravelsintheMediterranean

(lines23–28)functionsprimarilyto

Whichoneofthefollowingbestexpressesthefunctionofthethird

paragraphinthepassage?

LogicReasoningQuestionAquestionthatasksthetesttakertoapplyLogicalReasoningskillsinrelationtoaReadingComprehensionpassage.LogicReasoningquestionsoftenmirrorStrengthenorParallelReasoningquestions,andoccasionallymirrorMethodofArgumentorPrinciplequestions.Typicalquestionstems:

Whichoneofthefollowing,iftrue,wouldmoststrengthentheclaim

madebytheauthorinthelastsentenceofthepassage(lines51–55)?

Whichoneofthefollowingpairsofproposalsismostclosely

analogoustothepairofstudiesdiscussedinthepassage?

Author’sAttitudeQuestion

Page 210: LSAT PrepTest 81 Unlocked · Similarly, PT 81 was the first test since June '07 (PT June '07) to require at least 94 questions correct to get a 172 (99th percentile score). Number

Aquestionthatasksfortheauthor’sopinionorpointofviewonthesubjectdiscussedinthepassageoronadetailmentionedinthepassage.Sincethecorrectanswermayfollowfromthepassagewithoutbeingexplicitlystatedinit,someAuthor’sAttitudequestionsarecharacterizedasasubsetofInferencequestions.Typicalquestionstems:

Theauthor’sattitudetowardtheuseofDNAevidenceintheappeals

byconvictedfelonsismostaccuratelydescribedas

Theauthor’sstanceregardingmonetaristeconomictheoriescanmost

accuratelybedescribedasoneof

Vocabulary-in-ContextQuestionAquestionthataskshowtheauthorusesawordorphrasewithinthecontextofthepassage.Thewordorphraseinquestionisalwaysonewithmultiplemeanings.Sincethecorrectanswerfollowsfromitsuseinthepassage,Vocabulary-in-ContextquestionsarecharacterizedasasubsetofInferencequestions.Typicalquestionstems:

Whichoneofthefollowingisclosestinmeaningtotheword

“citation”asitusedinthesecondparagraphofthepassage(line18)?

Incontext,theword“enlightenment”(line24)refersto

WrongAnswerTypesinRC

OutsidetheScope(OutofScope;BeyondtheScope)Ananswerchoicecontainingastatementthatistoobroad,toonarrow,orbeyondthepurviewofthepassage

180Ananswerchoicethatdirectlycontradictswhatthecorrectanswermustsay

Page 211: LSAT PrepTest 81 Unlocked · Similarly, PT 81 was the first test since June '07 (PT June '07) to require at least 94 questions correct to get a 172 (99th percentile score). Number

ExtremeAnanswerchoicecontaininglanguagetooemphatic(e.g.,all,never,every,none)tobesupportedbythepassage

DistortionAnanswerchoicethatmentionsdetailsorideasfromthepassagebutmanglesormisstateswhattheauthorsaidaboutthosedetailsorideas

FaultyUseofDetailAnanswerchoicethataccuratelystatessomethingfromthepassagebutinamannerthatincorrectlyanswersthequestion

Half-Right/Half-WrongAnanswerchoiceinwhichoneclausefollowsfromthepassagewhileanotherclausecontradictsordeviatesfromthepassage

FORMALLOGICTERMS

ConditionalStatement(“If”-ThenStatement)Astatementcontainingasufficientclauseandanecessaryclause.ConditionalstatementscanbedescribedinFormalLogicshorthandas:

If[sufficientclause]→[necessaryclause]

Insomeexplanations,theLSATexpertmayrefertothesufficientclauseasthestatement’s“trigger”andtothenecessaryclauseasthestatement’sresult.

Formoreonhowtointerpret,describe,anduseconditionalstatementsontheLSAT,pleasereferto“ANoteAboutFormalLogicontheLSAT“inthisbook’sintroduction.

Page 212: LSAT PrepTest 81 Unlocked · Similarly, PT 81 was the first test since June '07 (PT June '07) to require at least 94 questions correct to get a 172 (99th percentile score). Number

ContrapositiveTheconditionalstatementlogicallyequivalenttoanotherconditionalstatementformedbyreversingtheorderofandnegatingthetermsintheoriginalconditionalstatement.Forexample,reversingandnegatingthetermsinthisstatement:

resultsinitscontrapositive:

If~B→~A

Toformthecontrapositiveofconditionalstatementsinwhicheitherthesufficientclauseorthenecessaryclausehasmorethanoneterm,youmustalsochangetheconjunctionandtoor,orviceversa.Forexample,reversingandnegatingthetermsandchangingandtoorinthisstatement:

IfM→OANDP

resultsinitscontrapositive:

If~OOR~P→~M